Sunteți pe pagina 1din 252

Revision

On

Gyna & Obstetrics


2008

By/

Dr.Adel Abdel Hamid

Index
1- All gyna & obstetrics last year exams classified according to chapters..( 4 )
2- Examples of short questions.( 11 )
3- Mcqs & false or true questions on gyna & obstetrics classified according to chapters.( 18 )
4- Mcqs exams...(218)
Exam 1 ..(219)
Exam 2 ..( 227)
Exam 3 ..(232 )
Exam 4 ..(239)

Gyna & Obstetrics Reveision by/Dr.adool

2008

Contents

This book contains


1- 1st part
All gyna & obstetrics last year exams classified according to chapters

2- 2nd part
Examples of short questions

3- 3rd part
Mcqs & false or true questions on gyna & obstetrics classified according to chapters

4- 4th part
Mcqs exams

Gyna & Obstetrics Reveision by/Dr.adool

2008

1st part

A)Gynecology
1)Anatomy & Embryology
1-Congenital anomalies of Uters

96

2-Anatomy of Cervix & Cervical ligaments.

99

3-Anatomy & anomalies of Corpus Uteri

2000

4-Anatomy of Fallopian tubes.

98

5- Anatomy of Fallopian tubes & its role in infertility 2001


6- Anatomy of Broad ligament.

2002

7- Anatomy of Perineal body

2003

8-Embryology of Vulva

2003

2)Reproductive development of female genital tract


1-Amenorrhea

98

2-Uterine causes of Amenorrhea 95, 96, 98, 2007


3-Causes & investigation of Ovarian Amenorrhea.
4-Spasmodic dysmenorrhea.

99, 2001

2003

5-Postpartum Amenorrhea 2003


-D.D

2006

Causes & Discuss one Pathological cause.


6-Cryptomenorrhea

2003

2004

7-Aetiology & investigations of bleeding in early pregnancy.

98

8-Precocious puberty 2003, 2007


9-Tubal factor of infertility 96
10-Cervical causes of infertility 98

Gyna & Obstetrics Reveision by/Dr.adool

2008

11-Ovarian factor of infertility

2007

12-Diagnosis &TTT of anovulation 2006


13-Diagnosis of ovulation

96

14-Induction of ovulation

98

15-Postmenopausal bleeding diag. & TTT


16-Complications of ovarian cyst

2003

98, 2007

17-Testicular Feminization Syndrome.

2002

3)Reproductive Tract Infection


1-Gardenella vaginalis 95
2-Vaginal discharge (Leucorrhea) 2002
-Investigations 96, 99, 2003, 2006
-Defintion, causes 2003, 2006
3-Trichomons Vaginalis 98, 2004
-Diagnosis & TTT 96
4-Bacterial vaginosis 98, 2001
5-1ry vaginitis 2003
6-Aetiology & management of recurrent vaginal moniliasis.

2000

4) Displacement, Trauma & Urogynecology.


1-Vesicovaginal fistulae
-Causes 96, 98, 2004
-Diagnosis 96,, 203, 2004, 2006, 2007
-TTT 2006
2-Postoperative care of repaired Vesicovaginal fistulae 2003
3-Definition & types of female urinary incontinence
4-Investigation of true urinary incontinence
5-Uretrovaginal fistula

2003

2002

98

Gyna & Obstetrics Reveision by/Dr.adool

2008

6-Uetrovaginal support

2002

7-Genital prolapse
-Recurrent Genital prolapse 2003, 2007
Causes of prolapse 96, 20048-Etiology & diagnosis of uterine prolapse 2006
9-Diagnosis & mang. Of old complete perineal tear
10-Pregnancy of RVF uterus

2006, 2007

2002

5) Diseases & Swelling of Vulva & Vagina


1-Vaginal cyst

2001

2-D.D of Vulval Ulcers

2003, 2007

6)Benign tumors of Uterus & Endometriosis


1-S&S of Adenomyosis Uteri
2-Endometriosis

96

96

-Diag. & TTT 99


-Mang. Of pelvic endometriosis 2000
3-Complications of Benign Ovarian tumors

7)Oncology
1-Aetiological factors & diagnosis of uterine body carcinoma

95

2-Chemotherapy "indication& complication" in gynecological practice

95

3-Cervical intraepithelial neoplasia CIN 98, 2003, 2006


-Diagnosis & TTT 96, 2002
4- Diagnosis & TTT of Uterine carcinoma 96
5- Diagnosis & TTT of endometrial carcinoma 98
Mang. 2001-Epidmiology, staging & prognosis 2003
6-Spread of cervical carcinoma 2003, 2004

Gyna & Obstetrics Reveision by/Dr.adool

2008

7-Clinical staging of cervical carcinoma 2004, 2006, 2007


8-Complications of ovarian tumors 2003
9-Early detection of cancer cervix
10-Complications of ovarian cyst

98, 2007

8)Contraception & Fertility control


1-Comp. of IUD 2002, 2003, 2006
Mang. 2006
2-Emergency contraception 2003, 2006, 2007

B)Obestetrics
1)Normal Pregnancy
1-Placental functions & morphology
2-D.D of Pregnancy

96

96

3-Cardiovascular adaptations during normal pregnancy 2001, 2003, 2007


4-Diag. of pregnancy in 1st trimester 2001, 2002
5-Instructions given to pregnant lady at her 1st antenatal care visit

2003

6-Vomiting in early pregnancy causes & mang. 2006


7-S&S of early pregnancy

2)Abnormal pregnancy
1-Ovarian pregnancy

95

2-Diagnosis of Molar pregnancy

2003

3-Diag. of advanced extrauterine pregnancy

2003

4-Aetiology & Investigations of bleeding in early pregnancy 98


5-Abnormal vaginal bleeding in childbearing period

2002

6-Accidental Hge
-Mang. 98

Gyna & Obstetrics Reveision by/Dr.adool

2008

-Causes & diagnosis 99


7-Hazards of abruption placentae 2002
8-Mang. of placenta previa 2006
9-Habitual abortion

2003

-Etiology 2004, 2007


-Diag. 96, 2004, 2007
-TTT 96

& Mang.

10-Polyhydramnios

98
98

11-Preeclampsia
-Cardiovascular changes in severe preeclampsia 2002
-Severe Preeclampsia 2002
-Diag. & TTT

2003

-Mild Preeclampsia Diag. & TTT

2006

-Mang. Of preeclampsia remote of term 2003


12-Eclampsia
-Diag. & Ttt of eclamptic fit 96
-Mang. 2006
-TTT 2004, 2007

3)Normal Labor
1-Mang. Of 2nd stage normal labor 96, 2004
2-Mang. Of 3rd stage normal labor 2007
3-Pain relief in labor 98
4)Malpresentation & Malpositioning.
1-Mang. Of persistent occipitotransverse
2-Deep transverse arrest of fetal head 96, 2003
3-Deep transverse arrest of fetal face.

2002

4-Diag. & Mang. Of Face presentation

98

Gyna & Obstetrics Reveision by/Dr.adool

2008

5-Mang. Of breech presentation in labor 98


6-Fetal hazards in breech delivery 2003, 2007
7-Arrest of fetus after coming in breech presentation.
8-Complicated breech delivery

2001

2000

9-Charcters of vaginal delivery in breech presentation 2006


10-Neglected shoulder 2003
11-Mechanism of labor in occipitoposterior position

2000

12-Occipitoposterior position of fetal head 2006

5)Abnormal Labor
1-Atonic postpartum haemorrage

2004

-Predisposing factors & Mang. 95


-Causes

96

2-Causes & diag. of 1ry postpartum Hge 96


3-2ry postpartum Hge.

98, 2003

4-Retained placenta

2000

-Causes & TTT 2006


5-Mang. Of pelvic outlet contraction 98
6-Obestetric pelvic outlet
7-Postpartum collapse

2002
2001

8-PROM
-Definition 2003, 2007

& -Diag. 2007

-Mang. 98, 2003, 2007


9-Hypofibrinogenemia in relation to obstetrics 2003
10-Complications of 3rd stage labor

98

11-Etiology & mang. of preterm ( premature) labor


12-Impending rupture of the uterus

98, 2000

2002, 2007

13-Uterine rupture during pregnancy 2003

Gyna & Obstetrics Reveision by/Dr.adool

2008

6)Fetus
1- IUFD
-Diag. & TTT 96

&

-Complications

-Mang. Of IFUD at 30 week of pregnancy


2-Fetal circulation

2003
95

96

3-Prenatal screening & diagnosis of fetal anomalies.


4-Intrapartum fetal monitoring

96

99, 2003, 2007

5-Tests for fetal maturity 2000


6-Antepartum evaluation of fetal morphology & genetics 2001
7-Biophysical evaluation of fetal health
8-Fetal health evaluation

2006

2006

9-Diagnosis of intrapartum fetal distress

2003

10-Fetal head birth injuries 2003

7)Neonates
1-Neonatal asphyxia

2006

8)Operative
1-Epsiotomy
2-Indications & complications of craniotomy

96

3-Indications & contraindications of vaccum extractions "Ventose"


4-Indications & compl. Of CS
5-Induction of labor

96

2004, 2007

96

-Methods & indications

99

9)Miscellaneous
1-Amniocentesis, therapeutic & diagnostic values

10

2002

Gyna & Obstetrics Reveision by/Dr.adool

2008

2nd part

Short Questions
1-Differences between the upper and the lower uterine segments:
2-Cardiovascular changes with pregnancy
3-Urinary changes with pregnancy
4-Drug categories during pregnnacy according to FDA classification:
5-Causes of Bleeding in early pregnancy
6-AETIOLOGY OF SPONTANEOUS ABORTION
7-TYPES OF ABORTION
8-Management of:
- Threatened abortion:
- Inevitable abortion
- INCOMPLETE ABORTION
- missed abortion
- SEPTIC ABORTION
9-AETIOLOGY OF RECURRENT ABORTION (RPL)
10-Antiphospholipid Syndrome
11-MANAGEMENT OF CASES WITH RPL
12-Diagnosis of isthmic incompetence:
13-Treatment of isthmic incompetence:
14-AETIOLOGY of TUBAL PREGNANCY
15-PATHOLOGY of tubal Ectopic pregnancy
16-CLINICAL PICTURE of tubal Ectopic pregnancy
17-TREATMENT of TUBAL PREGNANCY
18-PATHOLOGY of hydatidiform mole:

11

Gyna & Obstetrics Reveision by/Dr.adool

2008

19-CLINICAL FEATURES of hydatidiform mole


20-TREATMENT of hydatidiform mole
21-FOLLOW-UP of hydatidiform mole
22-CAUSES of Antepartum haemorrhage
23-CLINICAL DIAGNOSIS OF PLACENTA PRAEVIA
24-MANAGEMENT OF APH due to PLACENTA PRAEVIA
25-AETIOLOGY Of ACCIDENTAL HAEMORRHAGE
26-DIAGNOSIS OF ACCIDENTAL HAEMORRHAGE
27-MANAGEMENT OF APH due to ACCIDENTAL HAEMORRHAGE
28-Complications of accidental haemorrhage
29-MANAGEMENT of hyperemesis gravidarum
30-CLINICAL PICTURE OF PREECLAMPSIA
31-COMPLICATIONS OF PREECLAMPSIA
32-Criteria of severity of PE
33-PREVENTION and TREATMENT OF PREECLAMPSIA
34-Effect of Pregnancy on Diabetes
35-Effects of Diabetes on Pregnancy
36-Management OF DIABETES WITH PREGNANCY
37-Management OF CARDIAC DISEASES WITH PREGNANCY
38-Management OF Rh Incompatibility (Erythroblastosis Fetalis)
39-Management OF asymptomatic bacteruria WITH PREGNANCY
40-ENGAGEMENT
41-MANAGEMENT of occipito-posterior
42-MANAGEMENT of face presentation
43-Management Of Brow Presentation
44-DIAGNOSIS OF BREECH PRESENTATION
45-MANAGEMENT options in breech presentations

12

Gyna & Obstetrics Reveision by/Dr.adool

2008

46-Indications of CAESAREAN SECTION IN BREECH PRESENTATION


47-Management Of transverse lie
48-Management Of Cord prolapse
49-MANAGEMENT OF TWIN PREGNANCY
50-Indications of C.S. in multifetal pregnancy
51-Complications of multifetal pregnancy
52-MANAGEMENT OF Hypotonic inertia
53-MANAGEMENT OF Hypertonic inertia
54-MANAGEMENT OF Precipitate labor
55-MANAGEMENT OF CONTRACTION (CONSTRICTION) RING
56-MANAGEMENT OF CERVICAL DYSTOCIA
57-MANAGEMENT of contracted pelvis
58-Cephalopelvic disproportion (CPD) tests
59-Management of labour in contracted pelvis
60-Trial of labour (TOL)
61-Indications of Caesarean section in contracted pelvis:
62-Management of OBSTRUCTED LABOUR
63-MANAGEMENT OF RUPTURE UTERUS
64-MANAGEMENT OF CERVICAL LACERATIONS
65-Management of PRIMARY POST PARTUM HAEMORRHAGE
66-Management of Retained placenta
67-Management of Acute inversion of the uterus
68-Assessment of Antepartum: fetal wellbeing
69-Assessment of Intrapartum: fetal wellbeing
70-INTRAUTERINE GROWTH RESTRICTION (IUGR)
71-FETAL MACROSOMIA
72-Management of Preterm labour:

13

Gyna & Obstetrics Reveision by/Dr.adool

2008

73-Management of Post-term pregnancy


74-Management of Spontaneous Prelabour Rupture Of the Membranes
75-Management of OLIGOHYDRAMNIOS
76-Management of POLYHYDRAMNIOS
77-PHYSIOLOGICAL CHANGES IN THE PUERPERIUM
78-PUERPERAL PYREXIA
79-Management of PUERPERAL SEPSIS
80-METHODS OF INDUCTION OF LABOUR
81-METHODS OF INDUCTION OF ABORTION
82-The Bishop score
83-Classification of forceps operations
84-Indications for the use of forceps
85-Prerequisites before forceps application
86-COMPLICATIONS OF FORCEPS PROCEDURES
87-Classification of forceps operations
88-Indications for the use VACUUM EXTRACTOR
89-Prerequisites before VACUUM EXTRACTOR
90-COMPLICATIONS OF VACUUM EXTRACTOR
91-Episiotomy
92-INDICATIONS OF C.S
93-Advantages f the L.S.C.S
94-Causes of MATERNAL MORTALITY
B)Gynecology
95-Causes of infertility
96-Investigations of infertile couples
97-Treatment of infertility
98-CLASSIFICATION OF URINARY INCONTINENCE

14

Gyna & Obstetrics Reveision by/Dr.adool

2008

99-Management of stress incontinence


100-Management of Menopause
101-Causes of 2ry amenorrhea
102-Causes of 1ry amenorrhea
103-Diagnosis of amenorrhea
104-Treatment of amenorrhea
105-Primary dysmenorrhea
106-Secondary dysmenorrhea
107-Premenstrual tension syndrome
108-AETIOLOGY OF ANOVULATION:
109-DIAGNOSIS OF OVULATION
110-TREATMENT OF ANOVULATION
111-Management of PCO
112-Management of LPD
113-Management of Hirsutism
114-Management of:
-Trichomonas Vaginitis
-Monilia Vaginitis (Candidiasis)
-BACTERIAL VAGINOSIS
(Non-specific Vaginitis, Gardnerella Vaginitis)
-Acute Cervicitis
-Chronic Cervicitis
-Cervical Erosion
-PELVIC INFLAMMATORY disease (PID)
-Endometrosis
-GENITAL PROLAPSE
115-AETIOLOGY OF GENITAL PROLAPSE

15

Gyna & Obstetrics Reveision by/Dr.adool

2008

116-AETIOLOGY OF Perineal Lacerations AETIOLOGY OF Rectovaginal Fistula


117-Management of Perineal Lacerations Management OF Rectovaginal Fistula
118-AETIOLOGY OF VESICOVAGINAL FISTULA
119-Management of VESICOVAGINAL FISTULA
120-Management of URETERO-VAGINAL FISTULA
121-AETIOLOGY OF URETERO-VAGINAL FISTULA
122-Management of UTERINE LEIOMYOMATA
123-DIAGNOSIS OF CIN
124-TREATMENT OF CIN
125-PATHOLOGY Of INVASIVE CARCINOMA OF THE CERVIX
126-STAGING OF CERVICAL CARCINOMA
127-Management of INVASIVE CARCINOMA OF THE CERVIX
128-Management of Endometrial Hyperplasia
129-PATHOLOGY OF ENDOMETRIAL CARCINOMA
130-GRADING AND STAGING OF ENDOMETRIAL CARCINOMA
140-Management OF ENDOMETRIAL CARCINOMA
141-PATHOLOGY OF CHORIOCARCINOMA
142-Management OF CHORIOCARCINOMA
143-non neopl. cysts OF THE OVARY
144-CLASSIFICATION OF OVARIAN NEOPLASMS
145-Pathology of benign ovarian tumours
146-Management of benign ovarian tumours
147-Pathology of malignant ovarian tumours
148-Management of malignant ovarian tumours
149-COMPLICATIONS OF BENIGN OVARIAN NEOPLASMS
150-Surgical staging of primary ovarian cancer:
151-Complications of IUD

16

Gyna & Obstetrics Reveision by/Dr.adool

2008

152-Contraindications for IUCD


153-Complications of OCPS
154-Contraindications for OCP
155-Complications of Injectable Contraceptives
156-Contraindications for Injectable Contraceptives
157-Contraception Methods For Lactating Females

17

Gyna & Obstetrics Reveision by/Dr.adool

2008

3rd part

GYNa MCQs
Index
(1) *Anatomy & Embreology &Physiology 19
(2) * Amenorrhea . 29
(3) * Dysmenorrhea . 35
(4) *Abnormal genital tract bleeding 41
(5) * Ovulation and its disorders 51
(6) * Infertility & Hirsutism . 70
(7) * Displacements, traumatic lesions, urogynecology .. 76
*

Genitourinary fistula .. 76

URINARY INCONTINENCE . 82

Old complete perineal tear 86

(8) * Diseases of vulva& Infection 88


(9) * Endometriosis . 93
(10) * CANCER CERVIX .. 93
(11) * CERVIX ... NONE NEOPLASTIC LESIONS .. 111
(12) * Endometrial carcinoma .. 113
(13) * Choriocarcinoma .. 122
(14) * Ovarian tumours ............................................................................................................. 125
(15) * The ovary .. 136
(16) * Family planning and contraception .. 140

18

Gyna & Obstetrics Reveision by/Dr.adool

2008

(1) *Anatomy & Embreology &Physiology


1) In the embryology of genital organs what is the false?

a) Ovary from the genital ridge


b) Tubes, uterus and whole vagina from Wolffian ducts
c) Clitoris from genital tubercle
d) Labia minors from genital folds
e) Labia majora from genital swellings
2) What is false in failure of the longitudinal fusion Mullerian ducts?

a) Double (bodies, cervices) = uterus didelphys


b) Double bodies and one cervix = bicornuate unicollis
c) Intra-uterine septum = Septate & Subseptate
d) Fundal depression = Arcuate uterus
3) What is the false in anatomy of vulva?
a) Nerves: pudenda!, perineal, Genito-femoral and ilio-inguinal
b) Lymphatics to inguinal lymph nodes
c) Totally covered with keratinized epithelium
d) Bartholin's and Skenes glands open in vestibule
e) Blood vessels are pudendals (internal and external)
4) What is false about the hymen?
a) Incomplete membrane covered with stratified epithelium
b) Between upper and lower vagina
c) Annular, crescenteric, biperforate, cribriform and imperforate
d) Its remnants are carunculae myrtiformes
5) The vagina in a young woman has the followings except:
a) Both anterior and posterior walls = 8 cm
b) Consists of mucosa, submucosa, musculosa and adventitia

19

Gyna & Obstetrics Reveision by/Dr.adool

2008

c) Is supported by multiple ligaments and muscles


d) Has 4 types of lining cells which are sensitive to steroid hormones
6) What is false in the following anatomical features of the uterus?
a) Corpus is the main part
b) Fundus is the part above insertion of the round ligaments
c) Isthmus is between the anatomical os (above) and the histological (below)
d) Isthmus forms the lower segment of pregnancy
e) Mobile RVF uterus is normal in 15-20%
7) What is the false in anatomy of the broad ligaments?
a) Peritoneal leaves from the uterine side to the pelvic side
b) True ligaments contain two false ligaments (round & ovarian)
c) Contain tubes, ligaments (round & ovarian), vessels, nerves and Wolffian remnants
d) All the contents are extraperitoneal
8) The ovary has the following characters except:
a) 2 poles (upper & lower)
b) 2 surfaces (medial & lateral
c) 2 borders (anterior & posterior
d) 2 veins drain into the inferior vena cava (IVC)
e) 3 areas (cortex, medulla & hilum) and 3 x 2 x 1 cm
9) The non-pregnant cervix has the following anatomical features except:
a) One spindle shape canal with mucus folds [arbor vitae uteri]
b) One cm length
c) Two [portions, orifices & types of epithelium]
d) Three [layers (mucosa, muscle & adventitia) and ligaments]
10) The normal menstruation needs the followings except:
a) Normal hypothalamic pituitary ovarian functions .
b) Patent uterine tubes

20

Gyna & Obstetrics Reveision by/Dr.adool

2008

c) Healthy responsive endometrium (to steroids)


d) Patent outflow tract
11) Progesterone leads to the following effects except:
a) Secretory and decidual endometrial changes with myometrial relaxation
b) Viscid scanty cervical mucus
c) Proliferation of the vaginal squamous cells
d) Proliferation of the glandular breast tissue
12) LH surge stimulates the followings except:
a) Completion of meiosis of primary oocyte
b) Lutenization of granulosa cells
c) Synthesis of follicular progesterone, prostaglandins and proteolytic enzymes
d) Ovulation
e) None of above
13) Regarding hypoplasia of the uterus the followings art true except:

a) Uterine index < I


b) May cause oligomenorrhea, hypomenorrhea, amenorrhea,& spasmodic dysmenorrhea
c)Is of three types
d) Diagnosed by sounding &US
e) Treated by cyclic estrogen and progestin therapy
14) The imperforate hymen is characterized by the followings except:

a)

Bulging bluish membrane

b)

Monthly abdominal pain

c)

Collection of menstrual blood as hemato (colpos, metra and salpinx) )

d)

Causes true amenorrhea

e)

Treated by cruciate incision

21

Gyna & Obstetrics Reveision by/Dr.adool

2008

15) What is false in anatomy of the Labia minora?


a) Thick folds of hairy skin with sebaceous glands and fat
b) Divided anterior into the prepuce (above) and the frenulum (below) the clitoris
c) United posterior to form the fourchette
d) Contain erectile vascular and sensitive tissue
16) The pelvic floor includes the followings except:
a) From perineal skin up to peritoneum of the Douglas pouch
b) Perineal muscles (deep & superficial) and membranes
c) Levator ani & coccygeus and pelvic fascia
d) Obturator internus muscle
17) What is false in anatomy of the adult nulliparous uterus?
a) 4 parts: fundus, corpus, isthmus and cervix
b) 3 layers: endometrium, myometrium and perimetrium
c) 3 blood supply: uterine, ovarian and vaginal
d) 3 inches (length) x 2 breadth (at fundus) x one thickness (add inch for each in multipara).
e) Has thin myometrium
18) The cervix has the following anatomical features except:
a) Contains glands
b) Portiovaginalis and supravaginalis
c) The squamo-columnar junction is the origin of neoplasia
d) Sensitive for traction and dilatation and insensitive for cutting and heat
19) What is the false about the ovary during reproductive life?
a) Receives 2 blood supply (ovarian & uterine)
b) Has 2 functions (ova and steroid hormones)
c) Contains 2 types of follicles (functioning & degenerated)
d) Attached by 2 ligaments (ovarian & infundibulo-pelvic)
e) Has peritoneal covering

22

Gyna & Obstetrics Reveision by/Dr.adool

2008

20) What is false in the clinical effects of the following abnormalities?


a) Bicomuate uterus may cause habitual abortion &oblique lie
b) Rudimentary horn may cause ectopic pregnancy
c) Uterine hypoplasia causes menorrhagia
d) Cervical incompetence causes habitual abortion
21) Regarding the innervations of the genital organs the followings true except?
a) Only autonomic nerve supply
b) The sympathetic from the thoracic and the lumbar roots
c) The parasympathetic from the sacral roots
d) The nerve plexuses are the superior & inferior hypogastric
22) The followings are associations of ovulation except:
a) L.H. surge
b) Follicular prostaglandins synthesis
c) Oocyte reduction division
d) Reduction of the follicle size with follicular fluid in the Douglas pouch
e) Depression [reduction] in FSH
23) Estrogen has the following effects except:
a) Proliferation of the endometrium
b) Growth of ducts and stroma of the breast
c) Growth of the clitoris and pubic hair
d) Production of a copious watery thin cervical mucus
e) Proliferation of the vaginal squamous cells
24) Ovulation is due to:
a) FSH surge
b) Progesterone synthesis
c) Increased follicular prostaglandins and proteolytic enzymes
d) Increased androgens synthesis

23

Gyna & Obstetrics Reveision by/Dr.adool

2008

25) The embryological causes of cryptomenorrhea include Ike followings except:

a)

Imperforate hymen

b)

Turner syndrome

c)

Complete transverse vaginal septum

d)

Vaginal aplasia.

e)

Cervical atresia

26) What is the false about the congenital vaginal anomalies in females?

a)

Aplasia of upper 4/5 of vagina, uterus and tubes (Rokitansky syndrome)

b)

Transverse and longitudinal septa

c)

Congenital fistulae with the bladder and the rectum

d)

Short blind pouch in testicular feminization syndrome (TFS)

27) The clitoris has the following features except:


a) Cavernous erectile sensitive organ attached to front of the pubis by the suspensory ligament
b) Estrogen dependant organ
c) Consists of glans & body
d) Excised during circumcision
28) What is false about the perineal body?
a)

Pyramidal fibro-muscular mass between tower vagina & rectum

b)

Contains decussating fibers of levator ani, superficial &deep perineal muscles

c)

Includes the anterior part of external anal sphincter

d)

Surgically incised (episiotomy) or torn during some labors

29) What is the false in the anatomy of the uterus?


a)

In center of the true pelvis

b)

Has 5 ligaments.

c)

Forward bending of cervix on vagina by 90 (anteversion)

d)

Forward bending of corpus on cervix by 160 (anteflexion)

e)

Internal os is at the level of the ischial spine

24

Gyna & Obstetrics Reveision by/Dr.adool

2008

30) What is the false in anatomy of the broad ligaments?


e) Peritoneal leaves from the uterine side to the pelvic side
f)

True ligaments contain two false ligaments (round & ovarian)

g) Contain tubes, ligaments (round & ovarian), vessels, nerves and Wolffian remnants
h) All the contents are extraperitoneal
31) The ovary has the following characters except:
f)

2 poles (upper & lower)

g) 2 surfaces (medial & lateral


h) 2 borders (anterior & posterior
i)

2 veins drain into the inferior vena cava (IVC)

j)

3 areas (cortex, medulla & hilum) and 3 x 2 x 1 cm

32) The non-pregnant cervix has the following anatomical features except:
e) One spindle shape canal with mucus folds [arbor vitae uteri]
f)

One cm length

g) Two [portions, orifices & types of epithelium]


h) Three [layers (mucosa, muscle & adventitia) and ligaments]
33) The normal menstruation needs the followings except:
e) Normal hypothalamic pituitary ovarian functions .
f)

Patent uterine tubes

g) Healthy responsive endometrium (to steroids)


h) Patent outflow tract
34) L.H. has the following actions except:
a) Lutenization of the granulosa and theca cells
b) Stimulation of progesterone [P] synthesis
c) Stimulation of androgen synthesis
d) Production of L.H. receptors on the granulosa cells

25

Gyna & Obstetrics Reveision by/Dr.adool

2008

35) What is false in the effects of excess androgens in the adult female?
a) Atrophy of the secondary sex organs
b) Oligomenorrhea then amenorrhea
c) Hirsutism, deepening of the voice & clitoromegaly
d) All the above changes are irreversible
36) Normal menstruation is characterized by the followings except:
a) Regular every >3-5 weeks
b) Average duration 3-5 days (2-7 range)
c) Unclotted blood loss about 25-80 ml
d) The first one is menarche at age of 11-13 years
e) Usually it is asymptomatic [silent]
37) What is false in the following arteries of the genital organs ?
a)

The ovarian from the aorta

b)

The uterine from the internal iliac

c)

The vaginal and the internal pudendal from the int. iliac [to the vagina

d)

The external pudendal from the int. iliac [to the vulva]

38) Mechanism of menstruation includes the followings except:


a) Constriction of the spiral arteries [PG]
b) Endometrial ischemia
c) Endometrial desquamation & shedding
d) Excessive estrogen secretion
39) The luteal phase has the following features except:
a) Low basal body temperature [uni-phasic curve]
b) Secretory changes of the endometrium
c) Progesterone [P] dependant
d) Predominance of the intermediate vaginal cells
e) Fixed length (12-14 days)

26

Gyna & Obstetrics Reveision by/Dr.adool

2008

40) The average duration of normal menstrual cycle is:


a) 5 weeks one week
b) 40 days
c) 3 weeks one week
41) What is false about the pelvic portion of the ureter?
a)

Retroperitoneal with 15 cm length

b)

Enters the pelvis infront of the sacro-iliac joint and common iliac artery bifurcation

c)

At level of ischial spine passes medially and forwards under uterine artery

d)

Pierces Mackenrodt's ligament 5 cm lateral to the cervix

42) Fallopian tubes have the following anatomical features except:


a) 2 [in number, functions &blood supply]
b) 3 [layers & types of the lining cells]
c) 4 [parts & inches in length]
d) Complete peritoneal covering
43) What is false in the following anatomical features of the uterus?
f)

Corpus is the main part

g)

Fundus is the part above insertion of the round ligaments

h)

Isthmus is between the anatomical os (above) and the histological (below)

i)

Isthmus forms the lower segment of pregnancy

j)

Mobile RVF uterus is normal in 15-20%

44) What is the fake in the anatomy of the vagina?


a)

Elastic fibro -muscular canal from vulva to cervix

b)

Directed up and backwards by 60-70 angle on horizon

c)

Lined with keratinized stratified squamous epithelium with multiple glands

d)

Has multiple blood supply [vaginal, uterine, vesical and rectal (mid. & inferior) arteries]

50) What is the false about anatomy of Bartholin's gland?


a) Two racemose pea sized glands with columnar epithelium

27

Gyna & Obstetrics Reveision by/Dr.adool

2008

b) Palpable at junction of upper 2/3 and lower 1/3 of L. minora


c) Secrete lubricant serous fluid during coitus
d) Duct is lined by transitional epithelium
51) What is the false about the Wolffian remnants?

a) Koblet's tubules, epoophoron, paroophoron & Gartner's duct


b) Give to paroovarian cyst in the broad ligament
c) Give to Gartner's cyst in the anterolateral vaginal wall
d) Give to Skene's cyst
52) The followings can cause pregnancy loss and malpresentations except:

a)

Uterine hypoplasia

b)

Cervical incompetence

c)

Septate & bicornuate uteri

d)

Imperforate hymen

(2) * Amenorrhea,
1) The fallowings cause true amenorrhea [A] except:
(a) Alternations in hypothalamic-pituitary-ovarian axis
(b) Endometrial pathology
(c) Obstructed outflow tract
(d) Systemic disease [auxiliary factors]
2) What is false in the following definitions?
(a) Absence of menses up to age of 16 with developed secondary characters = primary one
(b) Absence of menses up to age of 14 without developed secondary sex characters = primary one
(c) Delayed period for 2 weeks = secondary type
(d) Missed periods for 3 cycles = secondary type
(e) Eumenorrhea means normal menstruation

28

Gyna & Obstetrics Reveision by/Dr.adool

2008

3) What is false in classifications of amenorrhea?


(a) Physiological, pathological
(b) Primary, secondary
(c) False (crypto.), true
(d) Cervical, vaginal & vulvar
(e) Hypothalamic, pituitary, ovarian, uterine
4) What is false in cause of physiological amenorrhea?
(a) Before puberty (low GnRH )
(b) During pregnancy (high estrogens and progesterone)
(c) During lactation (high oxytocin)
(d) After menopause (low estrogens)
5) What is false about the primary amenorrhea?
(a) Of hypothalamic causes
(b) Of pituitary causes
(c) Of ovarian & uterine causes
(d) Means absent periods after menarche
(e) Caused also by imperforate hymen
6)Cryptomenorrhea occurs with the fallowings except:
(a) Imperforate hymen
(b) Vaginal aplasia
(c) Incomplete transverse vaginal septum
(d) Cervical obstruction [congenital & acquired]
7) What is false in causes of the true amenorrhea?
(a) C.N.S. and hypothalamus
(b) Pituitary
(c) Ovarian
(d) Uterine

29

Gyna & Obstetrics Reveision by/Dr.adool

2008

(e) No other causes more than the above


8) The CNS and hypothalamic amenorrhea include the fallowings except:
(a) Psychological (anorexia nervosa, pseudocyesis, emotional trauma)
(b) Organic lesions (tumors, trauma, meningitis)
(c) Galactorrhea amenorrhea syndromes
(d) Infantilism
(e) Postpill amenorrhea
9) Pituitary amenorrhea includes the fallowings except:
(a) Simmond & Sheehan syndrome
(b) Adenomas
(c) Pituitary infantilism
(d) Halpans disease
10) Physiological ovarian amenorrhea is due to:
(a) Congenital agenesis and genetic (Turner's)
(b) Excision, destruction (surgery, TB, malignancy, irradiation)
(c) Postmenopausal absence of primordial follicles
(d) Virilizing ovarian tumors (Arrhenoblastoma)
11) Uterine amenorrhea includes the fallowings except:
(a) Congenital aplasia, and severe hypoplasia
(b) Removal & destruction
(c) Septate uterus
(d) Endometrial receptor defect
(e) Asherman's syndrome
12) Galactorrhea [G] amenorrhea [A] syndromes include the fallowings except:
(a) Chiari-Frommel
(b) Froehlichs syndrome
(c) Forbs & Albright's

30

Gyna & Obstetrics Reveision by/Dr.adool

2008

(d) Del Castillo's


13) In Asherman 's syndrome what is false?
(a) Intra uterine synechiae (endometrial trauma plus infection)
(b) Clinically: amenorrhea, refractory endometrium, infertility, abortion, and placenta accreta
(c) Diagnosed by laparoscopy
(d) Cut adhesions by resectoscope leave IUD, give estrogens
14) What is false in Cushing's syndrome?
(a) Pituitary acidophil adenoma
(b) Secondary amenorrhea & infertility
(c) Moon face, hirsutism, hypertension, diabetes,....
(d) May due to suprarenal hyperplasia & neoplasia
I5) Sheehan's syndrome has the fallowing features except:
(a) Complete or incomplete anterior pituitary necrosis due to severe postpartum bleeding
(b) Lactational amenorrhea
(c) Panhypopituitrism ->low (gonadotropins, prolactin, ACTH, TSH) -> hypofunction of the target
glands
(d) Needs hormonal replacement therapy
16) What is false in features of Frohlich 's syndrome?
(a) Hypothalamic pituitary defect
(b) Primary amenorrhea with genital hypoplasia
(C) Generalized body obesity
(d) Needs cyclic hormonal therapy
17) Kallmann's syndrome has the following characters except:
(a) Pituitary defect
(b) Anosmia
(c) Primary amenorrhea
(d) Sterility

31

Gyna & Obstetrics Reveision by/Dr.adool

2008

18) What is false about galactorrhea?


(a) Always is hyperprolactinemic
(b) iatrogenic
(c) Physiological
(d) Pathological
19) Pituitary prolactinoma has the fallowings except:
(a) Micro (<2 cm) or macroadenoma (>2 cm)
(b) Prolactin level > 100 ng/ml
(c) Galactorrhea, secondary amenorrhea, may blurring of vision
(d) Diagnosed by C.T
20) Hyperprolactinemia may lead to the fallowings except:
(a) Secondary amenorrhea, infertility
(b) Galactorrhea
(c) Luteal phase defect
(d)Hirsutism
21) Empty sella syndrome has the fallowing features except:
(a) Congenital dural defect in roof of sella diagnosed by MRI
(b) Pressure of C.S.F. on pituitary with widening of its fossa
(c) CSF rhinorrhea, galactorrhea
(d) Hypofunction of thyroid and suprarenal glands
22) Turner's syndrome has the following features except:
(a) Negative buccal smear (45XO)
(b) Long stature
(c) Primary amenorrhea
(d) Sterility
23) Athletic amenorrhea has the fallowing features except:
(a) Amenorrhea, oligomenorrhea

32

Gyna & Obstetrics Reveision by/Dr.adool

2008

(b) Excess catecholamines, prolactin, endorphin, and cortisone


(c) Normal ovarian function
(d) Underweight
24) A woman 47 year old with amenorrhea 7 month and sensation of harness on chest & face especially
during night, what is the possibility?
(a) Pregnancy
(b) Pituitary tumor
(c) Asherman's syndrome
(d) Postmenopausal
25) A woman 32 year old with missed period for 2 weeks all the fallowings
should be done except:
(a) Clinical examination
(b) Serum pregnancy test
(c) Pelvic ultrasound
(d) Hysteroscopic examination
26) A girl 17-year-old has never menstruated, she is dwarf, underdeveloped breast and scanty pubic hair,
what is the possible cause of amenorrhea?
(a) Imperforate hymen
(b) Uterine aplasia
(c) Pituitary infantilism, or Turner's syndrome
(d) Asherman's syndrome
27) A girl 18-year-old comes to you informing that she has never menstruated with monthly lower
abdominal pain, by examination she has well-developed body, breast and pubic hair with Suprapubic
cystic mass what is the possibility?
(a) Hypothalamic-pituitary ovarian defect
(b) Uterine aplasia
(c) Imperforate hymen
(d) Testicular feminization syndrome (TFS)

33

Gyna & Obstetrics Reveision by/Dr.adool

2008

28) A woman 38-year-old comes to gynecological clinic with amenorrhea 4 month, milky nipple discharge
with blurring of vision and persistent headache, by examination she has a normal sized uterus what is
the possibility?
(a) Pregnancy
(b) Pituitary macroadenoma
(c) Hypothalamic dysfunction
(d) Premature menopause
29) A woman 29 year old comes to you presented with uterine bleeding for 12 days after absent periods
for 3 cycles, by US examination she has an enlarged empty uterus with unilateral cystic ovary,
pregnancy test is negative what is the possibility?
(a) Ectopic pregnancy
(b) DUB (metropathia hemorrhagica)
(c) Threatened abortion
(d) Feminizing ovarian tumor
30) What is false in causes of galactorrhea [G]?
(a) Always pathological
(b) iatrogenic
(c) Prolactinoma
(d) Empty sella syndrome
31) Investigations of amenorrhea include:
(a) History and examination
(b) Laboratory
(c) Imaging technique
(d) Therapeutic tests
(e) All of above
32) What is false in evaluation of a girl with primary amenorrhea?
(a) Look for the height either normal or abnormal
(b) Look for the genital & somatic development

34

Gyna & Obstetrics Reveision by/Dr.adool

2008

(c) Do US, karyotyping pituitary necrosis due to severe g and hormonal assays
(d) Local examination is not needed
33) What is false in treatment of amenorrhea?
(a) Correction of the general factors as anemia ,obesity,
(b) Correction of the causal factors as removal of a tumor
(c) Cyclic estrogen and progestogen therapy for all cases
(d) Give ovulating drugs in cases with infertility

(3) * Dysmenorrhea,
1) Dysmenorrhea is:
(a) Heavy menstruation
(b) Painful menstruation
(c) Irregular menstruation
(d) Pain not related to menses
2) The common type of dysmenorrhea is:
(a) Crescendo (endometriosis)
(b) Membranous
(c) Spasmodic or congestive
(d) Ovarian
3) Spasmodic dysmenorrhea has the followings except:
(a) Intermittent lower abdominal colicky pain radiating to back
(b) Starts few hours premenstrual, persists for 1-3 days
(c) Maximum pains during the first menstrual day, then improves
(d) PV or PR examination pelvic pathology
4) Primary dysmenorrhea has the followings except:
(a) Starts with menarche
(b) Improves after labor
(c) Colicky pain may with low backache, nausea & vomiting an

35

Gyna & Obstetrics Reveision by/Dr.adool

2008

(d) Treated by PG synthetase inhibitors & spasmolytics


5) The most accepted etiology of primary dysmenorrhea is:
(a) Cervical stenosis
(b) Uterine hypoplasia
(c) Excess prostaglandins (PGF2a)
(d) Psychological
6) The suitable therapy for spasmodic dysmenorrhea includes:
(a) Reassurance, anti PG, spasmolytics
(b) >l Ovulation (pills, estrogen, androgens)
(c) Cervical dilatation (up to 12 Hegar)
(d) Laser uterine nerve ablation (LUNA)
7) Crescendo dysmenorrhea has the followings except:
(a) Present with myoma
(b) Starts premenstrual
(c) More severity during menses
(d) Needs causal treatment
8) Membranous dysmenorrhea is:
(a) Common
(b) Painful menses with a cervical membrane
(c) Pain with total endometrial shedding as a membrane
(d) Pain with endometriosis
9) The main treatment of membranous dysmenorrhea is:
(a) Pelvic decongestants
(b) Sedatives & analgesics
(c) Cervical dilatation
(d) Anti prostaglandins + repeated D & C.

36

Gyna & Obstetrics Reveision by/Dr.adool

2008

10) Spasmodic dysmenorrhea occurs with:


(a) Membranous endometrial shedding
(b) IUD
(c) Passage of uterine clots
(d) Bicornuate uterus
(e) Any of above
11) Unilateral dysmenorrhea does not occur in:
(a) Unilateral endometriosis
(b) Corneal unilateral myoma
(c) Uterine prolapse
(d) Rudimentary horn
12) Congestive (secondary) dysmenorrhea is only induced by:
(a) Any cause of pelvic congestion
(b) Constipation
(c) Adenomyosis, endometriosis
(d) IUIX PID, adhesions
(e) Neoplasm, displacement
13) What is false in congestive dysmenorrhea?
(a) Localized lower abdominal colic
(b) 2-3 days premenstrual
(c) Relief by onset of menstruation
(d) Radiation to back
14) Ovarian pain includes the fallowings except:
(a) Ovarian dysmenorrhea
(b) Tubal ectopic pregnancy
(c) Corpus luteum hematoma
(d) Mittel Schmerz

37

Gyna & Obstetrics Reveision by/Dr.adool

2008

15) Investigations of secondary dysmenorrhea include:


(a) CBP, ESR
(b) Endoscopy
(c) Pelvic sonogram
(d) Bacteriological cultures
(e) All of above
16) The therapy of congestive dysmenorrhea includes:
(a) Causal TTT
(b) Pelvic decongestants
(c) Anti-prostaglandins
(d) Daily cold baths, correction of faulty environment
(e) All of above
17) Low backache has the following characters except:
(a) Extra-genital or genital causes
(b) Genital backache is localized and radiates to the heel
(c) Mainly of non-genital origin (bone, muscle, joints,.)
(d) May be caused by pelvic congestion
18) What is false in premenstrual tension syndrome (PMS) ?
(a) Premenstrual (5-11 days) symptoms
(b) Has underlying pelvic pathology
(c) Physical symptoms as backache, headache, mastalgia,....
(d) Psychological symptoms as irritability, depression, anxiety
19) Theories of PMS include the fallowings except:
(a) Increased B endorphin
(b) + E / P ratio
(c) Renin-angiotensin-aldosterone
(d) Pelvic pathology

38

Gyna & Obstetrics Reveision by/Dr.adool

2008

20) Treatment of PMS includes the fallowings except:


(a) Sedatives, tranquilizers
(b) Progestogen
(c) Cervical dilatation
(d) Primrose oil
21) What is false in pelvic pain ?
(a) Acute or chronic
(b) Chronic pain is cyclic & acyclic
(c) Dysmenorrhea is acyclic pain
(d) Endometriosis leads to acute & chronic pelvic pain
(e) Gynecological & non-gynecological pain
22) What is False in nervous transmission of pelvic pain?
(a) Parasympathetic: S2,3 & 4
(b) Sympathetic: TIO - LI
(c) Pudendal - 82,3 & 4
(d) Vagina : only autonomic nerves
23) Clinical type of pelvic pain is:
(a) Cramping, colicky
(b) Constant
(c) Sharp
(d) Dull
(e) Any of above
24) Pain is induced by:
(a) Distension, contraction, obstruction, rupture, infection of a hollow organ
(b) Stretch, torsion, ischaemia, inflammation of a solid organ
(c) Peritoneal irritation
(d) Perforation, hemorrhage

39

Gyna & Obstetrics Reveision by/Dr.adool

2008

(e) All of above


25) Epigastric pain suggests a problem in:
(a) Stomach
(b) Duodenum
(c) Pancreas
(d) Liver & gall bladder
(e) Any of above
26) Peri-umbilical pain suggests a problem in the following organs except:
(a) Liver & gall bladder
(b) Small intestine
(c) Appendix
(d) Upper ureters
27) Suprapubic (hypogastric) pain suggests a problem in the following organs except:
(a) Uterus
(b) Urinary bladder
(c) Colon
d) Upper ureters

40

Gyna & Obstetrics Reveision by/Dr.adool

2008

(4) *Abnormal genital tract bleeding,


1) The normal menstruation needs the followings except:
(a) Normal hypothalamic pituitary ovarian functions
(b) Patent uterine tubes
(c) Healthy responsive endometrium (to steroids)
(d) Patent outflow tract
(e) Healthy body
2) Mechanism of menstruation includes the followings except:
(a) Constriction of the spiral arteries [PG]
(b) Endometrial ischaemia
(c) Endometrial desquamation & shedding
(d) Excessive estrogen secretion
5) The Proliferative (follicular) phase has the following features except:
(a) Fixed length
(b) Growth of ovarian follicles
(c) Secretion of estrogens
(d) Proliferation of endometrium
(e) FSH dependant
4) The followings are associations of ovulation except:
(a) L.H. surge
(b) Follicular prostaglandins synthesis
(c) Oocyte reduction division
(d) Reduction of the follicle size with follicular fluid in the Douglas pouch
(e) Depression [reduction] in FSH
5) L.H. has the following actions except:
(a) Lutenization of the granulosa and theca cells
(b) Stimulation of progesterone [P] synthesis

41

Gyna & Obstetrics Reveision by/Dr.adool

2008

(c) Stimulation of androgen synthesis


(d) Production of L.H. receptors on the granulosa cells
6) The luteal phase has the following features except:
(a) Low basal body temperature [uni-phasic curve]
(b) Secretory changes of the endometrium
(c) Progesterone [P] dependant
(d) Predominance of the intermediate vaginal cells
(e) Fixed length (12-14 days)
7) Progesterone leads to the following effects except:
(a) Secretory and decidual endometrial changes with myometrial relaxation
(b) Viscid scanty cervical mucus 1
(c) Proliferation of the vaginal squamous cells
(d) Proliferation of the glandular breast tissue
8) Estrogen has the following effects except:
(a) Proliferation of the endometrium
(b) Growth of ducts and stroma of the breast
(c) Growth of the clitoris and pubic hair
(d) Production of a copious watery thin cervical mucus
(e) Proliferation of the vaginal squamous cells
9) What is false in the effects of excess androgens in the adult female?
(a) Atrophy of the secondary sex organs
(b) Oligomenorrhea then amenorrhea ..
(c) Hirsutism, deeping of the voice & clitoromegaly
(d) All the above changes are irreversible
10) The average duration of normal menstrual cycle is:
(a) 5 weeks one week
(b) 40 days

42

Gyna & Obstetrics Reveision by/Dr.adool

2008

(c) 3 weeks one week


(d) 4 weeks one week
11) LH surge stimulates the followings except:
(a) Completion of meiosis of primary oocyte
(b) Lutenization of granulosa cells
(c) Synthesis of follicular progesterone, prostaglandins and proteolytic enzymes
(d) Ovulation
(e) None of above
12) Ovulation is due to:
(a) FSH surge
(b) Progesterone synthesis
(c) Increased follicular prostaglandins and proteolytic enzymes
(d) Increased androgens synthesis
13) Normal menstruation is characterized by the followings except:
(a) Regular every >3-5 weeks
(b) Average duration 3-5 days (2-7 range)
(c) Unclotted blood loss about 25-80 ml
(d) The first one is menarche at age of 11-13 years
(e) Usually it is asymptomatic [silent]
14) Polymenorrhea (epimenorrhea) has the followings except:
(a) Frequent cyclic menstruation less than 4 weeks
(b) Normal amount of menstrual blood loss
(c) Due to dysfunction of hypothalamic-pituitary ovarian axis
(d) TTT: luteal phase progestin support or cyclic E & P therapy
15) Menorrhagia (hypermenorrhea) has the followings except:
(a) Cyclic regular heavy menstruation (3/28)
(b) May prolonged periods (8/28) with normal amount

43

Gyna & Obstetrics Reveision by/Dr.adool

2008

(c) May with heavy arid prolonged period


(d) Always is of organic etiology
16) Metrorrhagia has the following characters except:
(a) Irregular amount
(b) Acyclic bleeding
(C) Often prolonged in duration
(d) Usually dysfunctional
17) What is the false in characters of hypomenorrhea?
(a) Menses < 2 days
(b) <30 ml blood loss
(c) Short cycle (<3w)
(d) Regular cycle
I8) What is false in classifications of abnormal uterine bleeding?
(a) According to the age (neonatal,.... up to postmenopausal)
(b) According to the origin (ovarian, uterine, cervical, & vaginal)
(c) According to the causes (dysfunctional, organic, and iatrogenic)
(d) According to the clinical form (menorrhagia, metrorrhagia)
19) A girl 7 years old with genital bleeding the cause may be one of the followings except:
(a) Trauma
(b) Precocious puberty
(c) Granulosa & theca cell tumor
(d) Arrhenoblastoma
(e) Sarcoma botryoids
20) To emergency room a woman 47 years old comes to you with postmenopausal bleeding, by examination
she has a small sized uterus and she gives a history of hormonal therapy to treat hot flashes, what is the
possible cause?
(a) Myoma
(b) Malignancy of genital tract

44

Gyna & Obstetrics Reveision by/Dr.adool

2008

(c) Systemic disease


(d) Hormonal withdrawal bleeding
21) One of the followings can cause postmenopausal bleeding:
(a) Pre-invasive cervical carcinoma
(b) Benign ovarian teratoma
(c) Endometrial carcinoma
(d) Atrophic subserous fibroid
22) What is false in dysfunctional uterine bleeding (DUB)?
(a) Ovular & non-ovular
(b) Cyclic and acyclic
(c) Ovarian dysfunction
(d) Can be caused by a small Myoma
23) Ovular (cyclic) DUB includes the followings except:
(a) Poor corpus luteum (CL) function-irregular endometrial ripeningmenorrhagia
(b) Slow degeneration of C.L. irregular sheddingmenorrhagia
(c) Short phase (follicular or luteal) Polymenorrhea
(d) Persistent Graffian follicleoligomenorrhea
24) Recent classification of DUB includes the followings except:
(a) Hypothalamic-pituitary-ovarian dysfunctionprimary DUB
(b) General causes of bleeding secondary DUB
(c) Drugs, steroids, IUD, iatrogenic DUB
(d) Genital infection pelvic congestionDUB
25) What is false in the features of metropathia hemorrhagica?
(a) Acyclic anovular bleeding due to persistent Graffian follicles
(b) Cystic glandular endometrial hyperplasia
(c) Painless prolonged bleeding after short amenorrhea (6-8 weeks)
(d) Bilateral cystic ovaries with excess androgens & hirsutism

45

Gyna & Obstetrics Reveision by/Dr.adool

2008

26) Threshold bleeding has the following features except:


(a) Irregular anovular bleeding in early and late reproductive life
(b) Excess estrogen
(c) Thin endometrium
(d) Treatment: cyclic therapy of estrogen and progestogen [E&P]
27) All the followings are valuable in diagnosis of abnormal uterine bleeding except:
(a) History and examinations (general, abdominal & local)
(b) Laboratory tests (pregnancy, CBP, hormones,... etc)
(c) Imaging technique (US, MRI) and endoscopy (colposcopy, hysteroscopy & laparoscopy)
(d) D & C without biopsy
(e) Diagnostic and fractional curettage, cytology
28) Choose the suitable line of treatment for abnormal uterine bleeding?
(a) Medical: iron, vitamins, anti-PG, tranexamic acid...)
(b) Hormonal (cyclic E & P, and progestogen for 2nd half, pills)
(c) Surgical (D & C, hysterectomy, ablation)
(d) Induction of artificial menopause
(e) One or more of above according to clinical data of each case
29) What is the dangerous cause of post-menopausal bleeding?
(a) Estrogen withdrawal bleeding
(b) Malignancy
(c) Systemic causes as purpura, anticoagulant therapy
(d) Senile endometritis and vaginitis
(30) Investigations of the postmenopausal bleeding include:
(a) History and examinations
(b) Cytology, biopsy & culture
(c) Endoscopy: colposcopy, hysteroscopy
(d) US, CT, MRI & barium enema

46

Gyna & Obstetrics Reveision by/Dr.adool

2008

(e) All of above

31. Dysfunctional uterine bleeding:


A. May be associated with hypothyroidism.
B. May be associated with postmenopausal bleeding.
C. May be associated with functional ovarian cysts.
D. May present as menorrhagia.
E. May present as metropathia hemorrhagica.
A. True B. False C. True D. True E. True
32. Dysfunctional uterine bleeding:
A. Dysfunctional menorrhagia is usually associated with poor function of the
corpus luteum.
B. Dysfunctional polymenorrhea means excessive but not prolonged menstruation.
C. Dysfunctional polymenorrhea shows tendency to spontaneous correction.
D. Metropathia hemorrhagica is the commonest type of dysfunctional
uterine bleeding.
E. Threshold bleeding occurs when estrogen level reaches high values.
A. True B. False C. True D. True E. False
33. Treatment of dysfunctional uterine bleeding may include:
A. Progestins.
B. Danazol.
C. GnRH analogues.
D. Hysterectomy.
E. Endometrial ablation.
A. True B. False C. False D. True E. True
34. Metropathia hemorrhagica:
A. Is the commonest type of DUB.
B. It is usually due to overproduction of gonadotropins by the pituitary.
C. Is commonly associated with theca lutein cysts.
D. Is associated with a variable period of amenorrhea followed by bleeding.
E. Is associated with endometrial hyperplasia.
A. True B. False C. False D. True E. True
35. Diagnosis of metropathia hemorrhagica:
A. The majority of cases are premenopausal.
B. A common presentation is a variable period of amenorrhea followed by
prolonged bleeding.
C. Ultrasonography usually reveals a follicular cyst.
D. Ultrasonography usually reveals thickened endometrium.
E. The uterus is usually asymmetrically enlarged.
A. True B. True C. True D. True E. False

47

Gyna & Obstetrics Reveision by/Dr.adool

2008

36. Management of metropathia hemorrhagica:


A. Some cases require blood transfusion.
B. Fractional curettage.
C. Oral contraceptive pills are contraindicated.
D. Young patients may benefit from induction of ovulation.
E. Some cases require hysterectomy.
A. True B. True C. False D. True E. True
37. Management of metropathia hemorrhagica:
A. Progestins have no role in treatment.
B. Endometrial ablation is the modern substitute of hysterectomy in all cases.
C. Some cases benefit from thyroxin.
D. Young patients should receive danazol therapy.
E. Antifibrinolytics may be used as adjuvant therapy.
A. False B. False C. True D. False E. True
38. Endometrial ablation:
A. May be an alternative to surgery in some cases of dysfunctional uterine bleeding.
B. May be an alternative to surgery in cases with asymptomatic fibroids.
C. Does not require any preparation.
D. Does not require anesthesia.
E. Is always followed by amenorrhea.
A. True B. False C. False D. False E. False
39. Premenopausal bleeding may be a presentation of:
A. Endometriosis.
B. Adenomyosis.
C. Metropathia hemorrhagica.
D. Complications of pregnancy.
E. Thecoma.
A. True B. True C . True D. True E. False
40. Premenopausal bleeding may be a presentation of:
A. Carcinoma of the cervix.
B. Atrophic endometritis.
C. Fibroids.
D. Cervical erosion.
E. Endometrial carcinoma.
A. True B. False C. True D. True E. True
41. Postmenopausal bleeding may be a presentation of:
A. Carcinoma of the cervix.
B. Atrophic endometritis.
C. Fibroids.
D. Cervical erosion.
E. Endometrial carcinoma.

48

Gyna & Obstetrics Reveision by/Dr.adool

2008

A. True B. True C. False D. True E. True


42. Investigation of a case of postmenopausal bleeding includes:
A. Full history.
B. General examination.
C. Local examination.
D. Vaginal ultrasonography.
E. Fractional curettage.
A. True B. True C. True D. True E. True
43. Menorrhagia:
A. Is a term describing excessive and/or prolonged menstruation.
B. There may be no organic cause.
C. May be associated with use of combined oral contraceptive pills.
D. May be associated with hypertension.
E. May be associated with hypothyroidism.
A. True B. True C. False D. True E. True
44. Menorrhagia may be associated with:
A. Intrauterine contraceptive device.
B. Subserous fibroid.
C. Adenomyosis.
D. Chronic PID.
E. Endometrial carcinoma.
A. True B. False C. True D. True E. False
45. Metrorrhagia:
A. Means irregular bleeding not related to menstruation.
B. The majority of cases are due to local lesions.
C. May be due to irregular use of contraceptive pills.
D. Is not related to dysfunctional uterine bleeding.
E. Is not related to malignant tumors of the genital tract.
A. True B. True C. True D. False E. False
(46)In metropathia hoemorrhagica, the uterine size is:
A) Normal.
B) Decreased.
C) 6 8 weeks pregnancy size.
D) None of the above.
(47)In metropathia hoemorrhagica there is:
A) Normal ovulation.
B) Anovulation.
C) Defective corpus luteum.
(48)The endometrium, in cases of metropathia hoemorrhagica, is:
A) Secretory.
B) Shows swiss cheese appearance.
C) Shows luteal phase defect.

49

Gyna & Obstetrics Reveision by/Dr.adool

2008

(49)Diagnosis of metropathia hoemorrhagica depends upon the following, EXCEPT:


A) Clinical examination.
B) Hysteroscopy.
C) Hysterosalpingoraphy.
D) Ultrasonography.
(50)GnRH analogues are used in the following conditions EXCEPT:
A) Treatment of endometriosis.
B) Treatment of uterine leiomyoma.
C) Treatment of ovarian cystadenoma.
D) Treatment of ovarian polycystic disease.
E) IVF.
(51)Estrogen is used in the following conditions EXCEPT:
A) Postmenopausal syndrome and prevention of osteoporosis.
B) Turners syndrome.
C) Fibroadenome of the breast.
D) Atrophic vaginitis.
E) In contraceptive pills.
(52)Progesterone needs oestrogen-priming in the following condition EXCEPT:
A) Secretory transformation of proliferative endometrium.
B) Alveolr development of glandular-breast tissue.
C) Increased bulk of uterine muscle.
D) Biphasic body temperature.
(53)Progesterone (not progestins) is used in:
A) Contraceptive pills.
B) Injectable contraceptives.
C) Luteal phase defect.
D) Treatment of recurrent cancer endometrium.
E) Treatment of dysfunctional uterine bleeding.
(54)Colposcopy in valuable in detection of epithelial abnormalities in the following organs, EXCEPT:
A) Vulva.
B) Vagina.
C) Cervix uteri.
D) Corpus uteri.

* Say true or false.


a- I.U.D. can cause menorrhagia through disturbance in the level of carbonic anhydrase enzyme.

b- Excessive intake of aspirin and other anti-inflammatory drugs can increase the menstrual blood flow through coagulation
disturbance.
c- Luteal phase defect may present by infertility, habitual abortion or menorrhagia
d- Bleeding in cases of metropathia hemorrhagica is mild following period of amenorrhea.
e-The cycle length in a patient of menorrhagia is prolonged.
f- Pathological changes of the endometrium in cases of abnormal bleeding include endometrial polyps.

50

Gyna & Obstetrics Reveision by/Dr.adool

2008

g-The commonest cause of post-menopausal bleeding is atrophic endometrium


h- Thyroid abnormalities are a very common cause of irregular vaginal bleeding.
i- Dysfunctional uterine bleeding is commonest around the age of 40 years.
j- Curettage of the endometrium in cases of metropathia hemorrhagica shows evidence of luteal phase defect.
k- Bilateral ovarian corpus luteum cyst is common in cases of metropathia hemorrhagica.
l- Induction of ovulation can be tried in cases of metropathia hemorrhagica in young age females.
m- Androgens are effective treatment for dysfunctional uterine bleeding.
n- Progestogens are the main line of treatment for cases of dysfunctional uterine bleeding.
o- Estrogen may be used in treatment of cases of severe dysfunctional bleeding to stop the bleeding by endometrial growth.
p- Cases of delayed shedding are diagnosed by doing curettage postmenstrual on day 7.
q-Hypertension might be a cause of cases of dysfunctional bleeding.
r-the commonest cause of postmenopausal bleeding is malignant tumors.
s-The commonest cause of bleeding at the childbearing period is pregnancy complications.
t- Pelvic congestion may result into menorrhagia and polymenorrhea.

(5) * Ovulation and its disorders


Enumerate : Tests for detection of ovulation :
Enumerate : Causes of luteal phase defect :
1.The following are inconsistent with a diagnosis of PCOS:
A. A-regular 26-day cycle.
B. Normal ovarian morphology on ultrasound scan.
C. LH:FSH ratio <2:1.
D. Normal body mass index.
E. Rapidly progressive Virilization.
2.Polycystic ovarian syndrome:
A. Is associated with UI hyper secretion.
B. Is associated with low TSH levels.
C. Typically presents with primary amenorrhoea.
D. Is associated with increased production of dehydroepiandrosterone sulphate (DHEAS), an adrenal androgen.
E. Is associated with markedly elevated serum prolactin
3. Symptoms suggestive of ovulation include:
A. Irregular cycles.
B. Spasmodic dysmenorrhea.
C. Premenstrual tension syndrome (PMS).
D. Congestive dysmenorrhea.
E. Galactorrhea.

51

Gyna & Obstetrics Reveision by/Dr.adool

2008

4. Acceptable methods for confirmation of ovulation include:


A. Folliculometry.
A. Folliculometry.
B. Positive thread test of cervical mucus.
C. Serum estrogen level on day 14.
D. Serum progesterone on day 21.
E. Premenstrual endometrial biopsy.
5. Luteal phase defect:
A. Is associated with delayed menses.
B. May be due to inadequate progesterone production.
C. Is a common cause of infertility.
D. Is a common cause of recurrent abortion.
E. The endometrium shows weak secretory changes.
6. With luteal phase defect:
A. Cycles may be short.
B. The basal body temperature chart is not biphasic.
C. Ovulatory pain is absent.
D. A defective corpus luteum can be diagnosed by ultrasonography.
E. Luteal support is best provided by progestins.
7. Luteal phase defect may be associated with:
A. Short cycles.
B. Endometriosis.
C. Low estrogen levels.
D. Endometrial biopsy showing weak secretory changes.
E. Low prolactin levels.
8- The female human sexual response cycle includes all expect:
A) excitement phase
B) plateau
C) resolution
D) refractory phase
9- The climacteric , all are true EXCEPT:
A) the average age of the menopause in Egypt has been shown to be 51 years
B) demineralisation of bone takes place after the menopause
C) the level of prolactin rises after the last period
D) the severity of flushing has been shown to be related to the level of oestrogen in the blood
E) clinical studies have shown that the later the menarche the earlier the menopause
10- Barr bodies in the buccal cells of a female are characteristic of:
A) Turners syndrome.
B) Normal female
C) Short stature
D) Sterility.

52

Gyna & Obstetrics Reveision by/Dr.adool

2008

E) Mental retardation.
11- A 50- year-old female complains of fewer periods recently. You might expect her:
A) FSH and LH to be high.
B) FSH and LH to be low.
C) FSH and estrogen to be high.
D) FSH and estrogen to be low.
E) FSH and progesterone to be low.
12- At the midpoint in the menstrual cycle, ovulation is triggered by a sudden seven-fold increase in the amount
of....................
A) estrogen
B) progesterone
C) LH
D) FSH
13- The average menstrual cycle is:
A) 14 days.
B) 18 days.
C) 24 days.
D) 28 days
14- Anovulatory cycles , all are true EXCEPT:
A) are typical of the early years after the menarche.
B) are associated with endometrial hyperplasia.
C) are a risk factor for ovarian cancer.
D) may be due to polycystic ovary syndrome.
E) are associated with menstrual cycles of variable length.
15- A Graafian follicle is one that:
A) is dormant, awaiting a signal to begin to develop
B) has already ruptured and released the ovum
C) is fully mature and ready to rupture
D) has been damaged through exposure to radiation or chemicals
16- What pituitary hormone(s) control ovulation and production of female hormones by the ovary?
A) estrogen
B) gonadotropin releasing hormone
C) human chorionic gonadotropin (HCG)
D) follicle stimulating hormone (FSH) and lutenizing hormone (LH)
E)progesterone
17- In a "typical" 28 day female reproductive cycle, on which day following the beginning of the menstrual period
is the level of luteinizing hormone most likely to be highest?
A) day 5
B) day 7
C) day 14
D) day 21
E) day 28

53

Gyna & Obstetrics Reveision by/Dr.adool

2008

18- During the human menstrual cycle, the body temperature:


A) rises slightly at ovulation
B) falls slightly at ovulation
C) rises slightly at the start of menstruation
D) falls slightly at the start of menstruation

19- Testosterone and other androgens:


A) are found only in men.
B) are found in higher concentrations in men than women.
C) are found in equal concentrations in men and women.
D) are found in higher concentrations in women than men.
E) are found only in women
20- During the early to mid follicular phase of the female reproductive cycle, the direct effect of inhibin is to:
A) inhibit the release of follicle stimulating hormone.
B) inhibit the release of luteinizing hormone.
C) inhibit ovulation.
D) inhibit endometrial proliferation.
E) inhibit uterine contractions.
21- The primary source of progesterone during the reproductive cycle of a non-pregnant female is (are) the:
A) granulosa cells.
B) theca cells.
C) corpus luteum.
D) pituitary.
E) hypothalamus.
22- GnRH, all are false EXCEPT:
A)
B)
C)
D)
E)

is distinct from LH-RH


is produced in the posterior pituitary
is a decapeptide produced in pulses at ninety minute intervals
exerts its main effect directly on the ovary
is used to induce ovulation in IVF programmes

23- The Anterior Pituitary, all are not true EXCEPT:


A) has a portal circulation
B) produces GnRH in pulsatile fashion
C) produces oxytocin
D) produces prolactin, under the influence of prolactin releasing hormone secreted by the hypothalamus
24- Life cycle of Corpus luteum is completed in:
A) 6 - 12 days
B) 12 - 14 days
C) 14 - 18 days
D) 18 - 22 days

54

Gyna & Obstetrics Reveision by/Dr.adool

2008

25- Human ovulation occurs:


A) At mid-luteal phase.
B) On day 10 of a 28-day cycle.
C) 14 days before menstrual bleeding.
D) Before rupture of the corpus luteum.
E) At the end of the secretory phase.
26- During nonnal menstrual cycle. ovulation occurs as a result of
A) excess Estrogen level
B) low Estrogen level
C) surge of leutenizing hormone
D) low level of HCG
27- The following statements concerning the menstrual cycle are true EXCEPT:
A) The follicular phase in the ovary is associated with a rise in the blood levels of estradiol
B) Ovulation is associated with a sudden rise in blood LH levels
C) The proliferative phase of endometrial growth depends on estradiol secretion
D) Ovulation can be prevented by non-steroidal anti-inflammatory drugs
E) During the secretory phase of the uterus, cervical mucus becomes more viscous
28- The squamous epithelium is target tissue for sex hormones. The vaginal epithelium of a 60-year- old female is
similar to that of a:
A) 2-day-old female.
B) 5-year- old female.
C) 11- year- old female in puberty.
D) 21-year-old female.
E) 26-year- old female on oral contraceptives.
29- The endometrium becomes secretory during the second half of the menstrual cycle under the influence of:
A) prolactin
B) oxytocin
C) estrogen
D) progesterone
30-The intercellular chemical signal that is released by cells and has a local effect on the same cell type as that from
which the chemical signal is released is:
A) autocrine chemical signal
B) pheromone
C) paracrine chemical signal
D) hormone
E) neurotransmitter

31- Neurohormones are intercellular chemical signals that are:


A) produced by groups of cells and affect neurons.
B) released by cells and affect other cell types locally.
C) produced by neurons and act like hormones.

55

Gyna & Obstetrics Reveision by/Dr.adool

2008

D) released by neurons and stimulate or inhibit other neurons.


32- Which is released by cells and affect other cell types locally without being transported in blood?
A)
B)
C)
D)
E)

Autocrine chemical signals


Pheromones
Paracrine chemical signals
Hormones
Neurotransmitters

33- The lipid hormones are either.. or derivatives of fatty acids.


A) glycolipids
B) phospholipids
C) steroids
D) triglycerides
E) glycerols
34- Estrogen,progesterone, testosterone, and glucocorticoids are all examples of:
A) proteins.
B) glycoproteins.
C) polypeptides.
D) amino acid derivatives.
E) steroids.
35- The monthly change in secretion of reproductive hormones that occurs in women during their reproductive years
is an example of:
A) acute hormone regulation.
B) chronic hormone regulation.
C) cyclic hormone regulation.
D) None of the above
36- Which one of the following statements is false?
A) Sperm are produced in seminiferous tubules of testes.
B) FSH effects the enlargement of ovarian follicles
C) FSH effects the ovarian secretion of estrogen.
D) LH enhances the effect of FSH.
E) FSH triggers ovulation.
37- Which one of the following statements is false?
A) Inhibin inhibits GnRH.
B) The corpus albicans is a regressed corpus luteum.
C) The menstrual cycle averages 28 days.

56

Gyna & Obstetrics Reveision by/Dr.adool

2008

D) The follicular phase of the menstrual cycle embraces menstruation and preovulation.
38- Which one of the following statements is false?
A) When the size of a secondary follicle becomes 2 cm, it is a Graafian follicle.
B) By day 6 of the menstrual cycle, usually one follicle is significantly larger than the others
C) A part of a Graafian follicle protrudes as a blister on the surface of an ovary.
D) Under the influence of an increasing concentration of LH, the preovulatory follicle produces estrogen at an
elevated rate.
E) Within 1-2 days prior to ovulation, the mature follicle produces a small amount of LH.
39- Which one of the following statements about preovulation and ovulation is false?
A) Follicles that secrete estrogens during preovulation stimulate cells of the stratum basalis to restore the stratum
functionalis via mitosis.
B) The endometrium and its arterioles and glands grow.
C) When the concentration of estrogens reaches a certain threshold, the hypothalamus is incited to release GnRH.
D) In response to GnRH, the adenohypophysis releases FSH, and not LH.
40- Which one of the following statements about day 14 of the menstrual cycle is false?
A) The surge of LH impels the preovulatory follicle to rupture and expel a 2nd polar body.
B) The surge of LH impels the preovulatory follicle to rupture and expel into the pelvic cavity a secondary oocyte that
is surrounded by a zona pellucida and corona radiata.
C) The secondary oocyte is usually swept into a uterine tube.
D) The positive feedback effect of estrogens on the hypothalamus and adenohypophysis is enhanced by
progesterone.
E) Following ovulation, the bleeding remnant of the Graafian follicle is a corpus hemorrhagicum.
41- Which one of the following statements is false?
A) Under the influence of LH, the corpus hemorrhagicum becomes a corpus luteum.
B) Postovulation typically continues from days 15-28 of the menstrual cycle.
C) The hormones that the corpus luteum secretes stimulates the secretion of GnRH, FSH, and LH.
D) Should the ovulated secondary oocyte not be fertilized, the corpus luteum regresses, and gonadotropins are
secreted. Hence, another menstrual cycle begins.
42- Which of these is an example of positive-feedback regulation in the endocrine system?
A) an increase in blood glucose causes an increase in insulin secretion; insulin moves glucose into cells
B) an increase in TSH causes an increase in thyroid hormone secretion; thyroid hormones inhibit TSH secretion
C) before ovulation, an increase in LH causes an increase in estrogen, which causes an increase in LH
D) an increase in TRH causes an increase in TSH secretion; thyroid hormone inhibits TRH secretion
E) an increase in blood calcium causes an increase in calcitonin secretion; calcitonin moves calcium into the blood.
43- Intracellular hormone receptors are located:
A) floating freely in the cytoplasm of target cells.
B) in the nucleus.
C) in the mitochondria.
D) in the endoplasmic reticulum.

57

Gyna & Obstetrics Reveision by/Dr.adool

2008

E) both a and b
44- Peptide hormones and protein hormones affect a cell by:
A) attaching to the chromosomes and altering the expression of the genes.
B) increasing the insulation provided by the myelin sheath.
C) attaching to receptors on the membrane of a cell.
D) entering the cell and being metabolized as a source of energy.
45- Which of the following is an Incorrect statement about human oogenesis.
A) One ovum is produced from each parent cell.
B) Primary oocytes can sit halted in prophase I for years.
C) After puberty begins, one primary oocyte, on average, reenters meiosis every 28 days.
D) One polar body is produced from each parent cell.
E) Meiosis II begins after a secondary oocyte is fertilized by sperm.
46- Which of the following statements about the menstrual cycle is False?
A) LH is produced by the pituitary in response to GnRH from the hypothalamus.
B) LH acts on the follicle, causing ovulation, around the 14th day of the cycle.
C) Birth control pills that contain estrogen and progesterone would inhibit the secretion of FSH by the pituitary.
D) The phase of menstruation at which pregnancy is most likely to occur is the secretory phase.
E) FSH and estrogen production are highest during days 25-28 of the menstrual cycle
47- Which structures controls the cyclic nature of the menstrual cycle?
A) oviducts and uterus
B) pituitary and testes
C) ovaries and umbilical cord
D) pituitary and ovaries
E) oviduct and thyroid
48- The primary function of progesterone in the menstrual cycle is to:
A) stimulate development of the egg.
B) stimulate development of the endometrium.
C) stimulate development of the corpus luteum.
D) stimulate development of the follicle.
E) trigger ovulation
49- What two hormones influence the development of the secondary sexual characteristics of the female?
A) testosterone and estrogen
B) androgen and estrogen
C) FSH and LH
D) progesterone and estrogen
E) testosterone and progesterone
50- Which one of the following statements about oogenesis is false?
A) Primordial germinal cells originate in ovarian epithelium.
B) Primordial germinal cells differentiate into oogonia.
C) Most oogonia experience atresia.
D) Oogonia develop into diploid oocytes.
E) <20% of the primary oocytes that exist in a newborn girl remain at puberty.

58

Gyna & Obstetrics Reveision by/Dr.adool

2008

51- Which one of the following statements about oogenesis is false?


A) Usually, only one primary oocyte per month completes meiosis I, and experiences cytoplasmic division.
B) Most primary oocytes experience atresia.
C) In primordial follicles, a primary oocyte is partially surrounded by a single layer of squamous cells.
D) In primary follicles, an oocyte is surrounded by no more than one layer of granulosal cells.
E) In an ovarian follicle, a pellucid layer of glycoproteins becomes interjected between an oocyte and granulosal cells.
52- Which one of the following statements about ovarian follicles is false?
A) Corona radiata is the name of the layer of glycoproteins that surrounds a follicular oocyte.
B) The theca includes secretory cells.
C) The theca interna is well vascularized with capillaries.
D) The theca externa contains fibroblasts, smooth myofibers, and collagenous fibers.
E) The accumulation of the secretions of the membrana granulosa causes an antrum to form.
53- Which one of the following statements about ovarian follicles and ovulation is false?
A) When a primary oocyte completes meiosis I, it divides into two secondary oocytes.
B) In a secondary oocyte of a Graafian follicle, meiosis II becomes arrest at metaphase II.
C) In any month, all growing follicles that do not ultimately become Graafian follicles, experience atresia.
D) A mature follicle ovulates a secondary oocyte into the pelvic cavity.
E) Ordinarily, fimbriae sweep the ovulated secondary oocyte into the infundibulum of a uterine tube.
54- The structure from which an egg is released during ovulation is called a(n)
A) ovarian cyst.
B) primary follicle
C) corpus luteum
D) secondary follicle
E) Graafian follicle.
55- The luteal phase of the ovarian cycle occurs, in a 28-day cycle, at about
A)
B)
C)
D)

days 1-5
days 6-13
day 14
days 15-28

56- Which of the following hormones is best associated with maturation of the egg?
A)
B)
C)
D)
E)

LH
FSH
HCG
testosterone
progesterone

57- Which of the following is Not true about estrogen?


A) Estrogen causes the endometrium to thicken.
B) Estrogen causes the endometrium to become vascular and glandular.
C) Estrogen causes a positive feedback on the hypothalamus to secret GnRH.
D) Estrogen causes a negative feedback on the anterior pituitary gland.
E) Estrogen stimulates the release of FSH.

59

Gyna & Obstetrics Reveision by/Dr.adool

2008

58- Estrogen has all of the following effects except:


A) Stimulates closure of the epiphyses
B) Causes positive fern test
C) Suppresses placental hormone secretion
D) Suppresses FSH secretion
E) Causes cornification of vaginal epithelium
59- The squamous epithelium is a target tissue for sex hormones. The vaginal epithelium of a 60 year old female is
similar to the of a:
A) 2 day old female
B) 5 year old female
C) 11 year old female in puberty
D) 21 year old female
E) 26 year old female on oral contraceptives
60- Which of these descriptions could be associated with the luteal phase of the uterine cycle?
A) low FSH, high estrogen, developing follicle, increase uterine lining
B) high LH, high estrogen, developing follicle, uterine lining breakdown
C) decrease in LH, increase in progesterone, corpus luteum present, secretory uterine lining
D) decrease in LH, decrease in progesterone, corpus luteum present, secretory uterine lining
E) increase in LH, increase in progesterone, corpus luteum present, secretory endometrium
61- After ovulation, the ruptured follicle:
A) disappears and all of its cells disintegrate.
B) passes on as waste material down the oviduct with the egg.
C) mends itself and begins the maturation of another egg.
D) becomes part of the epithelial covering of the ovary.
E) differentiates into the corpus luteum.
62- Menstruation begins because:
A) the pituitary triggers it.
B) LH activity is at its peak.
C) estrogen activity is at a peak.
D) progesterone production is highest.
E) progesterone and estrogen levels are both declining.
63- The occurrence of oligomenorrhea or amenorrhea in highly trained female athletes is...
A) rarely reversible
B) irreversible
C) typically reversible
D) fictitious
64- In the human female, estrogen and progesterone exert feedback control over the:
A)
B)
C)
D)
E)

testes
ovary
pituitary
uterus
prostate gland

60

Gyna & Obstetrics Reveision by/Dr.adool

2008

65- The uterine cycle at day 12 would primarily be under the influence of:
A)
B)
C)
D)
E)

estrogen
progesterone
epinephrine
thyroxin
testosterone

66- When an egg erupts from a follicle, the chamber that remains and secretes hormones is the:
A)
B)
C)
D)
E)

oocyte
stroma
corpus luteum
primary follicle
corpus cavernosum

67- Which of the following statements is incorrect?


A) The follicle in the ovary produces estrogen.
B) The corpus luteum produces progesterone.
C) The placenta can produce both estrogen and progesterone.
D) LH stimulates the formation of the corpus luteum.
E) A surge of FSH is believed to promote ovulation.
68- Which one of the following statements is false?
A) Testosterone is produced in interstitial cells.
B) Inhibin inhibits the secretion of FSH.
C) Inhibin inhibits the secretion of gonadotropin-releasing hormone.
D) Estrogens inhibit FSH.
E) Testosterone inhibits FSH.
69- The pituitary, which statement is true?
A) is located below the hypothalamus.
B) the pituitary is attached to the infundibulum or pituitary stalk.
C) the pituitary is developmentally two separate structures
D) none of the above are true
E) all of the above are true
70- An endometrial biopsy showing glands lined by cells containing subnuclear vacuoles is characteristic of what part
of the menstrual cycle:
A) Menstrual
B) Per-ovulatory (proliferative, postmenstrual)
C) Early post-ovulatory (early secretory, interval)
D) Late post-ovulatory (late secretory, premenstrual)
E) Pseudodecidual
71- Primary amenorrhea is associated with all except:
A) Turners syndrome.
B) Anorexia nervosa.
C) Testicular feminisation syndrome.
D) Primary hypothyroidism.
E) Asherman's syndrome following dilatation and curettage.
72- Secondary amenorrhea , all are true except:
A) may be due to hyperprolactinaemia
B) should be treated with the combined oral contraceptive

61

Gyna & Obstetrics Reveision by/Dr.adool

2008

C) may be due to Ascherman's syndrome


D) may follow cone biopsy
E) may be due to thyrotoxicosis
73- Secondary amenorrhoea , all are not correct EXCEPT:
A) is defined as cessation of menstruation for more than 12 months.
B) should be not be investigated unless persisting for > 12 months.
C) may be due to Ashermans syndrome.
D) is almost exclusively due to stress.
74- Secondary amenorrhoea may be due to all EXCEPT:
A) Anorexia Nervosa
B) Testicular Feminisation Syndrome
C) Polycystic Ovary Syndrome
D) Manchester repair
E) Ascherman's syndrome
75- Turner's Syndrome , all are false EXCEPT:
A) is associated with 46 XO
B) is always associated with primary amenorrhoea
C) is associated with congenital absence of the uterus.
D) is associated with coarctation of the aorta
E) is hereditary
76- Turner's Syndrome , all are false EXCEPT:
A) the gonads should be removed because of the risk of malignancy
B) is associated with estrogen insensitivity
C) is associated with insulin resistance
D) is associated with short stature
E) is associated with elevated FSH levels
77- Turner's syndrome , all are true EXCEPT:
A) is associated with 47 XXX
B) 98% of affected pregnancies will fail to reach term
C) is present in about 1% of conceptions
D) removal of gonads is advocated if a mosaic involving 46 XY exists
78- 46 XX is associated with one of the following:
A) Turners syndrome.
B) Testicular feminisation syndrome.
C) Klinefelters syndrome.
D) Congenital adrenal hyperplasia.
E) Sheehan's syndrome.
79- Klinefelter's Syndrome, all are true EXCEPT:
A) is 47, XYY
B) is associated with hypogonadism
C) is associated with gynaecomastia
D) is associated with infertility
E) affects 1: 600 males
80- Which of the following would probably develop as an intersex or pseudohermaphrodite?
A) a genetic female exposed to more testosterone than normal during early development
B) a genetic male exposed to more estradiol than normal during early development
C) a genetic female deprived of her normal amount of estradiol during early development
D) a genetic male exposed to a larger than normal amount of testosterone during early development
81- In what way are people with androgen insensitivity most like a normal male?
A) Appearance
B) genetics

62

Gyna & Obstetrics Reveision by/Dr.adool

2008

C) gender identity
D) rearing
82- Which of the following causes a male mammal to develop an anatomy that looks like a female?
A) deficit of testosterone during puberty
B) deficit of testosterone during an early stage of development
C) exposure to a high level of estradiol during puberty
D) exposure to a high level of estradiol during an early stage of development
83- A female human fetus exposed to excess testosterone during the sensitive period for genital development:
A) is unaffected, since she has no receptors for testosterone.
B) will often develop without any sexual organs.
C) will often develop with genitals that appear intermediate between male and female genitals.
D) will often develop a complete, functioning, set of male reproductive organs.
84- Which of the following would cause a genetic female to develop a partly masculinized anatomy?
A) excessive levels of alpha-fetoprotein in her blood
B) exposure of her mother to stressful experiences late in pregnancy
C) exposure to less than the usual amount of estrogen during an early sensitive period
D) exposure to more than the usual amount of testosterone during an early sensitive period
85- Cryptomenorrhoea is associated with all EXCEPT:
A) Turners syndrome.
B) abdominal pain.
C) cone biopsy.
D) Manchester repair.
86- Which of the following is true of the menopause?
A) FSH & LH levels are elevated
B) FSH rises, while LH and oestradiol fall
C) Prolactin levels are elevated
D) HRT should always be offered
87- In testicular feminisation syndrome , all are true EXCEPT:
A) Karyotype XY
B) Secondary sexual characters include undeveloped pubic hair
C) Karyotype XXY and well developed uterus is a characteristic feature
D) Well developed breast is a characteristic feature
88- In idiopathic hirsutism, all are not true EXCEPT:
A) Clitoromegaly
B) Male type of baldness
C) Presence of uterus
D) Steroids are effective
89- In Klinefelter's syndrome, all karyotypes may be found EXCEPT:
A) XXYY
B) XY or XX
C) XXXY
D) XXY
90- Osteoporosis, all are not true EXCEPT:
A) Characteristically serum Ca is increased
B) Fracture Shaft of humerus is common
C) More common in women than in men

63

Gyna & Obstetrics Reveision by/Dr.adool

2008

D) Treated normally with rest


E) Iron supplements can be of help
91- All of the following statements are true of physiological menopause except:
A) Some of the symptoms experienced include hot flashes headaches, and insomnia
B) Onset of menopause is usually between the early forties and mid fifties
C) Depression is a significant problem in all menopausal females
D) Vasomotor symptoms are caused by estrogen deprivation
92- Amenorrhea is seen in all EXCEPT:
A) XXX- karyotype
B) Testicular feminization syndrome
C) Turner's
D) Anorexia nervosa
93- Recognized associations of hyperprolactinaemia include all of the following EXCEPT:
A) Stress
B) Phenothiazines
C) Menorrhagia
D) Chest trauma
E) Cancer breast
94- Puberty is associated with all EXCEPT:
A) Growth spurt
B) Onset of mensturation
C) Alteration in the distribution of body fat
D) Characteristic reduction in FSH
E) Anovular cycles
95- In testicular Feminisation Syndrome , all are true EXCEPT:
A) the gonads produce testosterone
B) presentation is usually with secondary amenorrhoea
C) axillary and pubic hair are absent
D) breast development is normal
E) dysgerminoma may develop in the gonads, which should be removed after development of secondary
sexual characteristics
96- Short stature is associated with which of the following:
A) Turner's syndrome
B) Klinefelter's syndrome
C) Polycystic ovary syndrome
D) 47XXX
E) Testicular Feminisation syndrome
97- Puberty, all are true EXCEPT:
A) Breast development usually starts about the age of nine
B) The appearance of pubic and axillary hair at 10
C) Menstruation at 12-13
D) every fetus will develop female external genitalia unless exposed to the effect of testosterone
E) The age at puberty is defined as the age at menarche
98- All statements about Pre-menstrual Syndrome are true except:
A) is due to lack of progesterone.
B) may respond to removal of ovaries.
C) may respond to cyclical selective serotonin reuptake inhibitors.
D) danazol may be effective.
E) may respond to premenstrual estrogen.
99- Regarding Post-menopausal bleeding , the following statements are false except:
A) is frequently due to genital tract malignancy

64

Gyna & Obstetrics Reveision by/Dr.adool

2008

B) may be due to bladder pathology


C) is often to due to cervical erosions
D) initial investigation should include pelvic examination, cervical cytology and a transvaginal scan
100- Treatments for patients with Polycystic ovary syndrome include the following except:
A) reassurance only
B) combined oral contraceptive
C) cyclophosphamide
D) laser drilling of the ovaries
E) weight reduction
101- Hirsutism, all are true EXCEPT:
A) is synonymous with virilisation
B) is associated with polycystic ovary syndrome
C) is a side effect of danazol
D) may be due to an ovarian tumour
102- A sixteen year old girl has primary dysmenorrhoea. Suitable treatments is:
A) D&C
B) pre-sacral neurectomy
C) non-steroidal anti-inflammatory drugs
D) laser ablation of the endometrium
E) uterine artery ligation
103- Which of the following is not found in Polycyctic Ovarian Syndrome?
A) Increased estrogen levels
B) Increased free testosterone levels
C) Increased FSH levels
D) Increased LH levels
E) Decreased sex hormone binding globulin levels
104- Amenorrhoea can be caused by the following except :
A) hypogonadotrophic hypogonadism
B) anaemia
C) hypoprolactinaemia
D) Turners syndrome
E) Ashermans syndrome
105- Luteinising hormone releasing hormone (LHRH) analogues are effective in the treatment of all of the following
EXCEPT:
A) menorrhagia
B) endometriosis
C) uterine fibroids
D) osteoporosis
106- Pubertal menorrhagia is managed by :
A) Progesterone
B) GnRH
C) Danazol
D) Surgery
107- True about testicular feminization include the following EXCEPT:
A) Testes present
B) Female phenotype
C) XY
D) Primary amenorrhea

65

Gyna & Obstetrics Reveision by/Dr.adool

2008

E)

Uterus present

108- Klinefelters syndrome is associated with all EXCEPT:


A) XXY
B) Male phenotype
C) Infertility
D) Azoospermia
E) Bar body absent
109- A 22-year-old woman has irregular, unpredictable menstrual periods every 30-90 days. On examination she has
acne on her face and upper back and a few dark, coarse hairs on her chin and lower abdomen. Of the following
laboratory tests, the most sensitive marker of androgen excess is:
A) total testosterone
B)
free testosterone
C)
ratio of luteinizing hormone to follicle-stimulating hormone
D)
fasting insulin
E)
dehydroepiandrosteronesulfate
110- A 14-year-old girl had breast development at age 11 years and pubic hair at age 12 years but has never
menstruated. She has had increasing cyclic pelvic pain for 6 months. On physical examination, she has Tanner
stage IV breasts and Tanner stage IV pubic hair. Examination of her external genitalia reveals normal labia majora
and minora but no vagina. No obvious hymen is seen. Upon Valsalva maneuver, no bulging is noted at the
introitus. Ultrasonography reveals a distended endometrial cavity and a collection of echo-dense material caudal
to the uterus. The best management for this patient's pathology is:
A) cruciate incision of the imperforate hymen
B) complete excision of the transverse vaginal septum
C) fine-needle drainage of the hematocolpos
D) vaginal dilators
E) Mclndoe vaginoplasty
111- Ovarian hyperstimulation syndrome is characterized by:
A) Excessive uterine bleeding with severe intravascular volume depletion.
B) Bleeding into the ovaries secondary to excessive ovulatory stimulation.
C) Capillary vasodilatation leading to intravascular volume depletion.
D) Severe abdominal pain due to ovarian rupture
E) Death or renal failure in more than 50% of patients
112- A patient with amenorrhea had bleeding after giving a trial of progesterone. This implies all of the following
except:

A)
B)
C)
D)

Sufficient estrogen
Normal ovarian function
Intact endometrium
Intact pituitary axis

113- In polycystic ovarian diseases, all of the following are seen except :
A) Increased LH/ FSH
B) Streak ovaries
C) Insulin resistance
D) Hirsutism

66

Gyna & Obstetrics Reveision by/Dr.adool

2008

114- Preferable treatment for 32 ys. old multipara with dysfunctional uterine bleeding (DUB) is :
A) Progestrogens
B) Danazol
C) Prostaglandins
D) Endometrial ablation
E) Hysteretomy
115- All are seen in testicular feminization syndrome except :
A) 46 XY
B) Primary amenorrhoea
C) Short stature
D) Vaginal pouch present
116- Among the following, which is a feature of testicular feminization syndrome :
A) XX pattern
B) Commonly reared as male
C) Well formed female internal genitalia
D) High testosterone levels
117- Which of the following is not typically associated with hirsutism:
A) panhypopituitarism
B) ovarian thecal cell carcinoma
C) elevated urinary 17-ketosteroids excretion in patients with adrenal carcinoma
D) elevated plasma levels of DHEA-S in patients with polycystic ovarian disease (PCOD)
118- Which of the following statements about congenital adrenal hyperplasia is not correct:
A) it never presents in adulthood
B) 21-hydroxylase deficiency is the most common defect
C) 21-hydroxylase deficiency can present as acute adrenal insufficiency
D) 17 -hydroxyprogesterone can be used to diagnose and monitor the treatment of 21- hydroxylase
deficiency
119- The following statements about the pituitary is not correct:
A) the posterior pituitary secrets oxytocin
B) LH, FSH, and TSH share the same common subunit
C) LH and FSH are secreted by the same cells in the pituitary
D) the most common tumour type in the pituitary secrets prolactin
120- Prolactinomas can be associated with the following except:
A) Oral Contraceptive pills
B) Excessive milk secretion
C) Increased size of sella tursica
D) Acromegaly
E) Infertility
121- Amenorrhoea can be seen in the following except:
A) Testicular feminization
B) Turner's syndrome
C) Pituitary tumours
D) Anorexia nervosa
E) Fibroid uterus
122- In Polycystic Ovary all are false except:
A) Decreased LH
B) Adrenal Hyperplasia
C) Failure of maturation of secondary sex characters
D) Oligomenorrhea

67

Gyna & Obstetrics Reveision by/Dr.adool

2008

123- A woman of 23 years presents with a 12 month history of amenorrhoea. Circulating FSH concentrations
measured at intervals of six weeks were 40 and 45 iu/L. The following statements are correct except:
A) The most likely diagnosis is premature ovarian failure.
B) The woman may conceive spontaneously.
C) The woman should be encouraged to take estrogen replacement to prevent osteoporosis.
D) There is an increased incidence of this condition following some forms of chemotherapy.
E) The woman may conceive following oocyte donation.
124- Concerning hormonal assessment in gynecological endocrinology ,the following statements are correct except:
A) Ovulation can be determined by measuring the circulating concentration of progesterone.
B) In a woman with a regular 35 day cycle, ovulation should be detected by measuring the
progesterone concentration on day 21.
C) Gonadotrophins (follicle stimulating hormone and luteinising hormone) should be measured in the
early follicular phase.
D) A prolactin concentration of > 200 ng/mL is suggestive of a pituitary macroadenoma
125- The following treatments are effective in reducing menstrual blood loss EXCEPT:
A) Non-steroidal anti-inflammatory agents.
B) Drugs that promote fibrinolysis.
C) Dilatation and curettage.
D) The levonorgestrel releasing intrauterine system.
126- Treatment with hormone replacement therapy, all are incorrect except:
A) Should always include a progestagen.
B) Reduces the incidence of cardiovascular disease in post-menopausal women.
C) Should not result in vaginal bleeding.
D) Is not given in presence of a uterus.
127- The typical sequence of pubertal events in adolescent girls is:
A) Thelarche, menarche accelerated growth, adernarche
B) Adrenarche, thelarche, menarche, accelerated growth
C) Thelarche, adernarche, accelerated growth, menarche
D) Adrenarche, accelerated growth, thelarche, menarche
E) Accelerated growth, thelarche, adrenarche, menarche
128- A 16 year-old girl is evaluated for primary amenorrhea. Physical examination reveals Tanner stage III breast and
pubic hair development with an absent vagina. The most likely diagnosis in this patient is:
A) Isolated gonadotropin deficiency
B) Androgen insensitivity syndrome
C) 5-reductase deficiency
D) XX pure gonadal dysgenesis
E) Mllerian agenesis
129- A 19-year-old nulligravid woman comes to you for evaluation. Her physical examination is significant for short
stature (height of 144 cm [57 in.]); a webbed neck; a high, arched palate, a broad chest with Tanner stage III
breast development, and Tanner state III pubic hair as well as secondary amenorrhea. You suspect a diagnosis of
Turners syndrome. This patients karyotype is most likely to be:
A) 45,X
B) 45,X/46,XX
C) 46,XY
D) 47,XXX
E) 46,XX

68

Gyna & Obstetrics Reveision by/Dr.adool

2008

130- A 53-year-old menopausal woman was prescribed combined estrogen-progestin therapy 1 year ago. She now
presents with a 1-month history of vaginal bleeding. On objective questioning, she says that she has taken only
the estrogen for the past 9 months because of fear of weight gain from taking the progestin. Her medical history
is significant for hypertension controlled with atenolol). Her hemoglobin is 12.3 g/dL. An endometrial biopsy
reveals adenomatous hyperplasia with moderate cytologic atypia .The most appropriate management for this
patient is:
A) observation off estrogen for 3 months
B) repeat endometrial biopsy in 3 months
C) oral medroxyprogesterone acetate therapy for 3 months
D) endometrial ablation
E) hysterectomy
131- A 57-year-old postmenopausal woman is referred for consultation after her primary care physician ordered
transvaginal ultrasonography to evaluate postmenopausal bleeding. Her Pap test result was normal. The
ultrasonogram reveals a maximal endometrial thickness of 3.7 mm. The best next step is:
A) endometrial biopsy
B) hysteroscopy
C) hysterosonogram
D) progestational therapy
E) observation
132- A 57-year-old woman with post-menopausal bleeding is referred by her family physician for assessment. Review
of her endometrial biopsy reveals simple hyperplasia without atypia. She is reluctant to have any medical or
surgical treatment unless it is absolutely necessary. You counsel her that the chances that this will progress to
endometrial cancer if left untreated is approximately:
A) 1-5%
B) 5-10%
C) 10-20%
D) 25-50%
E) >50%
133- Chromosomal configuration in Klinefelter's syndrome is:
A) 47, XY, + 21
B) 47, XXX
C) 47, XXY
D) 46, XY.
134- Conditions in which obesity is an associated feature include all, except:
A) Laurence Moon-Biedl syndoome .
B) Steven Leventhal syndrome
C) Grave's disease
D) Cushing's syndrome

69

Gyna & Obstetrics Reveision by/Dr.adool

2008

(6) * Infertility & Hirsutism


(1)In Primary infertility the most probable cause is:
A) Ovualtion factor.
B) Tubal factor.
C) Endometrial factors.
D) Cervical factor.
(2)In primary infertility with tubal block, the cause may be:
A) Puerperal sepsis.
B) Postabortion fever.
C) Tubal tuberculosis.
D) Toxoplasma gondii.
(3)Tubal block, in primary infertility, may be due to any of the following, EXCEPT:
A) Gonorrhoea.
B) Chlamydia infection.
C) T.B. of female genital tract.
D) Bilharziasis of female genital tract.
E) Trichomonas vaginalis infection.
(4)Side and Location of tubal obstruction is detected by:
A) Ultrasonography.
B) Hysterosalpingography.
C) Hyseroscopy.
D) Colposcopy.
(5)Sperm penetration, in the cervical mucus, is detected by:
A) Fern Test.
B) Spinbarkeit.
C) Post coital test.
D) Vaginal smear.
(6)Treatment of hostile cervical antibodies is:
A) Six-month abstinence.
B) Corticoids.
C) Intrauterine insemination (I.U.I).
D) (A + B).
E) All of the above.
(7)Polycystic ovary syndrome is treated by:
A) Clomiphene citrate.
B) Clomiphene citrate + LH (Human menopausaul gonadotroptive)
C) Ovarian drilling.
D) (A, B).
E) All of the above.
(8)Best results of tuboplasty are found in cases of:
A) Post partum sepsis causing tubal obstruction.
B) Endometriosis in the region.
C) Post ligation.

70

Gyna & Obstetrics Reveision by/Dr.adool

2008

D) T.B. tubes.
(9)Neoplastic cysts of the ovary include:
A) Inclusion (Walthards) cysts.
B) Retention cysts.
C) Inflammatory cysts.
D) All of the above.
E) None of the above.
(10)Hirsutism in defined as:
A) Excessive growth of hair in an abnormal site for the female.
B) Increased muscle bulk.
C) Clitorial hypertrophy, breast atrophy and deepening of voice.
D) (A + B)
E) (A +B + C)
(11)Hair in the following areas is considered as hirsutism EXCEPT in:
A) Cheek.
B) Moustache area.
C) Arm.
D) Forearm.
E) Thigh.
(12)In a case of hirsutism with elevated serum DHEAS, one should suspect:
A) Ovarian neoplasm.
B) Polycystic ovary syndrome.
C) Suprarenal lesion.
D) Pituitary lesion.
(13)In luteal phase defect, the following procedure can be useful for diagnosis, Mark the WRONG statement:
A) Endometrial biopsy.
B) Basal body temperature.
C) Serum progesterone.
D) Ultrasonography.
E) None of the above.
(14)In Luteal phase defect the following drugs can be use for treatment, EXCEPT:
A) Progesterone.
B) Progesagens.
C) Clomphene citrati.
D) Human chorionic gonadotrophins.
E) Bromoergocriptine.
15. As regard of cervical factor of infertility, all are correct except :
a) Artificial insemination is a good treatment choice.
b) Clomiphene citrate therapy could be a cause.
c) Assessment of cervical mucus is done in day 18 of the cycle.
d) It is common in cases with secondary infertility.
16.As regard hyperprolactinemia, which is not true? :
a) Medication is a common cause
b) Bromocryptin is effective treatment

71

Gyna & Obstetrics Reveision by/Dr.adool

2008

c) Commonly associated with poor endometrial development


d) Galactorrhea always presents
17. Normal values of semen analysis are:
A. A weak acidic reaction.
B. A volume o 2 to 12 cc.
C. Abnormal forms not exceeding 50%.
D. A sperm count of at least 20 million/ml.
E. Progressive motility in at least 50%.
:
A. Volume of less than 2 ml.
B. Count of more than 40 million/ml.
C. Motility of 40 % after one hour.
D. Abnormal forms of less than 20 %.
E. Pus cells not exceeding 12-15 / HPF.
18. Symptoms suggestive of ovulation include:
A. Irregular cycles.
B. Spasmodic dysmenorrhea.
C. Premenstrual tension syndrome (PMS).
D. Congestive dysmenorrhea.
E. Galactorrhea.
19. Acceptable methods for confirmation of ovulation include:
A. Folliculometry.
A. Folliculometry.
B. Positive thread test of cervical mucus.
C. Serum estrogen level on day 14.
D. Serum progesterone on day 21.
E. Premenstrual endometrial biopsy.
20. Luteal phase defect:
A. Is associated with delayed menses.
B. May be due to inadequate progesterone production.
C. Is a common cause of infertility.
D. Is a common cause of recurrent abortion.
E. The endometrium shows weak secretory changes.
21. With luteal phase defect:
A. Cycles may be short.
B. The basal body temperature chart is not biphasic.
C. Ovulatory pain is absent.
D. A defective corpus luteum can be diagnosed by ultrasonography.
E. Luteal support is best provided by progestins.
22. Luteal phase defect may be associated with:
A. Short cycles.
B. Endometriosis.
C. Low estrogen levels.
D. Endometrial biopsy showing weak secretory changes.
E. Low prolactin levels.

72

Gyna & Obstetrics Reveision by/Dr.adool

2008

23. Clomiphene citrate:


A. Causes down regulation of the pituitary.
B. Causes increased release of FSH.
C. Has no effect on the quality of cervical mucus.
D. May result in multiple ovulation.
E. Inhibits prolactin secretion.
24. Potential complications of drugs used for induction of ovulation include:
A. Multiple pregnancy.
B. Ovarian hyperstimulation syndrome.
C. Headache.
D. Ascites.
E. Hyperprolactinemia.
25. Tubal patency may be demonstrated by:
A. Hysterosalpingography.
B. Hysteroscopy.
C. Laparoscopy.
D. Vaginal ultrasonography.
E. CT scan.
26. Regarding hysterosalpingography:
A. The procedure should be done premenstrual.
B. Chronic salpingitis is a contraindication to this procedure.
C. Abnormities of the uterine cavity can be diagnosed by this procedure.
D. Tuberculous salpingitis can be diagnosed by this procedure.
E. The procedure may have a therapeutic effect.
27. Cervical and uterine factors of infertility:
A. The post coital test should be done post menstrual.
B. Chronic cervicitis could be a cause of infertility.
C. Tuberculous endometritis is diagnosed by curettage.
D. Hysteroscopy is the most valuable diagnostic tool for cases of uterine factor
of infertility.
E. Hysteroscopy is the best method of treatment of intra-uterine adhesions.
28. Dyspareunia:
A. May be classified as primary and secondary.
B. May be classified as superficial and deep.
C. Is a cause of effluvium seminis.
D. May be due to pregnancy.
E. May be associated with atrophic vaginitis.
29. Causes of dyspareunia include:
A. Endometriosis.
B. Impotence.
C. Chronic salpingitis.
D. Chronic cervicitis. E. Use of IUD.

73

Gyna & Obstetrics Reveision by/Dr.adool

2008

30.Hirsutism:
A. Is excessive growth of androgen dependent hair.
B. Always reflects androgen overproduction.
C. The most common cause is androgen producing ovarian tumors.
D. Is commonly associated with oligohypomenorrhea or amenorrhea.
E. Hair removal has no place in treatment.
31. Hormonal measurement(s) required to differentiate ovarian from adrenal androgen:
A. Androstenedione
B. Free testosterone to total testosterone ratio.
C. Dihydrotestosterone (DHT).
D. Dehydroepiandrosterone (DHEA).
E. Dehydroepiandrosterone (DHEAS).
32. Treatment of hirsutism may include:
A. Combined oral contraceptive pills.
B. Danazol.
C. Progestins.
D. Cyproterone acetate.
E. Spironolactone.
33.A case of 2 years 1ry infertility of patient 22 years old with regular cycle, you will start work-up by:
a) Hysterosalpingography
b) Semen analysis
Laparoscopy
c)
d) Hysteroscopy
34.The followings are used in diagnosis of ovulation except:
a) Midluteal progesterone assay
b) Endometrial biopsy
Vaginal ultrasonography c)

d) Gysteroscopy

35. All of the following are correct for tubal infertility, except:
a) PID is a common cause.
b) Laparoscopy is a good diagnostic test.
c) Tuboplasty is the best treatment option.
d)

It is responsible for about 20% of infertility causes.

Enumerate :
1.Tests for detection of ovulation :
2.Criteria for normal semen analysis according to WHO :
3.Causes of luteal phase defect :
4.Investigations for a case with hyperprolactinemia with infertility :
5.Value of laparoscopy in infertile case :

74

Gyna & Obstetrics Reveision by/Dr.adool

2008

Give an account on :

The management of PCO in case of infertility.


Put () for the true and (X) for the False of the following :

Patients with PCOS at risk for development of luteal phase defect even if ovulating.

Normal laparoscopic tubal finding exclude it completely as a cause of infertility.

Infertility following post-abortive endometritis is commonly due to intrauterine syncheiae.

Dopamine agonist is effective in treatment anovulatory cases associated with hyperprolactinemia.

Say true or false.


a. Serum progesterone above 3ng/ml is suggestive of ovulation
b. Serum progesterone above 12ng/ml isa suggestive of ovlation with adequate corpus luteum.
c. Male factor is responsible for above 60% of causes of infertility.
d. ICSI can treat cases of infertility due to hypoplasia of the uterus.
e. ICSI gives best results in treating male factor of infertility.
f. Laparoscopy can diagnose some of unexplained factor of infertility.
g. Laparoscopy can be both diagnostic and therapeutic for cases of infertility.
h. Hysterosalpingography can diagnose cases of tubal, uterine and unexplained factors of infertility.
i. Luteal phase defect can result into menorrhagia, polymeorrhea, infertility or habitual abortion.
j. Pre-menstural endometrial biopsy can diagnose cases of tuberculous endometritis by finding beading.
k. P.E.B is best taken from the lateral edge of the uterus near the fundus.
l. Evaluation of serum progesterone is best done at day 19 of the cycle.
m. Post coital test should be done in the pre-mestural period for maximum progesterone effect.
n. Vaginal acidity can result into por post-coital test.
o. Tubal factor of ifertility ca be treated by I.U.I.
p. L.H.RH analouges can be used for induction of ovulation in cases of P.C.O by supertion of endogenous gonadotropines secretion.
q. Ovarian hyperstimulation usually ocuurs in P.CO patients with induction by gonadotrophiens.
r. Ovarian hyperstimulation may end fatally.
s. H.C.G having LH action is given when the folicle is mature (30 mm ) in diameter.
t. Tuboplasty surgery has a high success rate I cases of tuberculous salpingitis.
u. Unilateral tubal block is best treated with tuboplasty surgery.
v. Immuniologivcal factors may play a role I infertility or habitual aboprtion.
w. Anti-sperm antibodies can be treated by use of condom and administration of steroids to the male partener.
x. The suces rate of I.V.F & ET is ranging between 16-30% I best centres.
y. Cases of Azoospermia are indicated for testicular biopsy.
z. Salpingitis is one of the commoest causes of female infertility.

75

Gyna & Obstetrics Reveision by/Dr.adool

2008

(7) * Displacements, traumatic lesions, urogynecology


Genitourinary fistula
1. A vesico-vaginal fistula:
A. May be associated with amenorrhea.
B. The most common cause is obstructed labor.
C. Desire of micturition may be preserved in some cases.
D. May be associated with primary vaginitis.
E. Is not always associated with urinary incontinence.
2. A vesico-vaginal fistula may be a complication of:
A. Hysterectomy.
B. Classical repair.
C. Obstructed labor.
D. Cervical lacerations.
E. Cesarean section.
3. Causes of vesico-vaginal fistula include:
A. Obstetric injury.
B. Abdominal hysterectomy.
C. Cesarean section.
D. Radiotherapy for ovarian tumors.
E. Radical hysterectomy for cancer cervix.
4. Diagnosis of vesico-vaginal fistula:
A. All cases complain of urinary incontinence.
B. Some cases feel a desire to evacuate the bladder.
C. In Sims position, the patient lies on her left side with the right thigh semi flexed.
D. Urea and creatinine measurement in urine is essential before surgical repair.
E. Examination under anesthesia is required in all cases.
5. Diagnosis of vesico-vaginal fistula:
A. A picture of cystitis in the form of frequency and burning micturition is common.
B. A picture of uremia is sometimes present.
C. The kidneys may be palpable and tender.
D. Cystoscopy may be required in some cases.
E. IVP may serve as a kidney function test.
6. Diagnosis of vesico-vaginal fistula:
A. Amenorrhea is a common symptom.
B. Methylene blue test is of help in determining the cause of incontinence.
C. Sims speculum is used to visualize the posterior vaginal wall.
D. Ultrasonography is essential for the diagnosis.
E. A sample collected midstream is essential for complete urine analysis.
7. The value of cystoscopy in cases of vesico-vaginal fistula:
A. Confirms the diagnosis of vesico-vaginal fistula:
B. Helps to visualize a uretero-vaginal fistula.
C. Serves as a kidney function test.

76

Gyna & Obstetrics Reveision by/Dr.adool

2008

D. Helps to visualize the pelvis of the kidney in cases of hydronephrosis.


E. Helps to determine the relation of the fistula to the ureteric openings.
8. Diagnosis of urinary incontinence requires:
A. Urine analysis.
B. Hysterosalpingography.
C. Cystourethrometry.
D. Full history.
E. Cystoscopy.
9. Measures to avoid development of vesico-vaginal fistula include:
A. Evacuation of the bladder before pelvic operations.
B. Evacuation of the bladder after labor.
C. Careful dissection of the bladder during classical cesarean section.
D. Early detection and proper management of cases of cancer cervix.
E. Use of Trendelenberg position during laparoscopy.
10. Management options for cases of vesico-vaginal fistula include:
A. Recognition and immediate repair of bladder injury during the operation.
B. Abdominal repair is required for a fistula near the ureteric opening.
C. Repair using a flap is required for a large fistula.
D. With flap splitting operation, a vascular flap is used to close the defect.
E. Uretero-colic implantation is one of management options in primary cases.
11. Management options for cases of vesico-vaginal fistula include:
A. Leaving a catheter in the bladder for 2 weeks immediately after the
operative injury.
B. Repair of the defect using laser beam.
C. Abdominal repair is required for a large fistula.
D. Latzko operation is required for a fistula near the ureteric opening.
E. Complete closure of the vagina is one of management options.
12. Preoperative preparation for cases of vesico-vaginal fistula includes:
A. Waiting for 6-12 months after the injury.
B. Oral contraceptive pills should be used for 6 months.
C. Correction of anemia.
D. Treatment of urinary tract infection.
E. Treatment of vulvitis.
13. Postoperative care for cases of vesico-vaginal fistula includes:
A. Vaginal pack for 1 week.
B. Catheter for 2 weeks.
C. Urine amount and color should be examined every 12 hours.
D. No intercourse for 3 weeks.
E. No pregnancy for 3 months.
14. Uretero-vaginal fistula:
A. The majority of cases are congenital.
B. The parametrium is a common site of ureteric injury.
C. The patient usually complains of partial incontinence.
D. Cystoscopy shows intact bladder.

77

Gyna & Obstetrics Reveision by/Dr.adool

2008

E. A ureteric catheter can be passed on the affected side.


15. Uretero-vaginal fistula:
A. Exposure and complete dissection of the ureter helps avoiding its damage.
B. Patients feel a desire to evacuate the bladder.
C. Affected cases do not develop vulvitis.
D. Methylene blue test shows wet unstained gauze.
E. Cystoscopy shows urine efflux on the affected side.
16. Urethro-vaginal fistula:
A. Is usually traumatic.
B. Is associated with partial incontinence.
C. Affected cases do not develop vulvitis.
D. A ureteric catheter cannot be passed on the affected side.
E. Repair may be vaginal or abdominal.
17. Vesico-uterine fistula:
A. Is not uncommon.
B. Usually follows cesarean section.
C. Affected cases complain of persistent hematuria.
D. Methylene blue test shows wet unstained gauze.
E. Repair may be vaginal or abdominal.
17. Recto-vaginal fistula:
A. Is usually traumatic.
B. All patients are incontinent to stools.
C. Affected cases do not develop vulvitis.
D. Cases following hysterectomy require Latzko operation.
E. Some cases are treated by converting the lesion into complete perineal
tear followed by repair.
18- What is the most common type of fistula formation after prolonged obstructed labor?
A) cervicovaginal
B) rectovaginal
C) uterovaginal
D) vesicovaginal
19- Vesico-vaginal fistula, all are false EXCEPT:
A) usually presents with stress incontinence of urine.
B) is most often the result of obstructed labour.
C) is more often due to ventouse delivery than forceps, due to the vaginal wall
being caught within the cup.
D) cannot be repaired vaginally.
20- Vesico-vaginal fistula , all are true EXCEPT:
A) Injury to the dome of the bladder
B) The commonest area of trauma to the bladder with hysterectomy is the supratrigona! portion
C) The three-swab test with methylene blue dye should be performed
D) A Latzko operation is suitable for patients with large vault fistula

78

Gyna & Obstetrics Reveision by/Dr.adool

2008

21- A 43-year-old woman describes, dribbling of urine when standing up after completion of voiding. She says she
does not have urgency, frequency, or nocturia. She has had three episodes of cystitis diagnosed within the past
year. Physical examination reveals a 1 x 2- cm non tender swelling adjacent to the urethra approximately 2 cm
cephalad to the urethral meatus. The remainder of her examination is normal. Cystourethroscopy is performed
and does not identify an abnormality. The next step in the diagnosis of this patient is:
A) Voiding cystourethrography
B) Retrograde positive pressure urethrography
C) Intravenous pyelogram
D) Urethral pressure profilometry
E) Pessary
22- In the previous case , what is the possible diagnosis?
A) Cystocele
B) parauretheral abscess
C) Uretheral diverticulum
D) None of the above.
23- Urethral diverticulae, all are true EXCEPT:
A) Often multiple
B) Cause dyspareunia
C) Surgical excision is always required
D) May be congenital
E) May result from an abscess of the para-urethral tubules which bursts into the urethra
24- A 55 year old woman has recurrent urinary retention after a hysterectomy done for huge fibroid. The most likely
cause is:
A) Atrophic and stenotic urethra.
B) Lumbar disc prolapse.
C) Injury to the bladder neck.
D) Injury to the hypogastric plex.
25- Detrusor instability, all are false EXCEPT:
A) May occur de novo after bladder neck surgery
B) A pressure rise during filling of between 5-15 CmH2O
C) Always symptomatic
D) Implies a neurological disorder
26- The urethral syndrome , all are false EXCEPT:
A) Dysuria
B) Incontinence
C) Significant bacteriuria in over 50% of women
D) Urodynamic evaluation is useful
27- At the time of a difficult abdominal hysterectomy for fibroids and endometriosis, It is found that a clamp had been
Inadvertently placed on a ureter and that It had been there for at least 30 minutes before it was discovered:
A) The most frequent site of ureteral injury is at the infundibulopelvic ligament
B) Stricture or fistula formation is likely
C) If a high ureteric injury is found, direct end to end anastomosis is performed
D) Extraperitoneal drainage following repair

79

Gyna & Obstetrics Reveision by/Dr.adool

2008

28- The following may be used to assess the functioning of the proximal urethra and bladder neck EXCEPT:
A) Ultrasound
B) Uroflowmetry
C) Urethral electric conductance profile (UECP)
D) Bonney's test.

29- An extremely anxious 50-year-old woman presents to your office and complains of soaking her underpants when
arising in the morning. She also reveals that once initiated, she is unable to control micturition. The most likely
diagnosis is:
A) detrusor dyssynergia
B) urgency incontinence
C) urethral diverticulum
D) overflow incontinence
30- A woman has been anuric for the first 48 hours after undergoing abdominal hysterectomy. Bilateral ligation of the
ureters is suspected. Which of the following therapeutic measures should first be ordered?
A) Observation only
B) Ureteral catheterization
C) Rapid dialysis
D) Transabdominal deligation
E) Bilateral nephrostomy
31- If ureteral injury is recognized at the time of surgery, which of the following procedures could be recommended?
A) A longitudinal slit should be made in the ureter below the injury and a polyethylene tube threaded into the
bladder
B) If the ureter is not severed, the site of injury should be drained intraperitoneally
C) If the ureter is severed, ureteroureteral anastomosis should be attempted. regardless of the location of the injury
D) If possible, the severed ureter should be implanted into the bladder
32- During gynecological surgery, operative injuries to the ureter occur
A) more frequently in association with vaginal rather than abdominal hysterectomy
B) only rarely if periureteral tissue is dissected carefully
C) most commonly when the ureter lies between the anterior vaginal wall and the base of the bladder
D) often as a result of hasty reclamping of vessel clamps or ligatures
33- For each clinical description that follows, select the appropriate disorder:
1) A 45-year-old woman states that she has had increasing difficulty holding urine after feeling the need to void.
She occasionally loses urine when sitting down (E)
2) A 43-year-old woman, who recently underwent an anterior vaginal repair, complains of constant perineal
wetness and loss of urine while standing. She does not feel the urge to void ( D )
3) A 28-year-old diabetic woman reports that she loses large volumes of urine without warning. She says that she
can overcome the problem by frequent, voluntary emptying of the bladder ( A )
4) A 26-year-old woman, who has four children, is worried because her daily exercises seems to be causing
urinary dribbling ( C )
A) Neurogenic bladder
B) Overflow incontinence
C) Stress incontinence
D) Vesicovaginal fistula

80

Gyna & Obstetrics Reveision by/Dr.adool

2008

E) Urgency incontinence
34. A 60-year-old woman, gravida 5, para 5, presents with complaints of leaking urine
when she coughs and sneezes. She has not had constant wetness, urgency, or
frequency but has had some nocturia and several episodes of nocturnal enuresis. A
clean catch urine culture was negative for growth earlier this week at her internists
office. Her neurologic examination is within normal limits. A mild cystocele and
rectocele are noted on vaginal examination, and the vaginal epithelium appears
slightly atrophic. A cotton swab straining angle is 45 degrees. During the assessment, she leaks urine when coughing
with a full bladder Her post void residual volume is 30 mL. Initially, no fluctuation of the fluid level is noted on simple
office cystometry. However, during the bladder-filing phase, there is overflow of the water column soon
after the patient senses the urge to void. The clinical picture at this time is most consistent with
(A) Genuine stress urinary incontinence (GSUI)
(B) Overflow incontinence
(C) Mixed incontinence
(D) Detrusor instability
35.A 27year-old woman presents with a foul vaginal odor 7 days after a vaginal delivery complicated by a third-degree
perineal laceration. Examination reveals a 2-mm rectovaginal fistula 1.5 cm proximal to the vaginal introitus. The
correct initial management of this patient is
(A) Observation
(B) Closure of the rectovaginal fistula with a sliding rectal flap
(C) Fistulectomy with layered closure
(D) Closure with a bulbocavernosus fat pad (Martius) graft
(E) Diverting colostomy followed by a layered closure
36.A 43 year-old woman, gravida 2, para 2, undergoes a difficult total abdominal
hysterectomy for menorrhagia and an 18-week-sized leiomyomatous uterus.
Previous surgical history includes a myomectomy followed by 2 cesarean
deliveries. During dissection of the bladder off the lower uterine segment,
extravasation of urine is noted, and a 3-cm cystotomy is recognized above the
trigone. On completion of the hysterectomy, the gynecologist performs a 1-layer
cystotomy repair with absorbable suture. The most likely outcome over the next 8
weeks is
(A) Vesicovaginal fistula
(B) Ureteral obstruction
(C) A normal postoperative course
(D) Recurrent bouts of cystitis
(E) Intraperitoneal urinoma

81

Gyna & Obstetrics Reveision by/Dr.adool

2008

URINARY INCONTINENCE
1. Causes of urinary incontinence in the female include:
A. Stress incontinence.
B. True incontinence.
C. False incontinence.
D. Cervical incompetence.
E. Delayed puberty.
2. Normal mechanism of continence involves:
A. The sphincter at the uretero-vaginal junction.
B. The fascia around the sphincter.
C. Vascularity and edema of the urethral mucosa.
D. The intra-abdominal position of the lower urethra.
E. The intra-vesical pressure is higher than the intra-urethral pressure.
3. Incontinence of urine:
A. Is commonly associated with prolapse.
B. Stress incontinence is more common in older females.
C. May be related to repeated vaginal deliveries.
D. May be due to urethra-vaginal fistula.
E. Should be treated by surgery
4. Incontinence of urine may occur with:
A. A Vesico-vaginal fistula.
B. A uretero-vaginal fistula.
C. A urethra-vaginal fistula.
D. A urethral diverticulum.
E. Severe cystitis.
5. The most common cause of urinary incontinence in a menopausal female is:
A. Overflow incontinence.
B. Vesico-vaginal fistula.
C. Weakness of the fascia around the sphincter.
D. Urgency incontinence.
E. Nocturnal enuresis.
6. Stress incontinence:
A. Means that the patient has to strain to evacuate the bladder.
B. Is commonly associated with a stressful desire to micturate.
C. Is more common in multiparous women.
D. Occurs in all cases of prolapse.
E. Pelvic floor exercise is one of the lines of treatment.
7. Causes of stress incontinence include:
A. Repeated cesarean sections.
B. Postmenopausal atrophy of the fascia around the sphincter.
C. Excessive fibrosis around the bladder neck.
D. Prolonged stress.
E. Some antihypertensive drugs.

82

Gyna & Obstetrics Reveision by/Dr.adool

2008

8. Diagnosis of stress incontinence:


A. Clinical examination, asking the patient to strain.
B. Urine analysis.
C. Bonney's test.
D. Urodynamic studies to calculate the vesical closure pressure.
E. Cystourethrography to visualize the urethro-vesical angle.
9. Investigations for a case of stress incontinence include:
A. Urine analysis.
B. Cystoscopy.
C. Non-stress test.
D. Cysto-urethrography.
E. Intra venous pyelography.
10. Treatment lines for stress incontinence may include:
A. Avoiding stress.
B. Anabolic steroids.
C. Classical repair and Kelly's suture.
D. Kegel's exercises.
E. Burch operation.
11. Urgency incontinence:
A. May be due to urinary tract infection.
B. Is characterized by escape of urine on cough.
C. Is not associated with a desire to micturate.
D. Is treated by Kelly's suture.
E. Improves with treatment of prolapse.
(12)Paradoxical true incontinence indicates:
A) Uretero-vaginal fistula.
B) Large vesico-vaginal fistula.
C) Urethero-vaginal fistula.
(13)In a low urethero-vaginal fistula, the patient complains of:
A) True incontinence.
B) Stress incontinence.
C) Double stream at micturition.
(14)Necrotic vesico-vaginal fistula develops:
A) 2nd postoperative day after hysterectomy.
B) 2nd postoperative day after classical repair.
C) 7 days after prolonged vaginal delivery.
D) All of the above.
(15)In a three-cotton-piece test, the first piece is not discoloured, but soaked with urine. This indicates:
A) Tiny vesico-vaginal fistula.
B) Valvular vesico-vaginal fistula.
C) Uretero-vaginal fistula.
D) All of the above.

83

Gyna & Obstetrics Reveision by/Dr.adool

2008

(16)During cystoscopy, for true incontinence, the following is true, (Mark the WRONG statement:
A) Fistula appears as a hole in the bladder.
B) No hole is seen in the bladder wall in ureteric fistula.
C) Urine appears red-coloured after indigo-carmine test.
D) None of the above.
(17)Surgical operations for the repair of vesico-vaginal fistula include the following, ( Mark the WRONG
statement):
A) Dedoublement operation.
B) Saucerization operation.
C) Trans abdominal repair.
D) All of the above.
E) None of the above.
(18)The ureter is liable to injury in the following sites, EXCEPT:
A) During clamping of the infundibulo-pelvic ligament.
B) During clamping of the ovarian ligament.
C) During clamping of the uterine artery.
D) During clamping of the uterosacral ligament.
E) During clamping of the bladder pillars.

stress incontinence
1- The commonest cause of stress incontinence is:
A) Constipation
B) Raised intra abdominal pressure
C) Congenital weakness of sphincter
D) Childbirth trauma
E) Estrogen deficiency
2- Stress incontinence is recognised to occur with the following conditions EXCEPT:
A) Multiparity
B) Uterovaginal fistula
C) Previous pelvic surgery
D) Cystocoele
3- Stress urinary incontenence is characterized by involuntary loss of urine with all of the following EXCEPT:
A) Coughing
B) Exercising
C) Laughing
D) Sleeping
E) Sneezing
4- Diagnosis of genuine stress incontinence is made by which of the following before taking the patient for surgery:
A) History
B) Subjective demonstration of stress incontinence
C) Objective demonstration of stress incontinence
D) Urodynamic studies

84

Gyna & Obstetrics Reveision by/Dr.adool

2008

5- Treatment of choice for a 55 years old multipara with 3rd degree prolapse uterus with cystorectocele and stress
incontinence is:
A) Vaginal hysterectomy with pelvic floor repair
B) Kelly's repair
C) a) + b)
D) a) + colposuspension
6- Best treatment for severe stress incontinence without prolapse is:
A) Pelvic floor exercise
B) Kelly's repair
C) Burch colposuspension
D) MMK operation
E) Urethral collagen implant
7- In the treatment of stress incontinence, all are true EXCEPT:
A) Pelvic floor exercise.
B) An anterior repair may be useful in the presence of cystocele.
C) Colposuspension involves suturing the paravaginal tissues to the round ligaments.
D) The peritoneal cavity is not normally opened during colposuspension.
8- Stress incontinence in famales , all are false EXCEPT:
A) Is always due to urethral sphincter weakness
B) Is always associated with a cystocele
C) Physiotherapy may be indicated
D) An intravenous urogram is mandatory before surgery
9- A 35-year-old woman comes to the clinic for evaluation and management of her stress urinary incontinence
symptoms. She has a 9-month-old daughter, and would like to have another child in about 1 year. Her medical
history is significant only for hypertension. On evaluation, she has a mild cystocele, residual urine volume of 10
mL, normal urinalysis, a hypermobile urethra, and visible loss of urine in the standing position, with cough. She is
only interested in nonsurgical treatment options. The best management option is:
A) ring pessary
B) periurethral collagen injections
C) pelvic floor physiotherapy
10.A 36 years old lady presents with increasing leakage of urine since her last delivery which was 2 years ago leakage of
urine occurs on coughing ,sneezing or laughing . On examination there was no abdominal masses , vaginal examination
revealed mild uterine enlargement with 1st degree uterine prolapse and 1st degree R.V.F. uterus with moderate cystocele
and rectocele but failed to demonstrate any urinary leakage on coughing .
I) What others points should be evaluated by history taking:
a- patient gravidity & parity
b- mode of last delivery
c- the presence of rectal symptoms ( constipation).
d- The presence of chronic cough
e- All of the above.
II) What other points that should be evaluated by examination:
a- examination after reduction of the prolapsed structures.
b- Examination with partially full bladder
c- Examination in the standing position .
d- All of the above.

85

Gyna & Obstetrics Reveision by/Dr.adool

2008

III) If urine leakage appears after reduction of the prolapsed tissue


a- This test is called bonneys test
b- The cause of such leakage is weakness at the region of the bladder neck.
c- The best surgical treatment is BURCH colposuspention.
d- Correction of prolapse will improve the condition.
e- Urodynamic studies is important to exclude urinary tract infection.
f- All of the above
g- Non of the above.
VII) Post operative care include
a- Leaving a urinary catheter for 24 hours
b- Clamping the catheter to train the bladder & to avoid bladder hypotonia.
c- Any subsequent deliveries should be with generous episiotomy.
d- None of the above.
e- All of the above.

Say true or false


a.
b.
c.
d.
e.

Obstetrical trauma is the commonest cause of vesico- vaginal fistula


Urethro vaginal fistula is a common cause of incontinence
Latzo operation is done for cases with wide vagina
Closure of the vagina in repair of vesico- vaginal fistula should be watertight
Closure of the vagina in vesico vaginal fistula is made by single layer of delayed absorption suture
using inverted Lambert technique.
f. Catheter should be left for at least 7 days in the postoperative period.
g. Any subsequent delivery following successful repair of fistula is by lower segment cesarean section.
h. MARCHALL MARCHITI KRANTS operation is used to treat stress incontinence in a lady associated with
prolapse.

Old complete perineal tear


1. Old complete perineal tear:
A. Patients are incontinent to flatus and fluid stools.
B. There is loss of anal corrugations posteriorly.
C. In some cases, the mucosa of the rectum can be seen.
D. Enema is contraindicated preoperatively to avoid extension of the tear.
E. Postoperative care involves low residue diet and daily enema.
2. Old complete perineal tear:
A. Patients commonly develop vaginitis.
B. The dimple on the sides of the anus becomes lost.
C. Preoperative preparation involves low residue diet for 5 weeks.
D. Intestinal antiseptics are given preoperatively.
E. Surgery should be carried out immediately to minimize complications.
3. Recto-vaginal fistula:
A. Is usually traumatic.
B. All patients are incontinent to stools.
C. Affected cases do not develop vulvitis.
D. Cases following hysterectomy require Latzko operation.
E. Some cases are treated by converting the lesion into complete perineal
tear followed by repair.

86

Gyna & Obstetrics Reveision by/Dr.adool

2008

4.M ark the wrong statement in the case of a third degree perineal tear:
A) Patient is incontinent to fluid and gaseous stools.
B) Vagina and anus are separated by a transverse band.
C) Patient complains of vagino uterine descent.
D) Corrugations around the anus are U-shaped ending is two lateral dimples.
5.During repair of a third degree perineal tear, the landmark of the torn external anal sphineter is:
A) Two dimples to the sides of the anus.
B) Lateral to the edges of the torn rectum.
C) The levator ani muscles.
D) None of the above.
6.Dieting, after repair of a third degree perineal tear, is:
A) Low-residue diet for three weeks.
B) High calorie/fibrous diet.
C) Routine diet.
D) Liquid diet for three weeks.
7.Repair of recto-vaginal fistula includes the of following:
A) Dedoublement operation.
B) Conversion to a third degree tear then repair.
C) Restin Noble operation.
D) Colostomy then repair.
E) All of the above.

Say true or false:


a.
b.
c.
d.
e.
f.
g.
h.
i.
j.
k.
l.
m.
n.
o.
p.
q.
r.

Forceps delivery may lead to perineal, cervical lacerations or rupture uterus.


Vulval hematoma may extend to the broad ligament.
Cervical laceration may lead to broad ligament hematoma.
Rupture scar of LSCS is usualy incomplete.
Annular detachment of the cervix may lead to severe post-partum hemorrhage.
Cervical lacerations mostly affect the lateral angle of the cervix.
Cervical laceration may in the form of button hole of the cervix.
Ureteric injury is common with cervical lacerations.
Hysterectomy is the only line of treatment of rupture uterus.
Vaginal laceration may occur as an extention from perineal or cervical lacerations.
Concealed accidental hemorrage and perforating mole may be a cause of rupture uterus during pregnancy.
Placenta previa increases the risk of rupture uterus.
Vulval varcosities increases the risk of vulval hematoma.
Fetal distress is an early sign of rupture uterus.
Tenderness over the scar may a sign of threatened rupture uterus.
Hysterectomy done for cases of rupture uterus is usualy panhysterectomy.
In cases of rupture uterus shock is usuall propotinate to the amount of vaginal bleeding.
Vaginal pack for 24 hours is essential after treatig vaginal or cervical lacerations.

87

Gyna & Obstetrics Reveision by/Dr.adool

2008

(8) * Diseases of vulva& Infection


1. Primary vulvitis may be due to:
A. Syphilis.
B. Urinary incontinence.
C. Condylomata acuminata.
D. Herpes genitalis.
E. Glycosuria.
2. Primary vulvitis may be due to:
A. Lymphogranuloma inguinale.
B. Furunculosis.
C. Vaginal discharge.
D. Bilharziasis.
E. Tuberculosis.
3. Primary vulvitis may be due to:
A. Tinea cruris.
B. Allergy.
C. Parasitic infestation.
D. Lymphogranuloma inguinale.
E. Vaginal discharge.
4. Secondary vulvitis may be due to:
A. Urinary incontinence.
B. Glycosuria.
C. Candidiasis.
D. Complete perineal tear.
E. Vaginal discharge.
5. Secondary vulvitis may be due to:
A. Lymphogranuloma inguinale.
B. Furunculosis.
C. Rectovaginal fistula.
D. Oxyuris infestation.
E. Intertrigo.
6. Acute vulvitis may be due to:
A. Herpes genitalis.
B. Secondary vulvitis.
C. Bilharziasis.
D. Diabetic vulvitis.
E. Bartholinitis.
7. Clinical picture of acute vulvitis includes:
A. Local pain.
B. Local edema.
C. Itching.
D. Low backache.
E. High fever.

88

Gyna & Obstetrics Reveision by/Dr.adool

2008

8. Management of acute vulvitis includes:


A. Local hygiene.
B. Local antiseptic.
C. Anti allergic.
D. Biopsy from the lesions.
E. Dilatation and curettage.
9. Bartholinitis:
A. May be due to pyogenic or chlamydial infection.
B. Causes a tender swelling in the inferior third of the labia majora.
C. May turn into Bartholin's cyst.
D. May turn into Bartholin's abscess.
E. Is treated by marsupialization.
10. Appropriate treatment for an acute Bartholin's abscess includes:
A. Analgesics.
B. Local heat.
C. Antibiotics.
D. Incision and drainage.
E. Marsupialization.
11. Pruritis vulvae:
A. In the majority of cases pruritis is not associated with vaginal discharge.
B. May be due to vulval dystrophies.
C. Neurodermatitis is not a cause.
D. May be secondary to anal pruritis.
E. The condition is more evident in diabetic patients.
12. The following conditions may cause pruritis vulvae:
A. Vaginal moniliasis.
B. Hypertrophic dystrophy.
C. Urinary incontinence.
D. Chancre.
E. Neurodermatitis.
13. Vulval dystrophies:
A. Are more common during the childbearing period.
B. Are classified as sexually transmitted diseases.
C. May cause hypertrophic lesions.
D. May cause atrophic lesions.
E. Some cases require a biopsy.
14. Vulval dystrophy:
A. Usually results from chronic irritation of the vulva.
B. All types are precancerous.
C. The vulval skin is always thickened.
D. Present with vulval pain.
E. Is best treated by vulvectomy.
15. Causes of vulval ulcers include:
A. Chancre.

89

Gyna & Obstetrics Reveision by/Dr.adool

2008

B. Chancroid.
C. Gonorrhea.
D. Tuberculosis.
E. Rodent ulcer.
16. Swellings of the vulva may be:
A. Congenital hypertrophy of the labia.
B. Imperforate hymen.
C. Hematoma of the vulva.
D. Edema due to pelvic congestion.
E. Bartholin's abscess.
17. Swellings arising in the vulva may be:
A. Condylomata acuminata.
B. Brenner tumor.
C. Sebaceous cyst.
D. Dermoid cyst.
E. Fibroid polyp.
18. Swellings arising in the vulva may be:
A. Endometrioma.
B. Fibroma.
C. Primary carcinoma of the vulva.
D. Metastases from choriocarcinoma.
E. Desmoid tumor.

19. Bartholin's cysts:


A. Are painless swellings.
B. Should be excised.
C. Are situated in the inferior 1/3 of the labia majora.
D. Are usually bilateral.
E. May become infected by gonococci.
20. Carcinoma of the vulva:
A. A history of chronic vulval irritation is common.
B. Pelvic irradiation is a predisposing factor.
C. The main route of spread is lymphatic.
D. Stage Ia is VIN.
E. Radiotherapy is recommended for cases with involved pelvic lymph nodes.
21. Carcinoma of the vulva:
A. Is more common in old age.
B. Infection with human papilloma virus is a predisposing factor.
C. In stage II, the tumor extends to the lower urethra or the perineum.
D. Local excision is sufficient for microinvasive lesions.
E. Radical vulvectomy is required for the majority of cases with advanced disease.
22. Carcinoma of the vulva:
A. Is usually an adenocarcinoma.
B. Occurs commonly on top of hypertrophic dystrophy.

90

Gyna & Obstetrics Reveision by/Dr.adool

2008

C. Does not ulcerate until it is advanced.


D. The most important route of spread is through lymphatics.
E. Surgery and radiotherapy give similar results.
23- An 82-year-old woman underwent a vulvar biopsy for a 3-cm ulcerated lesion on the left labium majus, which had
been present for several years. The pathology revealed a basal cell cancer. Your treatment should be:
A) wide local excision
B) wide local excision and postoperative radiation therapy
C) radical partial left vulvectomy with ipsilateral inguinofemoral lymphadenectomy
D) radical vulvectomy
E) radical vulvectomy with bilateral inguinofemoral lymphadenectomy
24- A 63-year-old woman presents with a 2-cm erythematous lesion on the left labium majus. Biopsies of the lesion
reveal Grade 3 vulvar intraepithelial neoplasia ( VIN3). The optimal treatment for this patient is:
A) simple vulvectomy
B) wide local excision
C) laser vaporization
D) topical 5-fluorouracil
E) radical excision followed by local radiotherapy
25- Vulval Dystrophy, all are true EXCEPT:
A) Associated with pruritis
B) Extends to perineum
C) Premalignant condition
D) Malignant condition
E) Histological diagnosis
26- Vulval Cancer, all are true EXCEPT:
A) is more common than endometrial cancer
B) metastasises via the blood stream
C) is treated with radiotherapy
D) has a peak incidence at age 55
E) is associated with vulval warts
F) the tumour is resistant to chemotherapy
27- Vulval dystrophy (Lichen Sclerosus), all are false except:
A) is usually found in patients with diabetes
B) responds well to steroid ointments
C) usually clears spontaneously after a few years
D) is not associated with vulval ulceration
28- Lichen sclerosusthe following are true EXCEPT:
A) A hypoplastic dystrophy
B) There is inactive epithelium
C) estrogen creams is not benificial
D) Dyspareunia
29- Hyperplastic dystrophythe following are true EXCEPT:
A) Leads to introital contraction
B) Acanthosis
C) Hydrocortisone will help itching
D) Characterized by deepening of rete pegs

91

Gyna & Obstetrics Reveision by/Dr.adool

2008

30- Malignant neoplasms of the vulva ,the following are true EXCEPT:
A) Most are squamous in type
B) There are suspicious groin nodes palpable in stage II disease
C) The verrucous type. has a poor prognosis
D) Groin node dissection can be dispensed in all lesions where the depth of invasion is less than 5 mm
40- Recognised complications of radical vulvectomy and bilateral groin dissection include all EXCEPT:
A) Osteitis pubis
B) Inguinal hernia
C) Paraesthesia over the anterior surface of the thigh
D) Thromboembolic disease
E) Varicose veins
41- In patients with carcinoma of the vulva, lymphatic drainage characteristically:
A) is to the periaortic nodes
B) is to the superficial inguinal lymph nodes and from the clitoral region to the deep femoral lymph nodes
C) bypasses the deep femoral lymph nodes
D) None of the above
42- Melanoma of the vulva, all are false EXCEPT:
A) constitutes 2 to 9 percent of most series of vulvar cancer
B) occurs mostly in the fifth decade
C) occurs mostly in premenopausal women
D) has an overall survival rate of 70 percent
E) is non aggressive
43- A woman is found to have a unilateral, invasive vulvar carcinoma that is 2 cm in diameter but that is not
associated with evidence of lymph node spread. Initial management of this woman most likely would consist of:
A) chemotherapy
B) radiation therapy
C) simple vulvectomy
D) radical vulvectomy
E) radical vulvectomy and bilateral inguinal lymphadenectomy
44- Which of the following statements can characterize epidermoid carcinoma of the vulva?
A) It is associated with an increased incidence of epidermoid carcinoma of the endocervix
B) It is seen less frequently than adenocarcinoma
C) It tends to develop in women who are younger than those affected by adenocarcinoma
D) It tends to be more advanced when diagnosed than adenocarcinoma
45- Lichen sclerosis is characterized by all EXCEPT:
A) blunting or loss of the rete ridges
B) development of a homogeneous subepithelial layer in the dennis
C) a band of chronic inflammatory infiltrate below the dermis
D) an increase in the number of cellular layers in the epidermis

92

Gyna & Obstetrics Reveision by/Dr.adool

2008

(9) * Endometriosis
(1)Endometriosis means the presence of endometrial tissue in the following, EXCEPT:
A) In the lining of the uterine cavity.
B) Within the muscle layer of uterus.
C) In an ovarian cyst.
D) Within the peritoneum.
(2)Adenomyosis may be Associated with the following, EXCEPT:
A) Dysmenorrhaea.
B) Menorrhogia.
C) Amenorrhoea.
D) Infertility.

(10) * CANCER CERVIX


(1)In the oetiology of cervical dysplasia and cancer cervix, which of the following is MOST TRUE:
A) Both are caused by HPV.
B) Both are, PROBABLY, venereal disease.
C) They are associated with smoking, multiparity & obesity.
D) Both have genetic predisposition.
E) Both are caused by HSV-2.
(2)As regards HPV, following is true EXCEPT:
A) 80% of normal women have HEV/DNA (16, 18) in their cervical tissue.
B) 80 90% of dysplastic cells contain HPV sequences.
C) HPV 6,11 are associated with severe forms of dysplasia.
D) It is sex-transmitted.
E) Causes vacuolation of vaginal cells in pap-smear.
(3)Which of following is TRUE as regards Herpes simplex virus:
A) Sufficient evidence correlates HSV2 to cancer cervix.
B) May cause of warty mass on the cervix.
C) Genital region can be affected by HSV-1.
D) All of the above.
E) None of the above.
(4)Synonems for cervical intraepithelial neoplasia include:
A) Carcinoma in situ.
B) Cervical dysplasia.
C) Squamons intraepithelial lesion.
D) All of the above.
(5)The most common method for diagnosis of CIN is:
A) Complaints of abnormal discharge.

93

Gyna & Obstetrics Reveision by/Dr.adool

2008

B) Postcoital bleeding.
C) Chronic pelvic pain.
D) Vaginal WET smear.
E) Abnormal Pap. Smear.
(6)In Schillers Iodine test, the following is true, EXCEPT:
A) Schillers positive areas are stained brown.
B) Schillers positive areas are not stained brown.
C) Not all positive areas are malignant or premalignant.
D) Schillers positive areas are to be biopsied.
(7)In CIN the following is true EXCEPT:
A) CIN I has irrigular enlarged nuclei with abnormal mitosis.
B) CIN III includes severe dysplasia and carcinoma in situ.
C) CIN III lesions gradually blend will the adjacent normal epithelium.
D) Histological examination is needed to exclude active metaplasia.
E) CIN may involve the cervical glands.
(8)Cellualr atypia involving the basal cervical epithelium and more than one half of epithelial thickness is:
A) Adenocarcinoma.
B) Microinvasive carcinoma.
C) CIN II.
D) Carcinoma in situ.
E) Invasive squamous cell carcinoma.
(9)In 40-year-old patient, clinically normal but with Pap. Smear High Grade Squamons lntraepithelial lesion, H-SIL,
is managed by:
A) Fractional curettage.
B) Reexamination with colposcope-directed biopsy.
C) Repeat Pap-smear.
D) Punch biopsy.
E) Immediate hysterectomy
(10)Unsatisfactory Pap. Smear may b e due to:
A) Missing endocervical sampling.
B) Severe vaginitis.
C) Extensive malignant necrosis.
D) Misreading.
E) All of the above.
(11)In Pap-smear, false-negative results are found in:
A) Less than 10%.
B) 25%.
C) 50%.
D) More than 50%.

94

Gyna & Obstetrics Reveision by/Dr.adool

2008

(12)In sexually active females, Pap. Test is done:


A) Quarterly.
B) Half-annually.
C) Annually.
D) Every 3 years.
E) When attending a gyn. Clinic.
(13)Colposcopy permits one to:
A) View the cervix at 1 4 power magnification.
B) See the entire transition zone in all patients.
C) Choose the most suspicous site to biopsy.
D) Treat invasive cancer with a biopsy.
E) Make the diagnosis of cancer.
(14)On colposcopy, the factor MOST SUSPICIOUS for invasive cervical carcinoma is:
A) Aceto - white area.
B) Leukoplakia.
C) Abnormal blood vessels.
D) Punctation.
E) Mosaic pattern.
(15)Leucoplakia refers to:
A) A microscopic lesion.
B) A white lesion.
C) Shrinking atrophy.
D) A cancer.
(16)About colposcopy, which of the following is correct:
A) Mosaic pattern suggests CIN lesion.
B) Punctate pattern suggests CIN lesion.
C) Atypical blood vessels suggest invasive carcinoma.
D) CIN III is clearly delineated.
E) All of the above.
(17)Choose the correct statement as regards colposcopy:
A) It provides a magnification of approximately x 50.
B) It obviates the need for tissue diagnosis.
C) Aceto-white epithelium, in a young woman, mostly indictes squamous metaplasia.
D) Transformation zone is found medial to squomo-columnar junction.
E) Leucoplakia becomes visible after acetic acid application.
(18)The following statements are true, as regards colposcopy, EXCEPT:
A) Vascular pattern is apparent after use of green filter.
B) The transformation zone is absent in young adults.
V) Squamo-columnar junction is MOSTLY found at the external os.
D) Immature squamous metaplasia mimics neoplasia.

95

Gyna & Obstetrics Reveision by/Dr.adool

2008

E) Trichomonas vaginitis produces a punctate vascular pattern.


(19)Microcolpohysteroscopy, entails the, following EXCEPT:
A) Endometrail cavity, endocervical canal and vagina are visualized.
B) The hysteroscope is used.
C) The colposcope is used.
D) Detects the transformation zone in a post menopausal woman.
E) Allows for biopsy of an endocervical lesion.
(20)Limitations of colposcopy include.., EXCEPT:
A) It is poorly specific (30% false positive).
B) Cannot be done during pregnancy.
C) Endocervix cannot be visualized.
D) Inaccessable for screening.
E) Cannot distinguish between CIN and condylomate accuminata, without vascular pattern.
(21)Power of magnification of the colposcope is:
A) 10x to 40x.
B) 50x to 75x.
C) 80x to 100x.
D) 100x to 150x
E) 200 x.
(22)Columnar epithelium of endocervix appears as, at colposcopy:
A) Irrigular surface with long papilla.
B) Grape-like after acetic acid application.
C) Deep clefts.
D) All of the above.
E) None of the above.
(23)The squamocolumnar junction is present in:
A) The external os.
B) Outside the external os.
C) Inside the endocervical canal.
D) Position changes with age.
E) Transitional zone.
(24)Conization would be INAPPROPRIATE WHEN:
A) There is disparity between Pap. Smear and biopsy results.
B) When microinvasive carcinoma is found at biopsy.
C) Biopsy reports deeply invasive carcinoma.
D) Inadequate colposcopy.
E) All of the above.
(25)Advantages of conization are:
A) Safe procedure.
B) Accurate diagnosis.
C) May be therapeutic.

96

Gyna & Obstetrics Reveision by/Dr.adool

2008

D) Endocervical involvement-detection
E) Suitable for large lesions.
F) All of the above.
(26)Following is true, about coinization, EXCEPT
A) Needs general anesthesia.
B) May be followed by cervical stenosis.
C) Compromizes cervical function.
D) Risky if done during pregnancy (abortion, bleeding).
E) Adequate for treatment of adenocarcinoma in-situ.
(27)Conization is not always curative. The risk of recurrence of cervical malignancy after conization is:
A) 5%.
B) 10%.
C) 20%.
D) 30%.
E) 50%.
(28)CIN may progress to invasive carcinoma, within ten years in up to which percentage of patients:
A) 5%.
B) 30%.
C) 60%.
D) 80%.
E) 100%.
(29)CIN, may regress spontaneously:
A) True
B) False
(30Most cervical cancers arise in:
A) Internal os.
B) Endocervical canal.
C) Squamocolumnar junction.
D) Isthmus.
E) External os.
(31)In cervical adenocarcinoma, there is.. EXCEPT:
A) Incidence 15 20% of all cervical carcinoma.
B) Manifests early in tumour-life.
C) Cervix is barrel-shaped, rock-hard on palpation.
D) May develop after DES exposure.
E) May be seen in children 12 years old.
(32)In clinical-stage I cervical carcinoma, lymph node involvement is present in:
A) Less than 1%.
B) 5%.
C) 15%.

97

Gyna & Obstetrics Reveision by/Dr.adool

2008

D) 25%.
E) 50%.
(33)In micro-invasive cancer cervix (Stage IA), lymph node involvement is found in:
A) Less than 1%.
B) 5%.
C) 15%.
D) 25%.
E) 50%.
(34)Cancer cervix with parametric involvement not reaching the pelvic wall, and bilateral hydronephrosis due to
ureteric obstruction by malignant tissue is staged as:
A) Stage O.
B) Stage IA
C) Stage IB.
D) Stage IIA
E) Stage IIB
F) None of the above.
(35)Cancer cervix with involvement of the upper third of vagina ONLY, and at cystoscopy, bullous oedema of the
bladder is found. It is stage:
A) O.
B) I
C) IIA.
D) IIB.
E) III.
F) IV.
(36)Prognosis and clinical behaviour of cervical adenocarcinoma as compared to squamous carcinoma, stage to stage
are:
A) Same.
B) Worse.
C) Better.
(37)Cancer cervix infilterating the stroma to a 7 mm depth with vascular space involvement is stage as:
A) O.
B) IA1.
C) IA2
D) II.
E) III.
F) IV.
(38)All following statements are true as regards prognosis of cervical corcinoma EXCEPT:
A) Age of patient.
B) Histological type.
C) Vascular space involvement.
D) Histological grade.
E) It depends only on staging of cancer cervix.

98

Gyna & Obstetrics Reveision by/Dr.adool

2008

F) Lymph node involvement.


G) Size of the tumour.
(39)Clinical staging, of cancer cervix, will correlate with surgical finding in accuracy of which percentage:
A) 10.
B) 25.
C) 50.
D) 75.
E) 100.
(40)Cancer cervix stage III is assigned when
A) Carcinoma infiltrating, the bladder.
B) Carcinoma involving the distal vaginal wall.
C) Carcinoma involving the parametrium short of pelvic wall.
D) Carcinoma with pelvic lymph node involement.
E) Carcinoma with bullous oedema of the bladder.
(41)Often, the first symptom of cancer cervix is:
A) Leg pain.
B) Pain with intercourse.
C) Pruitus vulvae.
D) Vaginal bleeding.
E) Weight loss.
(42)A buky friable papillary mass, growing from the cervix is BEST termed:
A) Exophytic.
B) Endophytic.
C) Ulcerative.
D) Nodular.
E) Oedematous.
(43)Which of the following carcinomas carries the poorest prognosis (other factors being equal):
A) Well-differentiated keratinizing large-cell squqmous Carcinoma.
B) Poorly-differentiated, large-cell squamous carcinoma.
C) Well-differentiated, keratinizing small-cell squamous Carcinoma
D) Poorly differentiated, small-cell squamous carcinoma.
E) Adenocarcinoma.
(44)A cervical ulcer is BEST evaluated by:
A) Repeated examination.
B) Pap. Smear.
C) Colposcopy.
D) Punch biopsy.
E) Hysterectomy.
(45)42-year old woman, with postcoital spotting, shows circumscribed red area in the cervix. It could be.
A) Condylomatra.

99

Gyna & Obstetrics Reveision by/Dr.adool

2008

B) T.B ulcer.
C) Ectopoy.
D) Ectropion.
E) Cancer.
F) All of the above.
(46)43-year old woman, with normal pelvic findings and suspicious Pap. Smear, shows invasive carcinoma of the
cervix on colposcopically-dircted biopsy. Next management is:
A) Cone biopsy.
B) Hysterectomy (radical).
C) Metastatic evaluation.
D) Irradiation.
E) Both irradiation and radical hysterectomy.
(47)Limitations of cryosurgery for CIN include the following EXCEPT:
A) Does not allow for histological tissue sampling
B) Does not deal with endocervical lesions.
C) Painful procedure.
D) Not suitable for lesions > 1 cm.
E) High failure rate (6 32%) and in CIN III it is 20 40%.
(48)Side Effects of Cryocautery are:
A) Uterine cramping.
B) Light headedness.
C) Dizziness.
D) Excessive watery discharge.
E) All of the above.
(49)Laser treatment of CIN suffers the following disadvantages:
A) High cost.
B) Special training.
C) Not superior to cryotherapy with lesions < 1 cm.
D) All of the above.
E) None of the above.
(50)Following statements are true as regards laser vaporization of CIN, EXCEPT:
A) Uses carbon dioxide.
B) Done without general anaesthesia.
C) Obviates the need for biopsy.
D) Can destroy up to 7 mm depth.
E) 90% cure rate.
(51)In LEEP (loop Electrosurgical Excision Procedure), the following statements are true, EXCEPT:
A) High power (35 55 watts) is used.
B) Tiny loop (0.5 mm).
C) There is little thermal damage.
D) Electron flow will create high velocity steam current which will cut in tissues.
E) Suitable for large CIN lesions.

100

Gyna & Obstetrics Reveision by/Dr.adool

2008

(52)The majority of deaths for cervical carcinoma are due to:


A) Local spread, obstructing the ureters causing renal failure.
B) Brain metastasis with resultant cerebral failure.
C) Hoemorrhage into the pelvis from vascular erosion by tumour.
D) Pulmonary failure secondary to lung metastasis.
E) Bone metastasis causing injuries to the central nervous system.
(53)The approximate five-year survival rate for all stages of cervical carcinoma (together) is:
A) 15%
B) 35%.
C) 50%.
D) 65%.
E) More than 65%.
(54)Occult carcinoma of the cervix is:
A) Stage IA.
B) Microinvasive.
C) Best treated by cone biopsy.
D) Found incidentally in hysterectomy specimens.
E) Frequently associated with negative cytology.
(55)Conization is indicated in:
A) Cases with endocervical involvemnt.
B) Unvisualized squamo columnar junction..
C) Unsatisfactory calposcopic findings.
D) Large lesions.
E) All of the above.
(56)Whertheims hysterectomy is chosen for cases of :
A) Stage Ia1.
B) Stage Ia2.
C) Stabe Ib.
D) Stage IIa.
E) Stage IIb.
(57)In a 26-year old patient with stage I cancer cervix, choice of treatment modality takes the following in
consideration:
A) Desire to maintain reproductive life.
B) Size of tumour.
C) Depth of invasion.
D) Vascular space involvement.
E) All of the above.
(58)Extended radical hysterectomy (Meigs or type III hysterectomy)is indicated in stages of cancer cervix:
A) Ib.
B) IIa
C) IIb.
D) III.

101

Gyna & Obstetrics Reveision by/Dr.adool

2008

E) (a + b).
F) (a + C)
G) (b + c).
(59)Mark the true step in radical hysterectomy:
A) Dissection is carried down to the deep inginal nodes.
B) The uterine vessels are ligated medial to the ureters.
C) The common iliac nodes are removed.
D) Ureter is dissected FREE from pelvic brim to bladder.
(60)In radical hysterectomy, which statement in INAPPROPRIATE:
A) Suction drainage of the pelvic floor reduces the incidence of lymphocyst formation.
B) Bladder atony is common after operation.
C) The obturator nerve is rarely seen.
D) Genito-femoral nerve is frequently damaged.
E) The superior vesical artery is not ligated.
(61)In stage I cervical carcinoma, the ovaries SHOULD be removed:
A) Yes.
B) No.
(62)Radiotherapy of cancer cervix is associated with the following EXCEPT:
A) Reduced incidence of lymph-node involvement at subsequent lymphadenectomy.
B) More effective in cases of adenocarcinoma than in squamous cell carcinoma.
C) Megavoltage therapy has a lower incidence of skin damage than autovoltage (200 300 Kv).
D) Point A is 2 cm above the lateral fornix and 2 cm lateral to the central canal of the cervix.
E) Oestrogen therapy prevents vaginal atrophy.
(63)The most common post-operative complication of radical hysterectomy is:
A) Intra-operative death.
B) Genito-urinary fistula.
C) Bowel injury.
D) Premature menopouse.
(64)The following procedures can be performed for cases of cancer cervix with pregnancy less than 20 weeks EXCEPT:
A) Repeated Pap. Smear.
B) Expert colposcopy.
C) Conization.
D) I.V.P. for staging.
E) Cystoscopy for staging.
(65)During early pregnancy (less than 20 weeks), cancer cervix (stage Ib and more) is treated by:
A) Expectant treatment till viability.
B) Conization and follow-up till viability.
C) As in the none pregnant state.
D) All of the above.
E) None of the above.

102

Gyna & Obstetrics Reveision by/Dr.adool

2008

(66)Criteria to treat micro-invasive cervical carcinoma by conization include the following EXCEPT:
A) Microinvasim 3 mm.
B) No vascular space involvement.
C) Desire to preserve fertility.
D) Margins are free from CIN.
E) All of the above.
F) None of the above.
(67)In terminal carcinoma, which of the following is true:
A) It is painless.
B) Lung and/or liver secondaries are common.
C) Requires urinary diversion.
D) Pelvic exentraion is done.
E) Intrathecal alcohol injection is recommended.
68. Risk factors of cervical carcinoma include:
A. Prolonged unopposed effect of estrogen.
B. Nulliparity or low parity.
C. Infection with HPV.
D. Early age of marriage.
E. Late menopause.
69. Cervical intraepithelial neoplasia (CIN):
A. Arise from the squamo-columnar junction.
B. May regress spontaneously.
C. Usually become invasive within 1-2 years.
D. Appear as a red area on the ectocervix.
E. May be completely treated by conization.
70. Cervical intraepithelial neoplasia (CIN):
A. Is also known as ectropion.
B. Is usually asymptomatic.
C. May present with contact bleeding.
D. May present with intermenstrual bleeding.
E. May present with vaginal discharge.
71. Microinvasive carcinoma of the cervix:
A. May be diagnosed by cervical smear.
B. May invade to a depth of 5 mm.
C. Has a maximum width of 7 mm.
D. May invade lymphatic channels.
E. Is treated by Wertheim's operation.
72. Spread of carcinoma of the cervix commonly includes:
A. The ureter.
B. The parametrium.
C. The obturator lymph nodes.
D. The inguinal lymph nodes.
E. The vagina.

103

Gyna & Obstetrics Reveision by/Dr.adool

2008

73. Stage I cancer cervix:


A. Is confined to the uterus.
B. May not be visible on clinical examination.
C. May be associated with hydroureter or hydronephrosis.
D. May be associated with lymphatic spread to iliac nodes.
E. May be treated by external or internal irradiation.
74.In stage II carcinoma of the cervix:
A. The growth is confined to the cervix.
B. There may be extension to the body of the uterus.
C. There may be extension to the upper 1/3 of the vagina.
D. The tumor is fixed to the lateral pelvic wall.
E. The treatment is either radiotherapy or Wertheim's operation.
75. Endocervical carcinoma
A. Is usually an adenocarcinoma.
B. Causes a barrel shaped cervix.
C. Usually causes death from local invasion before metastases.
D. May occur in younger women.
E. May be treated by hysterectomy and radiotherapy.
76. Cervical smears:
A. Should be taken every 10 years.
B. Should be fixed immediately.
C. Should be stained with Schiller's iodine.
D. Cases showing inflammatory changes should be referred for colposcopy.
E. Cases showing dysplastic changes should be referred for guided biopsy.
77. A pap smear shows mild dysplasia, the next step in management would include:
A. Cone biopsy.
B. Treatment of inflammation.
C. Repeating the smear.
D. Hysterectomy.
E. Hysteroscopy.
78.Schiller's iodine applied the cervix will stain:
A. A dysplastic area.
B. A carcinoma.
C. An erosion.
D. Normal columnar epithelium.
E. Mature squamous epithelium.
79. Colposcopy:
A. The squamo-columnar junction is usually visible at the internal os.
B. The transformation zone is usually visible at the external os.
C. Acetic acid is applied to keep the cervix wet.
D. Leucoplakia becomes visible after acetic acid application.
E. Non-visualization of the transformation zone is an indication for 4 quadrant biopsy.

104

Gyna & Obstetrics Reveision by/Dr.adool

2008

80. Conization (cone biopsy) of the cervix:


A. Should be done in all cases of CIN.
B. Should be done in all cases of symptomatic cervical erosion.
C. Is indicated when the upper end of the lesion cannot be visualized.
D. Is indicated when the squamo-columnar junction cannot be visualized.
E. Increases the risk of abortion in subsequent pregnancies.
81. Investigations for adequate staging of cancer cervix include:
A. Bimanual examination under anesthesia.
B. PR examination under anesthesia.
C. Cystoscopy.
D. Lymph node biopsy.
E. Lymphangiography.
82. A carcinoma of the cervix extending to the upper part of the anterior vagina may be treated by:
A. Internal irradiation.
B. External irradiation.
C. Whole pelvic irradiation.
D. Internal irradiation and hysterectomy.
E. Wertheim's operation.
83. Structures removed in Wertheim's operation include:
A. The uterus.
B. The cervix.
C. The upper 2/3 of vagina.
D. The parametrium.
E. The pelvic and femoral lymph nodes.
84. Indications of Wertheim's operation include:
A. Cancer cervix associated with pelvic infection.
B. Cancer cervix associated with prolapse.
C. Radiosensitive tumors.
D. Cancer cervix associated with pregnancy.
E. Some cases of malignant ovarian tumors.
85. Radiotherapy for cancer cervix:
A. May be curative or palliative.
B. Has the same results as Wertheim's operation for stage I lesions.
C. Is contraindicated in the presence of active infection.
D. Should not be used after Wertheim's operation.
E. Consists of either external or internal irradiation.
86. Complications of radiotherapy for cancer cervix include: .
A. Flaring of infection.
B. Pulmonary embolism.
C. Stress incontinence.
D. Vesico-vaginal fistula.
E. Bleeding per rectum

105

Gyna & Obstetrics Reveision by/Dr.adool

2008

87- Which one of the following is not associated with cervical carcinoma?
A) Multiple sexual partners
B) Human papilloma virus (HPV)
C) Young age at first sexual intercourse
D) Intrauterine contraceptive devices (IUCDs)
88- In cervical cancer Stage Ib, which of the following is true?
A) Tumor has spread to adjacent organs.
B) Tumor involves the lower one-third of the vagina
C) Clinically apparent lesion involving only the cervix.
D) Tumor has spread to the upper two-thirds of the vagina and does not involve the parametria.
89- A 42-year-old woman comes for her routine annual examination. She has recently been divorced, and she
acquired a new sexual partner within the past 6 months. She has no history of abnormal Pap test results. Her
examination is unremarkable, but her Pap tests result indicates atypical squamous cells of undetermined
significance (ASCUS). The most appropriate management is:
A) Repeat Pap test in 6 months
B) Colposcopy
C) Viral testing for human papillomavirus (HPV) subtype
D) Screening of new partner for HPV
90- A 26-year old woman who is at 12 weeks of gestation comes for a follow-up visit after her Pap test result at 8
weeks of gestation showed a low-grade squamous intraepithelial lesion (LSIL). The next step in management
should be :
A) Cone biopsy
B) Loop electrosurgical excision procedure (LEEP)
C) Colposcopy
D) No further procedure until after delivery
91- A 46-year-old woman presents with postcoital bleeding. Pelvic examination reveals a large fungating cervical
lesion, 8 cm in maximum diameter, with parametrial extension on the right. Cervical biopsy demonstrates
moderately differentiated squamous cell carcinoma with lymphovascular space involvement. Baseline laboratory
studies reveal a normal serum creatinine and normal blood urea nitrogen, serum hemoglobin of 11.2 mg/dL, and
normal liver function. Chest X-ray shows no evidence of disease, and computed tomography scan shows no
evidence of lymphadenopathy or liver disease. The patient is fully active and able to engage in her usual daily
activities. The currently recommended therapy for her is:
A) whole abdominal radiation therapy
B) pelvic radiation therapy followed by brachytherapy
C) cisplatin (Platinol) chemotherapy concurrent with pelvic radiation therapy followed by brachytherapy
D) pelvic radiation therapy followed by extrafascial hysterectomy
E) radical abdominal hysterectomy followed by pelvic radiation therapy followed by
Brachytherapy
92- Cancer of the cervix, all are true EXCEPT:
A) has reduced in incidence due to cervical screening programmes
B) cigarette smoking is a risk factor
C) the incidence is inversely related to social class
D) is usually adenocarcinoma

106

Gyna & Obstetrics Reveision by/Dr.adool

2008

93- Exfoliative cytology is useful for the diagnosis of:


A) Meningioma
B) Multiple Myeloma
C) Cervical cancer
D) Teratoma of ovary
94- Carcinoma cervix may be caused by which of the following?
A) Herpes simplex type I
B) Syphilis
C) Papilloma virus
D) Trichomonas.
95- The following are the factors associated with CIN EXCEPT:
A) Onset of coitus at early stage
B) Multiple sexual partners
C) Lower socioeconomic status
D) Nulliparity
E) History of venerial disease
96- A 55 year old lady presenting to outpatient department with postmenopausal bleeding for 3 months, has a 1x1
cm nodule on the anterior lip of cervix. The most appropriate investigation to be done subsequently is:
A) Pap smear.
B) Punch biopsy.
C) Endocervical curettage.
D) Colposcopy
97- The time span over which a percentage of women with untreated CIN will develop cancer. Choose the single most
likely time or time-interval:
A) 3-6 months
B) 12 months
C) 24-36 months
D) 3-10 yrs
E) 30 years
98- The age at which the peak incidence of cervical carcinoma develops. Choose the single most likely time or timeinterval:
A) 30 years
B) 45 years
C) 50 years
D) 55 years
E) 65 years
99- The interval for cervical smears to be taken in a woman treated I year previously for CIN 2. Choose the single most
likely time or time-interval:
A) 3-6 months
B) 12 months
C) 24-36 months
D) 3-10 yrs
E) 30 years

107

Gyna & Obstetrics Reveision by/Dr.adool

2008

100- The time scale within which the 50% of the patients with recurrent disease will present, following treatment for
stage IB cervical cancer. Choose the single most likely time or time-interval:
A) 12 months
B) 24-36 months
C) 3-10 yrs
D) 30 years
E) 45 years
101- Regarding Wertheim's hysterectomy
A) Only palpably enlarged lymph nodes are removed.
B) It is the treatment of choice for Stage III carcinoma of the cervix.
C) The ovaries can be conserved.
D) It can be followed by adjuvant radiotherapy.
102- Cervical Polyps, All are true except:
A) are commonly pre-malignant
B) are associated with abnormal bleeding
C) may be fibroids
D) can cause postcoital bleeding
103- The Cervical Smear, all are false EXCEPT:
A) should be performed routinely every five years
B) may show evidence of endometrial cancer
inflammatory" smears should be repeated after three years
D) accurately reflects cervical pathology
104- Early symptoms of invasive cancer of the cervix may include all EXCEPT:
A) Increased vaginal discharge
B) Leg pain
C) Postcoital bleeding
D) Metrorrhagia
105- Occult invasive cervical cancer, all are false EXCEPT:
A) Is stage Ib
B) Associated with microinvasive carcinoma on pathological assessment
C) Rarely is associated with lymph node metastasis
D) Cannot be diagnosed by colposcopy
106- In cervical cancer , all are true EXCEPT:
A) The potential of rnicroinvasion depends on confluence
B) The potential of rnicroinvasion depends on volume
C) If endothelial lined channels contain tumour, then lymphatic spread is likely to have occurred
D) Where the depth of invading nests of cells exceed 5 mm then cone biopsy is warranted
107- In the treatment of stage Ib carcinoma of the cervix by Wertheim's hysterectomy, all are not true EXCEPT:
A) associated with a urinary fistula rate of 10%
B) cause bladder atonia
C) associated with survival rates of about 80% if only one to four pelvic lymph nodes are involved
D) Not associated with femoral nerve damage

108

Gyna & Obstetrics Reveision by/Dr.adool

2008

108- A 25-year old nulliparous woman is referred from her general practitioner with an abnormal smear, This could
mean all of the following EXCEPT:
A) That the smear is unsatisfactory
B) That there are inflammatory changes
C) That there is cervical intraepithelial neoplasia (CIN)
D) That there is a human papilloma virus (HPV) type 3 infection
109- Colposcopy , all are not true EXCEPT:
A) Acetic acid coagulates protein in the nuclei and cytoplasm
B) Only areas of intraepithelial neoplasia appear white
C) In CIN grade III the intercapillary distance is decreased
D) An abnormal vascular pattern suggests invasive disease
E) Columnar epithelium stains dark brown with Schiller's iodine
110- The following are characteristic features of HPV infection EXCEPT:
A) Koilocytic atypia
B) Multinucleation
C) No abnormal mitotic figures
D) Nuclear abnormalities in the basal layers
111- The following statements are not true concerning treatment of CIN EXCEPT:
A) The best long-term success rates have been reported using laser therapy
B) To be effective local destruction to depth of 10 mm is required
C) Cone biopsy is indicated where there is colposcopic suspicion of an invasive lesion
D) No long-term follow-up is required if hysterectomy is performed
112- Management of invasive cancer of the cervix, all are not true EXCEPT:
A) In stage Ib tumours the size of the tumour has little relevance to the 5-Year survival
B) In stage II disease there is extension of the tumour from the cervix into the uterine cavity
C) Barrel-shaped endocervical lesions have a worse prognosis than cervical cancers of a similar stage
D) Adenocarcinoma of the cervix has a poorer survival stage for stage than squamous lesions
113- Pelvic exenteration is contraindicated where:
A) There is unilateral leg edema and sciatica
B) There is pelvic fibrosis
C) The patient is sexually active
D) There is central recurrence after radiotherapy
114- A patient in her first trimester of pregnancy is diagnosed with invasive cervical cancer. Treatment would include:
A) waiting until after delivery before treatment
B) frequent pap.smear through gestation
C) Termination of the pregnancy and concomitant or subsequent radiation or surgery as soon as possible)
D) delivery by cesarean section at term
E) radiation only
115- Most deaths from cervical carcinoma can be attributed to:
A) local extension
B) metastasis to the centra! Nervous system
C) metastasis to the liver and lungs
D) iatrogenic causes
E) none of the above

109

Gyna & Obstetrics Reveision by/Dr.adool

2008

116- An exentration procedure for recurrent cervical cancer would include all of the following EXCEPT:
A) removal of the bladder
B) removal of the uterus
C) removal of the symphysis
D) removal of the rectum
E) removal of the pelvic nodes
117- Carcinoma in situ of the cervix is characterized by all of the following EXCEPT
A) involvement of the entire thickness of the squamous epithelium
B) cells resembling those of invasive carcinoma
C) evidence of stromal invasion
D) complete loss of stratification
E) occasional regression and disappearance
118- An intravenous pyelogram showing hydronephrosis in the work-up of a cervical cancer otherwise confined to a
normal size cervix would mean
A) stage I
B) stage II
C) stage III
D) stage IV
E) none of the above
119- True statements about clear cell adenocarcinoma of the cervix and vagina include the following EXCEPT:
A) It has been related lo prenatal DBS exposure
B) It shows a hobnail pattern on microscopic examination
C) It usually appears as an exophytic lesion
D) Affected women are best treated with radiotherapy
120- A patient presents with a Pap smear showing mild dysplasia. The next steps in management would include which
of the following?
A) hysterectomy
B) repeat Pap smear
C) cone biopsy
D) colposcopy
121- Cervical carcinoma is considered invasive when there are all of the following EXCEPT:
A) a breakthrough of the basement membrane
B) penetration of the stroma
C) involvement of the lymphatics
D) involvementoftheendocervical glands
E) All of the above
123- Studies show that cervical cancer is associated with which of the following?
A) early first coitus
B) incidence of cigarette smoking
C) nulliparity
D) use of oral contraceptives

110

Gyna & Obstetrics Reveision by/Dr.adool

2008

124-As regarding cancer cervix Say true or false and explain why
a.
b.
c.
d.
e.
f.
g.
h.
i.
j.
k.
l.
m.
n.
o.
p.
q.
r.
s.
t.
u.
v.
w.
x.
y.
z.
aa.
bb.
cc.

Cancer cervix can affect immune compromised ladies.


Cancer cervix is higher in smoker females.
Type 15 HPV virus is highly oncogenic for cancer cervix.
Surgical treatment can be applied for stage I or II. cancer cervix with pregnancy before fetal viability is an indication for
Upper segment C.S
Parametric infiltration is a common feature of stage I disease.
Malignancy starts at the transformation zone.
Lymphatic spread starts when the infiltration of the basement membrane is more then 5 mm in depth.
Radiotherapy can effectively kill cancer cells in the lymph nodes.
Colopscopy should detect the transformation zone completely to be satisfactory.
It is more common in those starting sexual activity after age of 40 years.
Abnormal cervical erosion or suspicious cervix are early signs of C.I.N
Adenosquamous carcinoma is the commonest pathological types
Most of the adenocarcinoma of the cervix arise fro the endocervical canal.
Cervical stump carcinoma spread late to the bladder.
Adenocarcinoma of the cervix has a much better prognosis.
Menorrhagia is the commonest type of bleeding with cancer cervix.
Routine removal of the para-aortic nodes is essential with Werthiems operation.
Intracaviatery irradiation is applied to treat cases cancer in the lymph nodes.
Avascular necrosis of the urter is common after Werthiems operation.
Cervical stump carcinoma is best to be treated surgically.
Pelvic infection is the major contraindication for radiotherapy.
Werthiem;s operation has a higher mortality rte compared by radiotherapy.
Shauta operation may be tried in stage III cancer cervix.
Pelvic exenteration is done for poor surgical risk cases.
Surgical staging of cancer cervix can determine the line and extent of treatment.
Cystoscopy should be done for all cases of diagnosed cancer cervix.
Recurrent cancer cervix is best treated surgically.
Small cell carcinoma is best treated by radiotherapy.
Combined surgery and radiotherapy gives the best results.

(11) * CERVIX ... NONE NEOPLASTIC LESIONS


(1)Which of the following statements is true:
A) Ectopy means ectropion.
B) Ectopy means eversion of the endocervix.
C) Ectopy means erosion of the cervix.
D) Ectopy means cervical ulcer.
E) All of the above.
(2)Contraceptive pills cause the following:
A) Glandular hyperplasia and hypersecretion.
B) Stromal oedema.
C) Increased stromal vascularity.
D) All of the above.
E) None of the above.

111

Gyna & Obstetrics Reveision by/Dr.adool

2008

(3)The increased risk of developing cancer cervix in pill users is due to the following causes EXCEPT:
A) Early sexual life.
B) Multiple partners.
C) Glandular hyperplasia.
D) Sexual overactivity.
E) More susceptibility to contract sex-transmitted HSV2, HPV.
(4)When a cervical polyp is removed, the most important part to be examined is:
A) Tip
B) Base
C) Middle part..
(5)A pedunculated small reddish mass protruding outside the endocervix, may be:
A) Cancer cervix.
B) Granulomatous mass.
C) Adenomatons polyp.
D) All of the above.
E) None of the above.
(6)In a 50-year old patient, polypectomy is done for metrorrhagia, Which of the following is true.
A) Fracional curettage is not needed.
B) Fractional curettage is advisable.
C) Fractional curettage is mandatory.
(7)In acute cervicitis, the following treatment-modalities are true EXCEPT:
A) Limitation of sexual activity.
B) Proper antibiotics.
C) Analgesics.
D) Cauterization.
(8)With chronic cervicitis, which of the following is observed:
A) Angry red cervix.
B) Mucopurulent discharge.
C) Cervical erosion.
D) Nabothian follicle.
E) All of the above.
(9)Chronic cervicitIs, may be associated with the following EXCEPT:
A) Leucorrhoea.
B) Low abdominal pain.
C) Backache.
D) Menorrhagia.
E) Contact bleeding.
(10)Cervical T.B. is caused by which strain of T.B:
A) Bovine Mycobacterium.
B) Human mycobacterium.

112

Gyna & Obstetrics Reveision by/Dr.adool

2008

C) Avian mycobacterium.
D) Mycobacterium murine.
E) Mycobacterium vole.
(11)In Cervical T.B., which of the following is NOT TRUE:
A) The cervix is ulcerated, distorted and mimics malignancy.
B) There is slight bleeding precipitated by contact.
C) It predisposes to cancer cervix.
D) Pap. Smear and cervical biopsy are mandatory.
E) Endometrial curettage should be done.
(12)Cervical chancre is associated with the following EXCEP:
A) 2nd site of chancre in the female genital tract after vulva.
B) Ulcer with indurated base and punched-out edge.
C) Erosion covered with grey membranous exudates.
D) Painful.
E) Post-coital bleeding.

(12) * Endometrial carcinoma


(1)The commonest female genital- tract-malignancy is:
A) Ovarian
B) Endometrial.
C) Cervical.
D) Vaginal.
(2)In cases of endometrial carcinoma, the mean age is around:
A) > 60 years.
B) 45 years.
C) 30 years.
D) (A + B).
E) (A + B + C)
(3)Endometrial carcinoma subjects are also susceptible to:
A) Cancer breast.
B) Cancer colon.
C) Cancer ovary.
D) All of the above.
(4)The following drugs have been implicated in the development of endometrial carcinoma EXCEPT:
A) Unopposed oestrogen stimulation.
B) Clomiphene citrae.
C) Tamoxifen.
D) Estrogen replacement therapy, unaccompanied with progesterone.

113

Gyna & Obstetrics Reveision by/Dr.adool

2008

(5)Lesions predisposing to endometrial carcinoma are:


A) Simple endometrial hyperplasia above the age of 50 years.
B) Complex endometrial hyperplasic.
C) Atypical endometrial hyperplasia.
D) Endometrial polyps in postmenopausal women.
E) All of the above.
(6)Lymphatic spread of endometrial carcinoma involves:
A) Lymphatic drainage of the ovary.
B) Lymphatic drainage of the cervix.
C) Lymphatic drainage of the vulva.
D) (A + B).
E) (A + B +C).
(7)Staging of endometrial carcinoma (FIGO 1988) is based on:
A) Clinical findings.
B) Surgical findings.
C) Patholgoical findings.
D) (A + B).
E) (B + C).
(8)Diagnosis of endometrial carcinoma relies upon:
A) Ultrasound examination.
B) Vaginal cytology.
C) Endometrial curettage (fractional).
D) Aspiration cytology.
(9)Endometrial carcinoma is dealt with by the following EXCEPT:
A) Total abdominal hysterectomy with bilateral salpingo-oophorectomy.
B) Radical (Wertheims) Hysterectomy.
C) High dose progestogens.
D) Endometrial ablation.
10. Uterine sarcoma:
A. May arise from the body or from the cervix.
B. The majority of cases occur in young females.
C. Cases present with painless bleeding.
D. D&C is the main method of diagnosis.
E. Treatment consists of hysterectomy followed by radiotherapy.
11. Peripheral conversion of androstenedione results in the formation of:
A. Estrone.
B. Estradiol.
C. Estriol.
D. Dehydroepiandrosterone.
E. Progesterone.

114

Gyna & Obstetrics Reveision by/Dr.adool

2008

12. Risk factors of endometrial hyperplasia include:


A. Multiparity.
B. Obesity.
C. Anovulation.
D. Estrogen secreting ovarian tumors.
E. Use of combined oral contraceptive pills.
13. Risk factors of endometrial carcinoma include:
A. Obesity.
B. Human papilloma virus (HPV) infection.
C. Anovulation.
D. Estrogen secreting ovarian tumors.
E. Multiparity.
14. Endometrial hyperplasia:
A. May predispose to carcinoma.
B. May be associated with ovarian cysts.
C. Diagnosis can be established through abdominal ultrasonography.
D. May be treated by progesterone.
E. May be treated by hysterectomy.
15. Endometrial carcinoma:
A. Is usually a squamous carcinoma.
B. Does not occur before the menopause.
C. Is more common in obese postmenopausal females.
D. May be accurately diagnosed by cytology.
E. Is preceded by endometrial hyperplasia.
16.Endometrial carcinoma:
A. Occurs at an age around 60 years.
B. Does not occur in young females.
C. Is more common in grand multipara than nullipara.
D. Is more common than carcinoma of the cervix.
E. Is related to use of oral contraceptive pills.
17. Endometrial adenocarcinoma:
A. Contains malignant glands and stroma.
B. Is derived from endometrial stroma.
C. May contain areas of benign squamous epithelium.
D. May contain areas of malignant squamous epithelium.
E. Is the commonest endometrial tumor.
18. Stage I endometrial carcinoma:
A. Is confined to the uterus.
B. May be associated with lymphatic spread.
C. Is best managed by vaginal hysterectomy.
D. The usual presentation is postmenopausal bleeding.
E. All cases should receive postoperative radiotherapy.

115

Gyna & Obstetrics Reveision by/Dr.adool

2008

19. Therapeutic uses of progesterone may include:


A. Endometriosis.
B. Induction of ovulation.
C. Contraception.
D. Dysfunctional uterine bleeding.
E. To postpone menstruation.
20- Endometrial Hyperplasia.
A)
is a pre-malignant condition in 50% of cases
B) is a common histological finding in patients on the combined oral contraceptive
C) is indication for hysterectomy and removal of ovaries
D) Progestagens tend to reduce hyperplasia so, used to treat it
21- A postmenopausal woman presenting with bleeding per vaginum has a mass in the fornix measuring 3X3 cms.
Which of the following can be considered in the diagnosis?
A) Carcinoma endometrium of uterine body
B) Tuboovarian mass
C) Ovarian theca tumors
D) Dermoid tumor of ovary
E) Subserous fibroid .
22- A 50 year old woman, nulliparous, diabetic and obese presenting with post-menopausal bleeding can be any of
the following except :
A) Carcinoma in situ of cervix
B) Carcinoma endometrium
C) DUB
D) None of the above
23- Which is not true regarding carcinoma of the endometrium?
A) Can involve the inguinal lymph nodes.
B) Is associated with recurrences in the vaginal vault after hysterectomy.
C) If metastatic, will not respond to progestagenic therapy
D) Can spread via the blood system to bone and lungs.
24- A 45-year-old woman presents with intermenstrual spotting. Physical examination is remarkable for obesity. Her
Pap test is normal. Findings on office endometiral biopsy revealed cystic endometrial hyperplasia. Complete
blood count is within normal limits. The most appropriate next step is:
A) 3-6 month course of daily progestins
B) Hysteroscopy
C) Dilation and curettage (D&C)
D) Endometiral ablation
E) Hysterectomy
25- A 48-year-old woman presents with menometrorrhagia. The uterus is enlarged, and your clinical diagnosis is
bleeding secondary to uterine leiomyomas. At the time of hysterectomy, you notice a lateral bulge in the uterus
and wormlike structures in the vessels of the adjacent broad ligament. The pathologist tells you that on opening
the uterus, there is an ill-defined, predominantly intra mural mass not far from these structures .The most likely
diagnosis is:
A) endometrial stromal nodule
B) leiomyoma with degenerative changes
C) leiomyosarcoma

116

Gyna & Obstetrics Reveision by/Dr.adool

2008

D) low-grade endometrial stromal sarcoma


E) mixed mullerian tumor
26- Endometrial carcinoma, all are false EXCEPT:
A) Can occur when equine estrogen is given for a long time
B) Can present as infertility
C) Surgery is always indicated
D) Not always associated with bleeding PV
E) More common in multiparous women
27- With regard to endometrial carcinoma , all are true EXCEPT:
A) simple hyperplasia carries little malignant potential
B) complex hyperplasia carries < 5% risk of progressing to malignancy
C) atypical hyperplasia carries about a 25% risk of progressing to malignancy
D) the risk increases with increasing parity
28- With regard to endometrial carcinoma , all are true EXCEPT:
A) extension to the cervix worsens the prognosis
B) the combined oral contraceptive reduces the risk of the disease
C) the risk increases with early menarche/ late menopause
D) polycystic ovary syndrome reduces the risk
E) obesity and hypertension are relative risk factors
29- With regard to endometrial carcinoma. , all are true EXCEPT:
A) diabetes is a risk factor
B) has been described in association with granulosa and theca cell tumours
C) secondary spread may be to the inguinal lymphatics
D) treatment of early disease is best done by Wertheim's hysterectomy
30- Endometrial Cancer, all are true EXCEPT:
A) is associated with Human Papilloma virus type 16
B) has poorer prognosis if extended to the cervix
C) has better prognosis if well differentiated.
D) usually presents with post menopausal bleeding
E) is more common with estrogen-only HRT
31- Endometrial Carcinoma with vaginal metastasis is staged as:
A) II b
B) III a
C) III b
D) III c
E) IV a
32- Radiotherapy is indicated in Endometrial Carcinoma in following situations EXCEPT:
A) Preoperative radiotherapy followed by TAH + BSO
B) Postoperative adjuvant radiotherapy after TAH + BSO
C) Primary treatment for medically inoperable
D) To specifically treat pelvic side wall involvement
E) Vaginal recurrence after hysterectomy

117

Gyna & Obstetrics Reveision by/Dr.adool

2008

33- Incidence of all of the following conditions are increased in tamoxifen users EXCEPT
A) Cervial dysplasia
B) Endometrial polyps
C) Endometrial hyperplasia
D) Uterine fibroids
E) Endometrial adenocarcinoma
34- Women with postmenopausal bleeding need endometrial sampling if the endometrium on TVS / USG is thicker
than:
A) 1mm
B) 2mm
C) 5mm
D) 8mm
E) 10mm
35- All of the following are known risk factors for development of endometrial carcinoma except:
A) Obesity.
B) Family History.
C) Use of Hormone Replacement Therapy
D) Early Menopause.
36- The peak age of presentation of endometrial carcinoma is:
A) 5-6 years
B) 10-20 years
C) 20-30 years
D) 50-60 years
E) 35-45 years 5 years
37- The times scale within which 80% of patients with recurrent disease will present following the treatment of a
Stage I tumour:
A) 5-6 years
B) 10-12 years
C) 2-3 years
D) 5 years
E) 2-3 months
38- Within this period, up to 80% of patients will be alive following radiotherapy for Stage I cancer of the
endometrium:
A) 5-6 years
B) 10-12 years
C) 5 years
D) 7 years
E) 2-3 months
39- The percentage of women who commonly present with both cervical and endometrial carcinoma:
A) <5%
B) <2%
C) 10-20%
D) 5-10%

118

Gyna & Obstetrics Reveision by/Dr.adool

2008

40- The percentage of women with stage IV cancer of the endometrium who will be alive after 5 years following
surgical exterentation:
A) <5%
B) 10-20%
C) <2%
D) 5-10%
41- Pyometra is a complication associated with all of the following conditions except:
A) Carcinoma of the vulva.
B) Carcinoma of the cervix.
C) Carcinoma of endometrium.
D) Pelvic radiotherapy.
42- Carcinoma of the uterine body, all are true EXCEPT:
A) The incidence is increasing
B) The prognosis for patients who develop endometrial cancer while on exogenous estrogen therapy is poor
C) 40% of cases of severe atypical hyperplasia will progress to an invasive carcinoma
D) Adenosquamous carcinoma has a poorer prognosis than a pure adenocarcinoma
43- Management of endometrial cancer, all are true EXCEPT:
A) stage II endometrial carcinoma should be managed by Wertheim hysterectomy
B) Peritoneal cytology is of no importance in this disease
C) Assessment must include fractional curettage
D) 5-10% of patients with a clinical stage I tumour have occult metastases in the ovary
E) The depth of myometrial invasion is associated with the grade of tumour
44- Cystic glandular hyperplasia of the endometrium, all are not true EXCEPT:
A) Progresses to endometrial cancer in 70% of cases
B) Crowding of glands is typically seen
C) There is no strormal proliferation
D) May be seen in dysfunctional uterine bleeding
45- In stage II carcinoma of the endometrium, all are false EXCEPT:
A) the length of the uterine cavity is less than 8 cm
B) the corpus and the cervix are involved
C) there is extension to involve the parametrium
D) there is extension to the ovaries
E) there is metastasis to the bladder
46- The primary mode of treatment for endometrial carcinoma confined to the uterine corpus is:
A) external beam radiation
B) intracavitary radium
C) hysterectomy
D) chemotherapy
E) progestin therapy
47- Fractional dilatation and curettage reveals endometrial carcinoma involving the cervix. This finding is:
A) of no prognostic significance
B) of some prognostic significance but does not require change in management
C) significant only if the cervical tumor is clinically obvious
D) significant even if the disease is present only microscopically
E) a contraindication for hysterectomy

119

Gyna & Obstetrics Reveision by/Dr.adool

2008

48- Women who have endometrial carcinoma most frequently present with which of the following symptoms?
A) Bloating
B) Weightless
C) Postmenopausal bleeding
D) Vaginal discharge
E) Hemoptysis
49- Young women whose mothers took diethylslilbesirol (DES) during pregnancy are most likely to develop which of
the following vaginal carcinomas?
A) Papillary adenocarcinoma
B) Squamous carcinoma
C) Carcinoma of the infantile vagina
D) Adenosquamous carcinoma
E) Clear cell adenocarcinoma
50- The class ofchemotherapeutic agents that is most effective in the management of women who have recurrent
endometrial carcinoma is:
A) antimelabolites
B) hormones
C) alkylating agents
D) Vinca alkaloids
E) antibiotics
51- Adenocarcinoma of the endometrium can be described by which of the following statements?
A) It is primarily a disease of postmenopausal women
B) The average age of affccied women is 10 years more than the average age of women who have cervical carcinoma
C) It has a more favorable prognosis than cervical cancer, with the 5 year survival rate approaching 75 percent
D) It is increasing in frequency relative to carcinoma of the cervix
E) All of the above
52- The spread of adenocarcinoma of the body of the uterus can be described by which of the following statements?
A) Distant organs, such as the liver are frequently involved
B) Dissemination is chiefly by way of the lymphatics
C) The tumor resembles cervical carcinoma in its frequency of disseminaiion
D) Direct extension is an important route ofdissemination
53- Women who have which of the following characteristics are at high risk for endometrial carcinoma?
A) Hypertension
B) Diabetes
C) Obesity
D) Familial history of endometrial carcinoma
E) All of the above
54- Evidence in evaluating cell types found in carcinoma of the endometrium suggests that:
A) poorly differentiated adenocarcinomas have a poor prognosis
B) adenoacanthomas have a poor prognosis
C) adenosquamous carcinomas have a good prognosis
D) adenosquamous tumors occur more often in young women

120

Gyna & Obstetrics Reveision by/Dr.adool

2008

55- For the management of women who have endometrial carcinoma, intracavitary radium has been employed
routinely
A) as a treatment for women who have ovarian metastases
B) as a treatment for women who have vaginal apical recurrences
C) as a treatment for women who have metastases of the pelvic sidewall
D) as primary therapy for operable patients
56- A 60 years old presented to a Gynecologist with an attack of post-menopausal bleeding for the 2nd time, the bleeding
attack was a mild one, she gave a history of liver problems, she is hypertensive 160/100 with echymotic patches. On pelvic
examination the uterus is symmetrically enlarged 8 weeks size of pregnancy.
57.What investigations should be done to this lady?
a) Blood sugar.
b) Abdominal ultrasound.
c) Liver functions.
d) Coagulations profile.
e) All of the above.
58.On doing vagianl ultrasound the endometrial thickness was found to be 8mm, give your recommendations about further
management.
a) Fractional curettage.
b) Hysterosalpingography.
c) Endometrial biopsy by Novak curette.
d) All of the above.

Say true or false:


a.
b.
c.
d.
e.
f.
g.
h.
i.
j.
k.
l.
m.
n.
o.
p.
q.
r.
s.
t.
u.

Endometrial carcinoma affects malnourished ladies.


Those with liver disease are more affected.
Young age females around the age of 40 are mostly affected.
Those of P.C.O. and chronic anovulation are more affected.
Carcinoma occurring in those on Estrogen replacement therapy is of better prognosis.
Combined surgery and radiotherapy is the most effective line of treatment.
Lymphatic spread is common.
Uremia is the main cause of death.
Fractional curettage is the main line of diagnosis.
Fractional curettage can accurately detect every case.
Menorrhagia is the main presenting symptom.
Squamous cell carcinoma has a very good prognosis.
Breast cancer may be associated with some cases of endometrial carcinoma.
Cigarette smoking offer protection against endometrial carcinoma.
Local spread and myometrial infiltration is late and rare.
Endometrial carcinoma has a much better prognosis than cancer cervix.
Those arising from the anterior wall of the uterus have a much better prognosis.
Stage Ia grade 1 tumors can be treated with surgery alone.
Vaginal vault is the commonest site of recurrence.
Screening programs for endometrial carcinoma are highly effective one.
Hysteroscopic guided biopsy increase the sensitivity of fractional curettage for detection of localized type of endometrial
carcinoma.
v. Simple endometrial hyperplasia has a high risk for malignant transformation.

121

Gyna & Obstetrics Reveision by/Dr.adool

2008

w.
x.
y.
z.
aa.
bb.
cc.

Progesterone can revert most of cases of endometrial hyperplasia.


Endometrial carcinoma may spread to the inguinal lymph nodes.
The isthmus has the same lymphatic drainage as the cervix.
Depot provera can treat advanced or recurrent cases of endometrial carcinoma.
Addition of progesterone to E.R.T is a good protection.
Radiotherapy alone has good results.
Vaginal hysterectomy is never tried.

(13) * Choriocarcinoma
1. Choriocarcinoma:
A. Usually follows an event of pregnancy.
B. A friable hemorrhagic polypoidal mass projects into the uterine cavity.
C. Shows early lymphatic spread.
D. The main presentation is bleeding.
E. The main method of investigation is ultrasonography.
2. Choriocarcinoma:
A. Is a malignant tumor of the trophoblast.
B. HCG always shows higher values than with vesicular mole.
C. Chest X ray is an essential investigative tool.
D. Cases following full term pregnancy are associated with poor prognosis.
E. Treatment always involves methotrexate.
3- Characteristic features of choriocarcinoma include which of the following:
A) Haematuria
B) Gastritis
C) Haemoptysis and vaginal bleeding
D) Haematemesis
4- All of the following are features of good-prognosis metastatic gestational trophoblastic disease except:
A) Serum beta-hCG of less than 40,000 mIU/mL.
B) Disease duration of less than four months.
C) Gestational trophoblastic disease following a normal term pregnancy.
D) No metastases to the brain or liver.
E) All of the above are features of good-prognosis gestational trophoblastic disease.
5- The treatment of choice for choriocarcinoma Is:
A) Methotrexate ,
B) Radical hysterectomy
C) Radium
D) External radiotherapy.
6- Exfoliative cytology, all are not true EXCEPT:
A) Involves the study of wax-embedded pieces of tissues.
B) Is performed on cells aspirated through a fine needle.
C) Is used to screen for Carcinoma of the uterine cervix.
D) May be used to diagnose carcinoma of the ovary.
E) Is often used in the diagnosis of breast lesions.

122

Gyna & Obstetrics Reveision by/Dr.adool

2008

7- Which of the following side effects is least common with cisplatin?


A) Thrombocytopenia
B) Hypomagnesimia
C) Cardiac toxicity
D) Neuropathy
E) High frequency hearing loss
8- Which is not an indication for chemotherapeutic treatment of postmolar trophoblastic disease?
A) Abnormal hCG regression curve
B) Presence of metastasis
C) Uterine infection
D) Uterine haemorrhage
E) Tissue diagnosis of choriocarcinoma
9- A case of Gestational Trophoblastic Neoplasia belongs to high risk group if disease develop after:
A) Hydatidiform mole.
B) Full term pregancy.
C) Spontaneous abortion.
D) Ectopic Pregancy.
10- Invasive molar tissue is most commonly found in:
A) Myometrium
B) Vaginal wall
C) Ovary
D) Liver
E) Lungs
11- Criteria that are especially reliable in determining a patient's response to chemotherapy for gestational
trophoblastic disease include all EXCEPT:
A) duration of the disease
B) urine levels of HCG
C) sites of metastases
D) the patient's age
12- The role of aclinomycin D and methotrexate in the management of gestational trophoblastic disease include all
EXCEPT:
A) resistance to one agcnl results in cross-resistance to the other
B) actinomycin D is safer than methoirexate for women whose liver function is impaired
C) there is additive effect in combining the two agents
D) actinomycin D is as effective as methotrexate for primary treatment
13- Disorders referred to as gestational trophoblastic disease include all EXCEPT:
A) hydatidiform mole
B)chorioadenoma destruens
C) choriocarcinoma
D) ovarian choriocarcinoma
14- Determination of the presence of a growing tumor in a patient who has gestational trophobtastic disease could be
established by which of the following:
A) serial chest x-rays
B) evaluation of menstrual function

123

Gyna & Obstetrics Reveision by/Dr.adool

2008

C) presence of lactation
D) serial HCG titers
15- The most common sarcoma of the uterus is:
A) endometrial stromal sarcoma
B) carcinosarcoma
C) leiomyosarcoma
D) mixed mesodermal sarcoma
E) rhabdomyosarcoma
16- Mixed mesodermal tumors of the uterus can be described by which of the following statements?
A) They are more common than previously assumed and constitute about 17 to 18 percent of uterine malignancies
B) Microscopic examination may reveal the presence of bone, cartilage, muscle, or other elements
C) Their development has been associated with maternal intake of estrogen during pregnancy
D) Five-year survival rate is quite good (approximately 80 percent)
E) They are not known to occur before the menopausal years
17- The neoplasm most sensitive to appropriate chemotherapy is:
A) gestational irophoblastic disease
B) ovarian dysgerminoma
C) Burkitt's lymphyma
D) endomelrial carcinoma
E) ovarian serous carcinoma

say true or false:


a.
b.
c.
d.
e.
f.
g.
h.
i.
j.
k.

124

It must present with bleeding.


Occurs on top of a previous pregnancy event.
Main line of treatment is hysterectomy.
Enlarged ovaries may occur in 90% of cases.
Irridation is indicated for cases with lung and brain secondaries.
High risk cases are treated with methotrexate only.
Curettage alone can diagnose all cases.
It gives cannon ball secondaries in the lung.
Vaginal secondaries are very common.
Histologically characterized by malignant cyto and syncito tropoblasts.
Lymphatic spread is the main route of spread.

Gyna & Obstetrics Reveision by/Dr.adool

2008

(14) * Ovarian tumours


1.

Ovarian tumors characterized by the following, except:


a) They occur throughout womens age.
b) Early detection is very difficult.
c) Usually asymptomatic.
d) Usually functioning.

2.

The following are germ cell tumors of the ovary, except:


a) Dysgerminoma.
b) Endodermal sinus tumor.
c) Benign cystic teratoma.
d) Granulosa cell tumor.

3.

The followings are functional ovarian tumour, except:


e) Theca cell tumour.
f)

Strauma ovarii.

g) Granulosa cell tumour.


h) Krukenberg tumour.
4.

5.

Benign papillary serous cell tumours have these criteria except :


a) Commonly bilateral
b) Rare to turn malignancy
c) Common type of ovarian tumour
d) Not functioning
The most important phase of treatment of all ovarian cancer is :
a) Surgery
b) Radiotherapy
c) Chemotherapy
d) Palliative treatment

6. Non-neoplastic cysts of the ovary include:


A. Follicular cysts.
B. Simple serous cysts.
C. Endometriotic cysts.
D. Dermoid cysts.
E. Theca lutein cysts.
A. True B. False C. True D. False ,E. True
7.
Follicular
cysts:
A. Result from cystic changes of the corpus luteum.
B. Are multilocular.

125

Gyna & Obstetrics Reveision by/Dr.adool

2008

C. Are usually of small size.


D. When complicated, may require surgical intervention.
E. Cannot be detected by ultrasonography.
A. False B. False C. True D. True E. False
8. Par ovarian cysts:
A. Arise from the surface epithelium of the ovary.
B. Arise from remnants of the wollfian duct.
C. Are usually bilateral.
D. Are usually multilocular.
E. Have a thin translucent wall.
A. False B. True C. False D. False E. True
9. Ovarian tumors may arise from:
A. Surface epithelium of the ovary.
B. Peritoneal covering of the ovary.
C. Granulosa cells.
D. Remnants of the wollfian ducts.
E. The ovum.
A. True B. False C. True D. False E. True
10. Epithelial ovarian tumors include:
A. Mucinous cystadenoma.
B. Mucinous cystadenocarcinoma.
C. Dysgerminoma.
D. Choriocarcinoma.
E. Brenner's tumor.
A. True B. True C. False D. False E. True
10. Ovarian epithelial tumors:
A. Are the most common ovarian tumors.
B. Arise from the ovarian stroma.
C. Serous cystadenoma are the largest.
D. Have a good prognosis if diagnosed at stage I.
E. May cause intestinal obstruction.
A. True B. False C. False D. True E. True
11. Carcinoma of the ovary:
A. Are the most common malignant tumor of the female genital tract.
B. Is classified as stage II if it has spread to the pelvic peritoneum.
C. Can be detected by tumor markers.
D. The most important route of spread is by lymphatics.
E. Occurs only in old females.
A. False B. False C. True D. False E. False

126

Gyna & Obstetrics Reveision by/Dr.adool

2008

12. Mucin secreting tumors of the ovary:


A. Can reach huge sizes.
B. Are usually bilateral.
C. Are usually malignant.
D. Are usually multilocular.
E. Can be seen by ultrasound.
A. True B. False C. False D. True E. True
13. Granulosa cell tumors:
A. Usually present early.
B. May occur at any age.
C. Are usually bilateral.
D. Produce estrogen hormone.
E. Are chemosensitive.
A. True B. True C. False D. True E. False
14. Ovarian fibromas:
A. Commonly become malignant.
B. May be associated with hydrothorax.
C. Have long pedicles.
D. Are frequently bilateral.
E. May contain theca cells.
A. False B. True C. True D. False E. True
15. Meig's syndrome includes:
A. Hydrothorax.
B. Ascites.
C. Fibroid.
D. Endometriosis.
E. Ovarian fibroma.
A. True B. True C. False D. False E. True
16. Germ cell tumors include:
A. Dysgerminoma.
B. Gonadoblastoma.
C. Thecoma.
D. Krukenberg's tumor.
E. Embryonal carcinoma.
A. True B. False C. False D. False E. True
17. Germ cell tumors include:
A. Choriocarcinoma.
B. Granulosa cell tumor.
C. Endodermal sinus tumor.
D. Benign cystic teratoma.
E. Brenner's tumor.
A. True B. False C. True D. True E. False

127

Gyna & Obstetrics Reveision by/Dr.adool

2008

18. Dermoid cysts:


A. Are commonly malignant.
B. May produce hormones.
C. Are germ cell tumors.
D. Occur more in older females.
E. May contain teeth.
A. False B. True C. True D. False E. True
19. Dermoid cysts:
A. Are usually bilateral.
B. Plain X-ray may help in diagnosis.
C. May cause septic peritonitis if ruptured.
D. Usually have a long pedicle.
E. Are lined by skin.
A. False B. True C. False D. True E. True
20. Dysgerminomas:
A. Are more common in younger females than in older females.
B. Are more common in dysgenetic gonads.
C. Are bilateral in 60% of cases.
D. Commonly present with pain.
E. Are radiosensitive.
A. True B. True C. False D. False E. True
21. Endodermal sinus tumors are characterized by:
A. They occur at a young age.
B. They secrete alpha fetoprotein.
C. They secrete human chorionic gonadotropin.
D. They may present by pain.
E. They are radiosensitive.
A. True B. True C. False D. True E. False
22. Metastasis in the ovaries:
A. Are collectively known as "Krukenberg tumors".
B. Account for 20% of all ovarian tumors.
C. Are bilateral in 50% of cases.
D. Are always apparent on the ovarian surface.
E. Are almost always solid.
A. False B. True C. True D. False E. True
23. The following substances may be secreted by ovarian tumors:
A. Thyroxin.
B. LH.
C. HCG.
D. FSH.
E. Serotonin.
A. True B. False C. True D. False E. True

128

Gyna & Obstetrics Reveision by/Dr.adool

2008

24. The following ovarian tumors are always malignant:


A. Dysgerminoma.
B. Endodermal sinus tumor.
C. Brenner tumor.
D. Solid teratoma.
E. Thecoma.
A. True B. True C. False D. False E. False
25. Hormone secreting ovarian tumors include:
A. Granulosa cell tumor.
B. Sertoli cell tumor.
C. Fibromas.
D. Simple serous cyst.
E. Benign cystic teratoma.
A. True B. True C. False D. False E. True
26. Risk factors of ovarian malignancy include:
A. Multiparity.
B. Family history of ovarian malignancy.
C. Old age.
D. Use of combined oral contraceptive pills.
E. Exposure to pelvic irradiation.
A. False B. True C. True D. False E. True
27. Spread of malignant epithelial ovarian tumors:
A. Blood spread occurs early.
B. Lymphatic spread occurs early.
C. Spread to para aortic lymph nodes puts the case at stage III.
D. Spread to appendix is common.
E. Spread to omentum is common.
A. False B. False C. True D. True E. True
28. Criteria suggesting malignancy in ovarian tumors include:
A. Solid areas in the tumor.
B. Bilateral tumors.
C. Bilateral edema of the lower limbs.
D. Old age.
E. Amenorrhea.
A. True B. True C. False D. True E. False
29. Radiosensitive pelvic tumors include:
A. Adenocarcinoma of the cervix.
B. Adenocarcinoma of the endometrium.
C. Squamous cell carcinoma of the cervix.
D. Squamous cell carcinoma of the vulva.
E. Dysgerminoma.
A. True B. True C. False D. False E. True

129

Gyna & Obstetrics Reveision by/Dr.adool

2008

30. Complications of benign ovarian tumors include:


A. Torsion.
B. Hemorrhage.
C. Infection.
D. Malignant transformation.
E. Hyaline degeneration.
A. True B. True C. True D. True E. False
31. Management of benign ovarian tumors:
A. Young patients, small cyst: ovarian cystectomy.
B. Young patients, large cyst: ovariotomy.
C. Older patients: bilateral ovariotomy.
D. Complicated cyst: ovariotomy.
E. Suspicion of malignancy: radical hysterectomy.
A. True B. True C. False D. True E. False
32. In the management of malignant ovarian tumors:
A. Extensive surgery is of very limited value in advanced stages.
B. Chemotherapy is more effective than with other epithelial tumors.
C. Hormonal therapy has almost no value in treatment.
D. Dysgerminoma is effectively treated by radiotherapy.
E. Second look laparotomy is required in all cases.
A. False B. True C. True D. True E. False
33. A second look laparotomy for an ovarian tumor is indicated when there is:
A. An incomplete resection of the primary tumor.
B. Clinically evident residual disease.
C. A favorable response to chemotherapy.
D. An original well differentiated stage I carcinoma.
E. Positive laparoscopic evidence of residual disease.
A. False B. False C. True D. False E. True
34. Prognostic factors in malignant ovarian tumors include:
A. Stage.
B. Chemo sensitivity.
C. Radio sensitivity.
D. Type of growth.
E. Efficiency of treatment.
A. True B. True C. True D. True E. True
35. Early blood spread is characteristic of:
A. Cervical carcinoma.
B. Endometrial carcinoma.
C. Choriocarcinoma.
D. Malignant ovarian tumors.
E. Carcinoma of the vulva.
A. False B. False C. True D. False E. False

130

Gyna & Obstetrics Reveision by/Dr.adool

2008

36- In a 55-year-old woman with stage II ovarian serous cystadenocarcinoma, the disease is most likely to recur in the:
A) Peritoneum
B) brain
C) lungs
D) skeleton
E) bone marrow
37- Primary malignant ovarian tumors include all of the following EXCEPT:
A) Krukenburgs tumor
B) Arrenoblastoma
C) Granulosa cell tumor
D) Endodermal sinus tumor
38- Alpha-fetoprotein marker is most commonly associated with which ovarian tumor:
A) Epithelial cell ovarian tumor
B) Endodermal sinus tumor
C) Dysgerminoma
D) Mature teratoma
39- During an elective Cesarean section at 39 weeks on a previously healthy 21 year G1P0 with breech
malpresentation, a 3 cm ovarian mass is noted on the left ovary with a very small streak-like right ovary. A biopsy
is taken and sent to pathology. In this patient, which of the following investigations would confirm the most
likely diagnosis?
A) AFP
B) chromosomal studies
C) beta HCG
D) estrogen
E) CA-125
40- Alpha fetprotein (AFP) is most often useful for monitoring the response to treatment of which germ cell tumor:
A) Dysgerminoma
B) Immature teratoma
C) Endodermal sinus tumor
D) Choriocarcinoma
41- Granulosa cell tumors are associated with all EXCEPT:
A) Isosexual precocious puberty
B) Endometrial hyperplasia & adenocarcinoma
C) Present most commonly before the age of 5
D) Vaginal bleeding
42- Combination chemotherapy regimen in Cancer ovary, all are not true EXCEPT:
A) Is the treatment of choice for most patients with residual disease following surgery
B) Less toxic than single agent therapy
C) Less expensive than single agent therapy
D) Produces better results if it is a combination containing cyclophosphamide
E) None of the above
43- Laporatomy performed in a case of ovarian tumor revealed unilateral ovarian tumor with ascities positive for
malignatn cells and positive pelvic lymph nodes. All other structures were free of disease. what is stage of the
disease:
A) Stage IIc. or Stage IIIc.

131

Gyna & Obstetrics Reveision by/Dr.adool

2008

B) Stage IIIa.
C) Stage IIIb
D) Stage IV
44- Which of the following is most reliable in diagnosing ovarian malignancy?
A) Medical history
B) Plain Xray
C) Physical examination
D) Ultrasonography
45- Which of the following is not associated with an increased risk for ovarian carcinoma ?
A) Nulliparity
B) Radiation exposure
C) A diet low in fibre and Vit A.
D) Familial tendency
E) Ovulation induction
46- Approximately 75 - 80% of ovarian tumors originate from:
A) Epithelium
B) Stroma
C) Germ cells
D) Mesoderm
E) Unclassified tissue
47- A 16 year old woman presents with a sudden onset of severe right iliac fossa pain. On vaginal ultrasound
examination a 6cm diameter echogenic cystic mass is seen in the region of the right adnexum. What is the single
most likely diagnosis?
A) Pelvic inflammatory disease
B) Renal colic
C) Septic abortion
D) Threatened miscarriage
E) Torsion of ovarian cyst
48- 30 year old woman is concern about her health and her chances of ovarian cancer.Her mother had ovarian cancer
in past and was operated.What is your advice?
A) annual pelvic examination
B) annual CA-125 +/- USG
C) annual CT scan
D) annual USG +pelvic examination
E) annual PAP smear only
49- A 26 year old nullipara presents with smooth , pedunculated ovarian mass anterior to the uterus. What is the
most likely diagnosis?
A) luteoma
B) cystic teratoma
C) theca lutein cyst
D) serous cystadenoma
E) brenner's tumor

132

Gyna & Obstetrics Reveision by/Dr.adool

2008

50- A 60-year-old woman is referred for annual gynecologic examination. She has no complaints. Her examination is
normal except for a mobile cystic mass in the left adnexa. The patient has no family history of cancer.
Ultrasonography revealed a simple, anechoic left ovarian cyst measuring 2.25 x 4.56 cm. In follow-up
ultrasonography in 4 months, the cyst is slightly smaller. The most appropriate management is:
A)continued observation with examinations and ultrasonography
B) cyst aspiration with ultrasound guidance and fluid cytologic evaluation
C) computerized tomography of abdomen and pelvis
D) laparoscopy and ovarian cystectomy
E) total abdominal hysterectomy and bilateral salpingo-oophorectomy
51- The following are not characteristics of a dermoid cyst EXCEPT:
A) They are most often bilateral
B) The commonest ovarian neoplasm to be found in pregnancy
C) Lies in the broad ligament
D) Psammona bodies
E) Pseudomyxoma peritonei
52- A unilateral ovariotomy for a presumed endometriotic cyst has been performed on a 38-year-old woman. At
operation, the ovarian mass measured 12 cm in diameter and there were a few filmy adhesions between tumour
and large bowel. The opposite ovary appeared normal. There was no ascites or spillage of the tumour. On the
seventh postoperative day the pathological report has suggested adenocarcinoma. The capsule appears intact
and there was no serosal involvement:
A) She has a stage Ia carcinoma
B) She can be treated conservatively
C) A laparoscopy should be performed to check for residual disease
D) Omental metastases may be present
53- The following are not true statements about ovarian cancer EXCEPT:
A) The prognosis is uniformly poor if large masses remain in the abdomen following surgery
B) A good response to chemotherapy rarely occurs in advanced disease
C) There are no advantages in using intraperitoneal chemotherapy
D) Radiotherapy has an established role in stage III disease
54- In the evaluation of ovarian cancer, all are false EXCEPT:
A) About 75% are of germinal epithelial origin
B) The tumour marker CA 125 is only associated with mucinous tumours
C) The histological type of the tumour is more important than the clinical extent of the tumour in determining
the prognosis
D) Lower abdominal pain is the commonest initial symptom
55- The FIGO classification of ovarian cancer, all are false EXCEPT:
A) Stage I is subdivided according to differentiation of the tumour
B) Ascites is not present in stage I disease
C) Stage IV disease is present if there are positive retroperitoneal lymph nodes
D) In stage III disease there may be seedlings on the surface of the liver
56- A second look' operation for ovarian cancer:
A) Applies to re-exploration of the abdomen because of failure to perform definitive surgery at the first
operation
B) Peritoneal washings should be taken
C) Laparoscopy is preferable to laparotomy

133

Gyna & Obstetrics Reveision by/Dr.adool

2008

D) If there is no evidence of disease at operation then there is a greater than 95% chance of 5-year survival
57- The following are not true concerning chemotherapy for ovarian carcinoma EXCEPT:
A) Permanent sterility is inevitable
B) There is a significant incidence of chronic lymphatic leukemia
C) Therapy should be interrupted if the neutrophil count falls to below 4000 neutrophils/mm3
D) Cyclophosphamide may be administered intraperitoneally
E) Cardiomyopathy is a complication
58- Germ cell tumours, all are true EXCEPT:
A) Are all malignant
B) Progress in patients with endodermal sinus tumours can be monitored using alpha-fetoprotein (AFP)
C) Etoposide is a useful agent for anaplastic tumours
D) Lactate denydrogenase (LDH) may be raised in some of these tumours
59- Metastatic tumours of the ovary, all are true EXCEPT:
A) Accounts for 10% of all malignant tumours
B) Many of the patients with Krukenberg tumours are less than 40 years
C) Krukenberg tumours are usually unilateral
D) Commonly comes from a targe bowel primary
60- Granulosa cell tumours of the ovary, all are not true EXCEPT:
A) Found in association with endometrial cancer in 50 % of cases
B) Usually highly malignant
C) Can cause hirsutism
D) Highly sensitive to radiotherapy
61- Dysgerminoma of the ovary, all are false EXCEPT:
A) Unilateral salpingo-oophorectomy is often sufficient surgery
B) Tends to occur in women over 30
C) Associated with 45, X gonadal dysgenesis
D) Is unresponsive to chemotherapy
62- The Brenner tumour of the ovary, all are false EXCEPT:
A) Is not hormonally active
B) Is classified as a germ cell tumour
C) Usually measures 6-10 cm in diameter
D) Contains nests of epithelial cells
63- Serous cystadenoma, all are true EXCEPT:
A) More frequently encountered than the mucinous cystadenomas
B) Usually multilocular
C) Commonly bilateral
D) commonly found to have papillary excrescences on the inner surface of the cyst wall
64- Pseudomyxoma pertonei ,all are true EXCEPT:
A) Patient usually dies of malignant cachexia
B) Chemotherapy can prevent re-accumulation of the mucoid material
C) Is impossible to completely remove at operation
D) Loculation in the peritoneal cavity often occurs

134

Gyna & Obstetrics Reveision by/Dr.adool

2008

65- The Sertoli-Leydig tumour, all are true EXCEPT:


A) Does not causes amenorrhoea
B) Is an androblastoma
C) Usually cured by surgery
D) Also known as an arrhenoblastoma
66- Endometroid carcinoma of the ovary, all are true EXCEPT:
A) Most are well differentiated
B) Often co-exists with an adenocarcinoma of the endometrium
C) Is an uncommon ovarian tumour
D) Is associated with endometriosis
E) Squamous metaplasia is often seen
67- Ovarian tumours of borderline malignancy , all are not true EXCEPT:
A) Stromal invasion must be present
B) 15% behave in a malignant fashion
C) The 20-year survival rate is in excess of 90%
D) Usually occur in the elderly
68- The struma ovarii, all are true EXCEPT:
A) A teratoma
B) Usually malignant
C) May metastasise to lungs and bone
D) Found in association with a carcinoid tumour of the ovary
*- Say true or false
a. Torsion of ovarian tumors is most common during puerperium.
b. Fibroma of the ovary represents 20% of ovarian tumors.
c. Papillary serous cystadenoma is inherited as autosomal dominant triat.
d. Germ cell tumors are commonest at the age of 50 years.
e. Sex cord stromal tumors are hormonally active.
f. Mucinous cystadenoma of the ovary is the largest known ovarian tumors.
g. Mucinous cystadenoma of the ovary is the one of commonest benign ovarian tumors.
h. Border line ovarian tumors are more commonly bilateral than benign tumors.
i. Germ cell tumors are common in old age patients.
j. Granulosa cell tumor can occur at any age group.
k. Endodermal sinus tumor may present with acute abdomen
l. Mixed germ cell tumors is the commonest form of germ cell malignant tumors.
m. Dysgerminoma occurs in females with abnormal autosomes.
n. Examination of large number of section is essential before deciding border line malignant tumor of the ovary.
o. Mucinous tumors can attain a huge size
p. Burners tumor represents 30% of ovarian tumors.
q. Serous cystadenocarcinoma is the commonest 1ry ovarian malignancy.
r. Follicular cyst of the ovary occurs as a result of weak corpus luteum.
s. Secondary ovarian tumors are almost always bilateral.
t. Ovarian cancer is more common in females with low parity.
u. Most of cases of ovarian cancer are discovered early in stage I
v. Ovarian cancer is almost always bilateral.
w. The fallopian tube is stretched over most of ovarian. neoplasms.
x. Second look laparotomy is done for cases with residual ovarian mass.
y. Stage I C may be treated with ovariotomy.
z.Granulosa cell tumor need long term follow up

135

Gyna & Obstetrics Reveision by/Dr.adool

2008

(15) * The ovary


(1)Endometriosis means the presence of endometrial tissue in the following, EXCEPT:
A) In the lining of the uterine cavity.
B) Within the muscle layer of uterus.
C) In an ovarian cyst.
D) Within the peritoneum.
(2)Adenomyosis may be Associated with the following, EXCEPT:
A) Dysmenorrhaea.
B) Menorrhogia.
C) Amenorrhoea.
D) Infertility.
(3)The leading cause of death, due to female genital neoplasm, is:
A) Ovarian
B) Tubal.
C) Uterine corpus.
D) Cervical.
E) Vulval.
(4)The Earliest Symptoms of Ovarian malignancy is:
A) Abdominal pain.
B) Dysuria.
C) Constipation.
D) Menorrhagia.
E) None f the above.
(5)Diagnosis of Cancer ovary, in the first place, depends upon:
A) CA 125 level.
B) Alfa faeto protein level.
C) Pelvic examination.
D) Ultrasonographny.
E) CT scan.
(6)Ovarian Neoplasms, most commonly, arise from:
A) Ovarian epithelium
B) Ovarian stroma.
C) Germ cells.
D) Sex cords.
(7)Ruptured Dermoid Cyst will Cause:
A) Ascitis.
B) Chemical peritonitis.
C) Pseudomyxoma peritonei.
D) Malignant spread.

136

Gyna & Obstetrics Reveision by/Dr.adool

2008

(8)Ruptured mucinous cystadenoma will lead to the development of:


A) Ascitis.
B) Chemical peritonitis.
C) Pseudomxoma peritonei.
D) Malignant spread.
(9)Diagnosis of twisting of an ovarian cyst depends, in the first place, upon:
A) History and clinical examination.
B) CA 125 estimation.
C) Ultrasonography.
D) Hysteroscopy.
(10)The Common Neoplasm to be twisted during pregnancy is:
A) mucinous cystadenoma.
B) Dermoid cyst.
C) Papillary cystadenoma.
D) Fibroma of the ovary.
(11)Twisted ovarian cyst, during pregnancy, is treated by:
A) Laparotomy/ovariatomy.
B) Conservative measures.
C) Postponement until foetal viability, then coesarean + ovariotmy.
D) Laparoscopic drilling.
(12)Which of the following is NOT an ovarian neoplasm of epithelial origin:
A) Papillary cystadenoma.
B) Mucinous cystadenoma.
C) Endodermal sinus tumour.
D) Endometroid tumour.
(13)Differentiation between ovarian fibroma and Brenners tumour is by:
A) History and clinical examiantion.
B) Ultrasonography.
C) CA 125.
D) Pathological examiantion.
(14)Ovarian Fibroma is susceptible to:
A) All complications of uterine myomata.
B) Meigs syndrome.
C) Twisting.
D) All of the above.
(15)The Ovarian cyst with external papillae, is:
A) Dermoid cyst.
B) Papillary cystadenoma.
C) Mucinous cystadenoma.
D) Endometroid cyst.

137

Gyna & Obstetrics Reveision by/Dr.adool

2008

(16)Structure of epithelium in papillary cystadenoma looks like:


A) Tubal epithelium.
B) Cervical epithelium
C) Vaginal epithelium.
D) Endometrium.
(17)Mucinous Cystadenoma:
A) Constitutes 30% all ovarian tumours.
B) May reach huge size, though benign.
C) Its cells look like those of the endocervix.
D) All of the above.
E) None of the above.
(18)Solid ovarian tumours include the following, EXCEPT:
A) Fibroma of the ovary.
B) Brenners tumour.
C) Benign cystic tertoma.
D) Epithelial carcionoma.
(19)Which of the following tumours is andrgoen secreting?
A) Granulosa cell tumour.
B) Theca cell tumour.
C) Sertoli Leyding cell tumour.
D) Germinoma.
(20)Removal of an ovarian cyst together with the ovary in called:
A) Ovariotomy.
B) Oophorectomy.
C) Ovarian cystectomy.
D) Panhysterectomy.
(21)Krukenberg tumour is a secondary ovarian metastasis in which:
A) The tumour shows the same pathology as the primary.
B) The tumour is unilateral.
C) The ovaries are enlarged, yellowish and retain their shape.
D) All of the above.
(22)Subjects susceptible to ovarian carcinoma, are also susceptible to:
A) Cancer breast, cancer cervix and cancer colon.
B) Cancer breast, cancer endometrium and cancer colon.
C) Cancer thyroid, cancer endometrium and cancer colon.
D) All of the above.

138

Gyna & Obstetrics Reveision by/Dr.adool

2008

(23)Monodermal teratomas include:


A) Struma ovarii
B) Argyntaphinoma (Carcinoid).
C) Non-gestational choriocarcinoma.
D) All of the above.
E) Non of the above.
(24)Ovarian tumour in young subjects (20 years or less) is:
A) Primary ovarian carcinoma.
B) Germinoma.
C) Krukenberg tumour.
D) Theca cell tumour.
(25)Ovarian Tumour, limited to one ovary, no pelvic seedling but with malignant cells in the peritoneal aspirate, is
staged as:
A) IA Stage.
B) IIA Sage.
C) IC Stage.
D) IIB Stage.
(26)The most important principle in the treatment of ovarian cancer is:
A) Cytoreductive surgery
B) Irradiation.
C) Chemotherapy.
D) Examination of tumour cells cultured in vitro.
(27)Which, of the following subjects, is LEAST susceptible to cancer ovary:
A) Woman with history of breast cancer.
B) Nulliparous woman with a history of normal menses.
C) Multiparous woman with history of oral contraception.
D) None of the above.

139

Gyna & Obstetrics Reveision by/Dr.adool

2008

(16) * Family planning and contraception


1- The theoretical effectiveness of a given contraceptive is affected by
A) patient motivation
B) its relationship to the act of coitus
C) its ease of use
D) its antifertility action
2- The major source of the differences between the theoretical expected failure rate and the failure rate in users for
the various contraceptive methods is:
A) attitude toward sexual behaviour
B) patient compliance
C) frequency of sexual intercourse
D) age of users
3- Conventional contraceptive includes one of the following :
A) Condom
B) Copper- T
C) Oral pills
D) Tubectomy.
4- Which is Not a barrier method of birth control?
A) condom
B) cervical cap
C) diaphragm
D) douche
5- All the following methods of contraception are barrier methods except:
A)
B)
C)
D)

spermicides
male condoms
withdrawal
Female condom

6- The Contraceptive Diaphragm , all are true EXCEPT:


A)
B)
C)
D)

should be checked for fit postpartum


should be used in conjunction with a spermicide
can safely be removed immediately after intercourse
should not be left in situ for more than 24 hours

7- Using a condom and spermicide concurrently has a method effectiveness rate of:
A)
50 percent
B)
75 percent
C)
85 percent
D)
>90 percent
8- Lactational amenorrhea is prolonged in association with:
A) Increased suckling of the infant
B) Decreased frequency of nursing
C) Low plasma prolactin
D) Ovulation

140

Gyna & Obstetrics Reveision by/Dr.adool

2008

9- In a woman exclusively breast-feeding, the risk of ovulation within the first 6 months postpartum is:
A) 1-5%
B) 5-10%
C) 10-20%
D) 20-30%
10- Coitus interruptus (withdrawal) often fails as a method of birth control, because:
A) prostatic secretions containing sperm are released during the excitement and plateau phases of sex response
B) many men start ejaculating before they realize it
C) Cowper's gland secretions often contain sperm
D) All of the above
11- Methods of contraception designed to help the woman determine when she will ovulate include all of the
following except :
A) calendar method
B) basal body temperature measurement
C) cervical mucus observation
D) combined oral contraceptives
12- A 24-year-old woman seeks information on contraceptive efficacy and mechanism of action. In discussing oral
contraceptives, you correctly advise her that the low-dose combination OCP's prevent
pregnancy primarily by altering:
A)
B)
C)
D)

gonadotropin secretion
cervical mucus thickness
endometrial maturation
Fallopian tube motility

13- All of the following are proven to be established benefits of the estrogen / progestogen oral contraceptive pill
EXCEPT:
A)
B)
C)
D)
E)

Reduction in incidence of cervical carcinoma


Reduction in incidence of menorrhagia.
Reduction in incidence of benign breast disease.
Reduction in incidence of pelvic inflammatory disease.
Reduction in incidence of ovarian carcinoma.

14- Which of the following is not a well-established health benefit of the oral contraceptive pill?
A)
B)
C)
D)
E)

reduction in ovarian cancer


reduction in endometrial cancer
reduction in benign breast disease
protection against pelvic inflammatory disease
reduction in size of uterine leiomyomas

15- Absolute contraindications to the combined Pill are:


A) breast cancer and arterial thrombosis
B) ovarian endometriosis
C) uterine fibroids
D) Von Willebrands disease
E) previous ectopic pregnancy

141

Gyna & Obstetrics Reveision by/Dr.adool

2008

16- What hormone normally produced in the corpus luteum that inhibits development of ovarian follicles and is a
common component of oral contraceptives?
A)
B)
C)
D)
E)

testosterone
follicle-stimulating hormone (FSH)
lutenizing hormone (LH)
prolactin
progesterone

17- A combined birth control pill prevents pregnancy because the hormones in it do all of the following except:
A) inhibit ovulation
B) make cervical mucus impenetrable for the sperm
C) are spermicidal
D) Alter the endometrium
18- The following are side effects of oral contraceptives except:
A) Breast pain.
B) Break-through bleeding.
C) Increased libido
D) Depression
19- Female birth control pills act by inhibiting the production of:
A) CTH and LH
B) TSH and FSH
C) ACTH and LTH
D) FSH and LH
E) estrogen and progesterone
20 -The effectiveness of a combined oral contraceptive may be reduced by all EXCEPT:
A) Phenytoin
B) Rifampicin
C) Ampicillin
D) Digoxin
21- Oral contraceptives are contraindicated in patients with all EXCEPT:
A) Migraine headaches.
B) Past history of infectious hepatitis.
C) Past history of a benign breast cyst.
D) Past history of thrombophlebitis.
E) Dysmenorrhea.
22- low dose contraceptive pill is contraindicated in which of the following conditions:
A) regular menses
B) age over 35 and heavy smoker
C) menorrhagia
D) Dysmenorrheal
23- The greatest risk for healthy woman under 35 , who is on OCP is:
A) pregnancy from forgetting to take the pill
B) ovarian carcinoma
C) endometrial carcinoma

142

Gyna & Obstetrics Reveision by/Dr.adool

2008

D) heart attack and strokes


E) contracting STD's
24- The following statements are true concerning the combined oral contraceptive pill (COC):
A) Patients for laparoscopic sterilisation should stop COCs 4 weeks prior to the operation
B) Pill hypertension' bears no relationship to the progestogen content of the pill
C) The incidence of gallstones and cholecystitis is increased
D) COCs should not be used by insulin-dependent diabetics because of interference with diabetic control
E) Used safely in women with hepatitis
25- The combined pill is protective against which of the following:
A) Cervical dysplasia
B) Benign breast disease
C) Ovarian cancer
D) Crohn's disease
26- Which of the following characterizes the use of combined oral contraceptives during breast feeding?
A) Is contraindicated
B) Causes a dose-related suppression of the quantity of milk produced
C) Masculinizes the female fetus
D) Decreases child's growth
27- Which of the following contraceptives does not inhibit lactation?
A) Ethinyl estradiol 50 g
B) Mestranol 50 g
C) Norethindrone 0.35g
D) Ethinyl estradiol 35 g
28- For the woman who wishes to lactate and take oral contraceptives (OC) the OC of choice is:
A) A low dose combined pill
B) A high dose combined pill
C) A triphasic preparation preparation
D) Progestin only
29- The absolute contraindications to the POP are:
A) Hypertension
B) Following evacuation of a hydatiform mole
C) Ectopic pregnancy
D) Jaundice in pregnancy
E) Thromboembolic disease
30- Birth-control pills have been associated with hepatic adenomas. Most of these tumors are:
A) benign
B) hormone producing
C) diabetogenic
D) associated with thromboembolism
E) associated with urinary tract infections
31- In patients taking combination type birth-control pills, altered metabolic functions may include:
A) a decrease in glucose tolerance
B) an increase in binding globulins

143

Gyna & Obstetrics Reveision by/Dr.adool

2008

C) an increase in triglycerides
D) All of the above
32- The manufacture of ethinyl estradiol was an important break-through in the development of oral contraceptives,
because the agent was discovered to be
A) an especially effective estrogen
B) orally active
C) less potent and therefore better tolerated than diethylstilbestrol
D) the endogenously active form of estrogen
E) an estrogen with a unique action on the hypothalamus
33- The major cause of oral contraceptive failure resulting in an unplanned pregnancy is:
A) breakthrough ovulation at midcycle
B) frequency of intercourse
C) incorrect use of oral contraceptives
D) gastrointestinal malabsorption
E) development of antibodies
34- Progestational agents in birth control pills have which of the following actions?
A) Inhibition of secretion of follicle stimulating hormone
B) Endometrial hyperplasia
C) Thickening of cervical mucus
D) Prevention of irregular menses
35- The contraceptive effect of birth control pills containing both synthetic estrogen and progestin is related to the:
A) inhibition of ovulation
B) impaired penetrability of sperm into the cervical mucus
C) atrophic changes of the endometrium impairing implantation
D) All of the above
36- For each situation listed below select the most appropriate response.
A) Stop pills and resume after 7 days
B) Continue pills as usual
C) Continue pills and use an additional form of contraception
D) Take an additional pill
E) Stop pills and seek a medical examination
1) Nausea during First cycle of pills
2) No menses during 7 days following 21 -day cycle of correct use
3) Forgot pill 1day
4) Forgot pill 10 continuous days
5) Light bleeding at midcycle during first month on pill
6) Hemoptysis
37- In a 29-year-old woman taking oral contraceptives, amenorrhea is most likely due to:
A) pregnancy
B) pituitary tumor
C) Asherman syndrome
D) relative progresterone excess in the contraceptive
E) relative estrogen excess in the contraceptive

144

Gyna & Obstetrics Reveision by/Dr.adool

2008

38- A 21-year-old woman with regular menses occurring at 26-29-day intervals is referred to you by her dermatologist
for treatment of acne and for contraception. You counsel her regarding therapy, and she elects to begin hormone
contraception. The most appropriate contraceptive for her is :
A)
B)
C)
D)
E)

Norethindrone, 0.35 mg orally daily


Ethinyl estradiol, 35 g, and norgestimate of varying doses 21 out of 28 days
Ethinyl estradiol, 50 g, and norgestrel, 0.5 mg 21 out of 28 days
Depot medroxyprogesterone acetate (Depo-Provera) intramuscularly every 3 months
Subdermal levonorgestrel implants (Norplant)

39- A 28-year-old woman, gravida 2, para 2, has been taking an oral contraceptive containing 35 g ethinyl estradiol
and 1 mg norethindrone for 1 year. Her only concern is vaginal spotting, which has interfered with her daily
activities for several cycles. The spotting is most evident in the early part of her cycle. The most appropriate
option is to:
A)
B)
C)
D)
E)

discontinue the pill


add 0.625 mg conjugated estrogen daily to the current pill for several cycles
switch to a 50-g g ethinyl estradiol pill with the same progestin
switch to a 35-g g ethinyl estradiol pill with a more potent progestin
switch to a progestin-only pill for several cycles

40- If during OC pills intake severe abdominal pain occur. What is your possible diagnosis?

A)
B)
C)
D)

Acute intestinal obstruction


Systemic lupus
Mesentric vein thromobosis
Anemia

41- A 23 year old has a new sexual partner. She has been on the combined OC pill for last 6 yrs. She presents with a 2month history of breakthrough bleeding. What is the single most relevent next examination?
A) Endocervical swab
B) Endometrial sampling
C) Full blood count
D) Gonadotropin levels
E) Hysteroscopy
42- Which of the following statements is the best explanation for the mechanism of the action of the IUD?
A) Hyperperistalsis of the fallopian tubes accelerates the transport of the ovum. thereby preventing fertilization
B) The IUD causes a bacterial endometritis that interferes in implantation
C) The IUD produces menorrhagia and the embryo is aborted in the heavy menstrual flow
D) A sterile inflammatory reaction of the endometrium to the IUD prevents implantation
E) A hormonal imbalance is caused by the IUD.
43- Pick out the false information regarding Copper-T:
A)
B)
C)
D)

145

Best time of insertion is 2-3 days after the periods are over
Cu-T should not be inserted post abortal
Withdrawal technique is used for insertion of Cu-T.
In Cu-T 200, the numerical part stands for the surface area of copper wire.

Gyna & Obstetrics Reveision by/Dr.adool

2008

44- Copper IUCD, all are true EXCEPT:


A)
B)
C)
D)

is associated with increased menstrual bleeding


is contra-indicated in the patient with uterine malformation
the risk of infection is mainly in the first three weeks after insertion
is contraindicated in patients with previous CS.

45- A lady with IUCD becomes pregnant with tail of IUCD being seen, next course of action is :

A)
B)
C)
D)

Remove the IUCD and terminate pregnancy


Remove IUCD Continue the pregnancy
Remove IUCD and terminate pregnancy
Leave the IUD and terminate pregnancy

46- Lost intra-uterine contraceptive device, all are true EXCEPT:


A)
B)
C)
D)

Pregnancy may be a cause as threads may no longer become visible.


alternative contraception should be arranged until the IUCD has been located
the threads may be retrieved in most cases from within the cervical canal
an extra-uterine IUCD should be removed by hysteroscopy

47- The standard (Cu T 380 A ) IUCD is appropriate for a uterus having uterocervical length of:
A) 5-6 cm
B) 6 - 8 cm
C) 9 - 10 cm

D) > 10 cm
48- An ideal candidate for the use of IUCD as a contraceptive method should not be:
A) Nulliparous
B) Multipara
C) Having normal menstrual cycle
D) Breast feeding
50- The best of the following choices of contraceptives for a woman who is 42 years old, smokes, and has four
children (and wants to be sure she has no more) is:
A) combined birth control pills
B) intrauterine device
C) natural fertility awareness
D) contraceptive foam
51- The risks of the intrauterine contraceptive device (lUCD), all are not true EXCEPT:
A) Higher incidence of pelvic infection amongst parous than nulliparous women
B) The majority of ectopic pregnancies seen with the IUCD are ovarian
C) The risk of perforation is increased by using a tenaculum
D) May cause endocarditis in patient with rheumatic heart disease
52- The following statements are true concerning the IUCD EXCEPT:
A) The risk of infection increases in the period following insertion
B) The uterine cavity remains colonised with bacteria while the IUCD remains in situ
C) Menorrhagia may be controlled by aminocaproic acid
D) If an 1UCD is still present at the start of the second trimester, abortion is increased by 50%

146

Gyna & Obstetrics Reveision by/Dr.adool

2008

53- The amount of progesterone released per day by the progesterone releasing IUCD ( Progestarest ) is:
A) 60 mugm
B) 65 mugm
C) 70 mugm
D) 75 mugm
54- Absolute contraindications to the IUCD is:
A) Uterine fibroids
B) Cervical ectopy
C) Immunosuppressive therapy
D) Previous caesarean section
E) Lactation
55- A pregnancy of approximately 10 weeks gestation is confirmed in a 30-year-oid woman (gravida 5, para 4) with an
intrauterine device (IUD) in place. The patient expresses a strong desire for the pregnancy to be continued. On
examination the strings of the IUD are protruding from the cervical os, The most appropriate course of act ion
would be:
A) leave the IUD in place without any other treatment
B) leave the IUD in place and continue prophylactic antibiotics throughout pregnancy
C) remove the IUD immediately
D) terminate the pregnancy because of the near certain risk of infection. abortion, or both
E) perform laparoscopy to rule out an ectopic pregnancy
56- The insertion of an IUD would not be contraindicated in a 24-year-old woman under which of the following
conditions?
A) A recent, unexplained, abnormal Papanicolaou (Pap) smear
B) Nulligravid with multiple partners
C) Previously treated epsiodes of pelvic inflammatory disease
D) The last day of menses
57- Depot Provera , all are not true EXCEPT:
A) contains levonorgestrel
B) is normally given monthly
C) may produce amenorrhoea
D) causes permanent infertility in a small % of patients
E) inhibits lactation
58- Depot-medroxypcogesterone acetates (DMPA), all are false EXCEPT:
A) Fertility usually returns within 2 months after the last injection
B) There is increase in the incidence of moniliasis
C) Weight gain usually occurs
D) After 1 year regular cycles usually return with increasing duration of use
59- Norplant implants work by slowly releasing ____________ over a period of years, which effectively prevents
conception.
A) progesterone
B) estrogen
C) a combination of both a and b
D) a combination of other hormone

147

Gyna & Obstetrics Reveision by/Dr.adool

2008

60- Norplant, all are true EXCEPT:


A)
B)
C)
D)
E)

contains levonorgestrel
erratic bleeding is the commonest reason for stopping the method
pregnancy rate is <0.5 per 100 woman years
contraceptive effect persists for three months after removal
it is effective for up to five years

61- Which progestin is used for contraception in an implanon implant :


A) Levonorgestrel
B) Norgestimate
C) Norethindrone
D) Etonorgestrel
62- Sterilisation , all are not true EXCEPT:
A) is the best method of contraception.
B) vasectomy increases the risk of coronary artery disease.
C) tubal ligation at the time of Caesarean section has a higher failure rate than laparoscopic sterilisation in the
non-pregnant.
D) female sterilisation eliminates the risk of ectopic pregnancy.
E) female sterilisation advances the age at the menopause by an average of three years.
63- Vasectomy, all are not true EXCEPT:
A) There should be two "blank" specimens over the next 3-6 months before the man can be deemed
B) involves occlusion of the seminiferous tubules.
C) is immediately effective.
D) can be successfully reversed in 70% of cases.
E) is marginally less effective than female tubal clip occlusion.
64- Birth control in females by surgical sterilization generally involves cutting and tying the cut ends of the ________
A)
B)
C)
D)
E)

endometrium
urethra
ovaries
uterus
Fallopian tubes (oviduct)

65- The gynecologic complication which usually follows postpartum tubal ligation is:
A) Ovarian atrophy.
B) Ovarian cystic disease.
C) Metrorrhagia.
D) Pelvic inflammatory disease.
E) None of the above.
66- The commonest complication of laparoscopic ring tubal ligation is
A) Failure of sterilization
B) Menstrual abnormality
C) Mesosalpingeal bleeding
D) Bowel injury
E) Wound infection

148

Gyna & Obstetrics Reveision by/Dr.adool

2008

67- Which of the following cancidates will be immediately eligible for sterilization
A) Women having uncontrolled diabetes
B) Women having Hb < 8 gm %
C) Past history of treated pulmonary TB
D) Febrile patient
68- After vasectomy , a backup contraceptive method is required for how many weeks
A) 8 wks
B) 10 wks
C) 12 wks
D) 14 wks
68- Which of the following is Not true after a person has had a vasectomy?
A) Cowper's glands are not affected by the operation.
B) The vas deferens still transport sperm to the urethra.
C) The interstitial cells can still release hormones.
D) The semen will not contain sperm.
E) The oviduct will not be affected.
69- Sterilisation , all are false EXCEPT:
A) Sterilization of women is safer than Vasectomy
B) The commonest complication of laparoscopic sterilization is failure to obtain a pnemoperitoneum
C) The rate of pregnancy after female sterilization increases after the first year
D) Bowel trauma with sepsis is the commonest cause of death following laparoscopic sterilisaiion
70- Reversal of sterilisation
A) The rate of ectopic pregnancy is 10%
B) Falope rings destroy less tube than Filshie clips
C) Patients from the higher socioeconomic groups are more likely to request reversal
D) Likely to be requested in women sterilized below the age of 30
71- Procedures appropriate for postpartum sterilization at the time of cesarean section include all of the following
except:
A) Pomeroy tuba! ligation
B) Irving tubal ligation
C) Falope rings
D) Hysterectomy
72- Sequelae of vasectomy include one of the following:
A) varicocele
B) rupture testis
C) torsion of the testis
D) the production of sperm antibodies
73- True statements regarding operative procedures for sterilization include which of the following?
A) They can be performed immediately postpartum
B) They are permanent methods of contraception
C) They can be considered effective immediately in females (bilateral tubal ligation)
D) They can be considered effective immediately in males (vasectomy)

149

Gyna & Obstetrics Reveision by/Dr.adool

2008

74- For each method of surgical sterilization described below select the name of the procedure.
1) The distal segment of the fallopian tube is removed either vaginally or abdominally
2) A plain catgut ligature is placed around a knuckle of tube, which is then excised
3) The serosa of the tube is stripped from the muscular portion and 5 cm of tube is excised. The stump is ligated
and the edges of the serosa are tied around the distal tube
4) The tube is transected in the midpoition and the proximal stump is buried into the myometrium
A) Irving technique
B) Pomeroy method
C) Uchida method
D) Fimbriectomy
75- Menstrual regulation can be performed up to
A) 6 wks
B) 12 wks
C) 18 wk
D) 20 wks
76- Which method of contraceptive failure has the highest ectopic rate?
A)
B)
C)
D)

IUD.
Oral contraception.
Tubal sterilization.
Hysterectomy.

77- A 20-year-old woman, gravida 1, para 1, comes to the clinic for her postpartum examination. She is not
breastfeeding. She is undecided on contraception, and she does not want to gain weight. She has multiple sexual
partners. In addition to recommending the use of condoms, you suggest that the best contraceptive method for
her is
A)
B)
C)
D)
E)

Depot medroxyprogesterone acetate (Depo-Provera)


Combined oral contraceptives
Intrauterine device (IUD)
Spermicide
Diaphragm

78- A 25-year-old woman presents to the clinic 3 days after having had unprotected intercourse near midcycle. This
was her only pregnancy exposure during this cycle. A urine pregnancy test is negative, and she has no medical
problems and is taking no medication. Of the following options, the postcoital contraception regimen you would
recommend at this time is:
A) None because postcoital contraception must be initiated within the first 48 hours
B) Danazol (Danocrine), 200 mg
C) Two doses taken 12 hours apart, each dose consisting of two pills containing 50-g ethinyl estradiol and
0.5 mg norgestrel
D) Introduction of a copper intrauterine device
E) Emergency menstrual extraction
79- All of the follwing are mechanisms of action of emergency contraception except:
A) Delaying ovulation.
B) Inhibiting fertilization.
C) Preventing implantation of the fertilized egg.
D) Interrupting an early pregnancy.

150

Gyna & Obstetrics Reveision by/Dr.adool

2008

80- For emergency contraception, a copper-bearing intrauterine contraceptive device


A) Can cover multiple unprotected intercourse in a cycle
B) Is as, but not more, effective than the emergency Pill
C) Is not subject to the traditional contraindications for IUCDs
D) Can be fitted up to five days after the earliest calculated day of ovulation
E) Can be removed 72 hours after insertion
81- Postcoital contraception, all are false EXCEPT:
A) Usually involves taking a total of 200 mg ethinylestradiol
B) If the 'morning after' oral contraceptive procedure fails then there is a substantial risk of fetal abnormalities
C) The overall pregnancy rate fallowing unprotected intercourse at mid-cycle is 3%
D) Following unprotected intercourse an 1UCD must be inserted within 24 hours to prevent implantation
82- Which of the following is/are effective for postcoital contraception?
A) Diethylstilbestrol, 25 mg bid for 5 days
B) Conjugated estrogens. 30 mg for 5days
C) Elhinyl estradiol, 5 mg per day for 5 days
D) Ovral 2 tabs,POq 2h for l day
E) All of the above
83- Concerning other methods of contraception, the following are false except:
A) The pregnancy rate using the diaphragm is less than 5 per 100 women years
B) Spermicides should be used when "natural family planning' ('rhythm' and 'safe period') methods are employed
C) Coitus interruptus has been approved by the Roman Catholic Church
D) Nonoxynol 9 causes aspermia
E) Menstrual regulation can not be achieved using prostaglandin analogues
84- Contraception for the older woman all are not correct EXCEPT::
A) The IUCD can be removed as soon as cessation of the periods occurs
B) Most hormone replacement regimens provide satisfactory contraceptive cover
C) 95% of women who menstruate regularly are ovulating regardless of their age
D) The combined pill is absolutely contraindicated
E) Abdominal hysterectomy
85- Choice of Contraception
For each of the patient described below choose the single most likely preferred method of contraception from the
above list of options. Each option may be used once, more than once or not at all.
1) A married 30 year old mother has a history of deep vein thrombosis. She would like to practice contraception. ( F)
2) A young 19 year old girl has started having sexual relationships. She has come to you for counselling regarding
contraception. (I )
3) A 42 year old woman with 3 children has a recent history of pelvic inflammatory disease. She wants to know what
is the preferred method of contraception for her? ( C )
4) A newly married young woman who has a low IQ wants to postpone her family for another 2 years. ( D )
5) A 28 year old female comes to you and says that her husband is suffering from HIV infection. She wants to know
what is the best method of contraception for her? ( I )
Options:
A.
OC pills
B.
Male condoms
C.
Laparoscopic sterilization
D.
Depot contraception
E.
Vasectomy

151

Gyna & Obstetrics Reveision by/Dr.adool

2008

F.
G.
H.
I.
J.

IUCD
Rhythm method
Emergency contraception
OC pills and male condoms
Female condoms

86. The use of combined oral contraceptive pills reduces the incidence of the following, except:
a) Endometrial carcinoma.
b) Endometriosis.
c) Cervical carcinoma.
d) Dysmenorrhea.
e)
87. In a recently married woman, the most suitable contraceptive method is:
a) OC pills.
b) Minipills.
c) IUD.
d) Tubal sterilization.
88. Minipills contain :
a) Levonorgestrel
b) Medroxyprogesterone acetate
c) Natural progesterone
d) Estradiol + progestin
89.Condoms:
A. Have the additional benefit of reducing sexually transmitted diseases, including human immunodeficiency virus (HW)
disease.
B. Have a higher failure rate in circumcized men.
C. Have a reduced effectiveness in the presence of oestradiol creams.
D. Have a reduced effectiveness in the presence of K-Yjefly.
E. Have a quoted contraceptive efficacy of 323 per 100 woman-years.

90.With regard to laparoscopic clip sterilization:


A. There is a failure rate of approximately 0.5 per cent.
B. Most deaths are due to anaesthetic complications.
C The procedure should not be performed during menstruation.
D. All women should be counselled that male sterilization is a safer alternative.
E. Surgical emphysema is a recognized complication.

152

Gyna & Obstetrics Reveision by/Dr.adool

2008

91. Intrauterine contraceptive devices (IUDs):


A. Increase the risk of pelvic inflammatory disease.
B. Are commonly used as a method of contraception in Egypt.
C. May be used in a nulligravida.
D. Act through prevention of implantation.
E. Commonly cause an increase in menstrual flow.
A. True B. True C. False D. True E. True
92. Intrauterine contraceptive devices (IUDs):
A. Should be inserted premenstrual.
B. Have a high failure rate.
C. Some types are medicated with progesterone to control menstrual flow.
D. Should be removed in early pregnancy if threads are visible.
E. Perforation of the uterus may occur during insertion.
A. False B. False C. True D. True E. True
93 Contraindications of IUD:
A. Pelvic infections.
B. Pregnancy.
C. Diabetes.
D. Nulliparity.
E. Lactation.
A. True B. True C. False D. True E. False
94. Advantages of IUDs:
A. Cheap, available.
B. Does not affect lactation.
C. Requires little motivation.
D. Low failure rate.
E. No effect on sexual intercourse.
A. True B. True C. True D. True E. True
95. Complications of IUD include:
A. Menorrhagia.
B. Pregnancy.
C. Increased risk of endometrial carcinoma.
D. PID.
E. Expulsion.
A. True B. True C. False D. True E. True
96. Complications of IUD include:
A. Difficult removal.
B. Perforation.
C. Copper toxicity.
D. Sexual dysfunction.
E. Salpingitis.
A. True B. True C. False D. False E. True

153

Gyna & Obstetrics Reveision by/Dr.adool

2008

97. Combined oral contraceptive pills:


A. Contain 30 to 50 mcg of ethinyl estradiol.
B. Act through inhibition of the hypothalamus and pituitary.
C. Increase the risk of pelvic inflammatory disease.
D. Increase the risk of thrombo-embolic disease.
E. Commonly cause menorrhagia.
A. True B. True C. False D. True E. False
98. Side effects of combined oral contraceptive pills include:
A. Headache.
B. Nausea.
C. Menorrhagia.
D. Hypotension.
E. Weight gain.
A. True B. True C. False D. False E. True
99. Side effects of combined oral contraceptive pills include:
A. Irritability.
B. Increased risk of thrombo-embolic disease.
C. Hirsutism.
D. Breakthrough bleeding.
E. Galactorrhea.
A. True B. True C. False D. True E. False
100. Regarding the relation between combined contraceptive pills and cancers:
A. Pills may increase the risk of endometrial carcinoma.
B. Pills may increase the risk of choriocarcinoma.
C. Pills may increase the risk of breast carcinoma.
D. Pills may decrease the risk of ovarian carcinoma
E. Pills may decrease the risk of cervical carcinoma.
A. False B. False C. True D. True E. False
101. Contraindications of combined oral contraceptive pills include:
A. Thrombo-embolic disease.
B. Cerebro-vascular disease.
C. Hypotension.
D. Cancer breast.
E. Nulliparity.
A. True B. True C. False D. True E. False
102. Absolute contraindications of combined oral contraceptive pills include:
A. Thrombo-embolic disease.
B. History of thrombo-embolic disease.
C. Diabetes.
D. Age above 35 years.
E. Pregnancy.
A. True B. True C. False D. False E. True

154

Gyna & Obstetrics Reveision by/Dr.adool

2008

103. Non contraceptive uses of combined pills include:


A. Treatment of dysfunctional uterine bleeding.
B. Treatment of benign breast masses.
C. Treatment of fibroids.
D. Treatment of endometriosis.
E. Treatment of congestive dysmenorrhea
A. True B. False C. False D. True E. False
104. Progestin only pills:
A. Exert their contraceptive effect through suppression of ovulation.
B. Exert their contraceptive effect through their effect on the endometrium and
the cervical mucus.
C. Have a failure rate similar to that of combined oral contraceptive pills.
D. Cause a decrease in breast milk production.
E. Decrease the risk of pelvic inflammatory disease.
A. False B. True C. False D. False E. False
105. Progestin only pills exert their contraceptive effect through:
A. Suppression of ovulation.
B. Induction of endometrial changes.
C. Induction of cervical changes.
D. Act as spermicidal.
E. Suppression of the hypothalamus.
A. False B. True C. True D. False E. False
106. Hormonal contraception:
A. Subdermal implants consist of silicone implants releasing levonorgestrel
acting like injectables.
B. Hormone releasing vaginal rings are left for 21 days or for 6 months.
C. GnRH analogues are too expensive with many side effects to be used
as contraceptives.
D. Mifepristone is an antiprogesterone steroid.
E. Mifepristone may be used with prostaglandin as an abortificient.
A. True B. True C. True D. True E. True
107. Injectables:
A. Act through inhibition of ovulation.
B. Act through inducing an unfavorable effect on the cervical mucus
and endometrium.
C. The main drawback of injectables is menstrual irregularities.
D. Suppress lactation.
E. Commonly, there is delay in return of fertility.
A. True B. True C. True D. False E. True

155

Gyna & Obstetrics Reveision by/Dr.adool

2008

108. Barrier methods of contraception include:


A. The condom.
B. The IUD.
C. The dutch cap.
D. The cervical cap.
E. The mifepristone.
A. True B. False C. True D. True E. False
109. The condom:
A. Is made of thick latex rubber.
B. Is supplied lubricated by a spermicidal material.
C. Comes in different sizes.
D. May be used before ejaculation.
E. May be used for treatment of premature ejaculation.
A. False B. True C. False D. False E. True
110. The diaphragm:
A. Is a commonly used method of contraception in Egypt.
B. Should be inserted at least 6 hours before intercourse.
C. Should be left in situ for at least 8 hours after intercourse.
D. Should be used with spermicidal cream or jel.
E. Comes in different sizes.
A. False B. False C. True D. True E. True
111. Physiological methods of contraception:
A. The safe period is a reliable method of contraception.
B. The fertile period is from day 9 to day 16 in all women.
C. The high prolactin level associated with lactation inhibits implantation.
D. The safe period is a suitable method of contraception for a newly married couple.
E. Prolonged lactation is a reliable method of contraception.
A. False B. False C. False D. False E. False
112. Chemical methods of contraception:
A. Preparations include creams, jells, foams and tablets.
B. Act through exerting a toxic effect on the fertilized ovum.
C. Are highly reliable.
D. Are usually combined with barrier methods.
E. May cause allergic reactions in the female but not in the male.
A. True B. False C. False D. True E. False
113. Post coital contraception includes:
A. Insertion of an IUD within 48 hours.
B. Use of spermicidal within 12 hours.
C. Douching immediately after intercourse.
D. Mifepristone.
E. Hormone withdrawal.
A. True B. False C. True D. True E. True

156

Gyna & Obstetrics Reveision by/Dr.adool

2008

114. Sterilization:
A. Tubal ligation is an easily reversible method of contraception.
B. Ectopic pregnancy may occur after tubal ligation.
C. Should be done when the use of IUD is contraindicated or has failed.
D. Tubal ligation may be done during cesarean section.
E. Tubal ligation may be done through laparoscopy.
A. False B. True C. False D. True E. True

* Say true or false about I.U.D


a- It can only cause contraception through inducing monthly abortion.
b- It is impregnated with barium to increase its efficiency.
c- Vaginal discharge is the main cause of its discontinuation.
d- An ideal method in those with double uterus.
e- Should not be used in cases of menorrhagia.
f- Difficult extraction may be due to perforation.
g- Copper impregnated I.U.D. Are more efficient.
h- Should be sterilized before insertion.
i- Insertion in the immediate post-partum period increases risk of perforation and expulsion.
j- It decreases the incidence of pelvic infections.
k- It should be inserted after at least 40 days from previous ectopic pregnancy.
l- Intrauterine pregnancy on top of I.U.D is associated with increased risk of congenital anomalies.
m- Perforation commonly occurs with long standing use of the I.U.D
n- The expired validity of the nova t may approach 6 years.
o- Increase in the menstrual blood loss due to inhibition of carbonic anhydrase.
p- Pregnancy makes removal of the I.U.D easier.
q- Pelvic infections are more common with prolonged use.
r- Lippies lope can be used following cutting of intra-uterine adhesions.
s- Failure rate is less than oral pills.
t- Perforation with copper I.U.D carries the risk of intestinal obstruction.
u- Delay in return of fertility is expected for few months.
v- The user should attend for regular check up.
w- Progestasert I.U.D can decrease the menstrual blood loss.
x- Expulsion is the commonest cause of missed I.U.D.
y- It is an ideal method for lactating females.
z- Pregnancy on top of an I.U.D carries higher risk of abortion.

* As regarding oral contraception pills say true or false.


a- Combined pills can be used for lactating females.
b- Diabetic females should avoid its use.
c- Estrogen component increases acne.
d- Increases risk of hypercoagulability is due to increase platelet adhesiveness by Estrogen.
e- Inhibition of ovulation is the main mechanism of action.
f- Should be stopped before any elective surgery because of the risk of poor wound healing.
g- Amenorrhea may develop in some users due to hypothalamic hypersensitivity.
h- Concomitant use of anti-convulsant drugs increases the efficiency.

157

Gyna & Obstetrics Reveision by/Dr.adool

2008

i- Missing one pill is an indication to stop this cycle.


j- The pills should be taken at the same time every day.
k- Smoking is a relative contraindication for pill use.
l- Spotting and breakthrough are common complications of sequential pills.
m- Triphasic pills are the most physiological.
n- Endometrial atrophy and hyper-prolactinemia may occur with prolonged use.
o- Pregnancy rate in pill users is 2/HWY.
p- Pills can be used to regulate previously irregular cycles
q- Pills increase the risk of endometriosis and pelvic inflammatory disease.
r- Severe headache, chest pain or leg pain is an indication of discontinuation.
s- It increases the risk of spasmodic dysmenorrhea.
t- It increases the risk of gallstones and liver tumors.
u- There is delay in return of fertility up to9 months after stoppage.
v- Spotting needs doubling of the dose then a higher dose preparation.
w- Cervical erosion may occur during administration of the pills.
x- It increases appetite causing increase in body weight.
y- All types increase the risk of endometrial adenocarcinoma.
z- Progesterone only pills can be used in lactating females.

*-As regarding condom say true or false


abcdefghi-

It is usually lubricated to facilitate its application.


Should be applied before erection
Should be removed immediately after ejaculation.
Can be used with other contraceptive methods.
Can be used in those with sexual disorders.
It increases the pleasure of the act.
Can prevent sexually transmitted diseases.
Failure rate is about 10/H.W.Y.
May rupture during intercourse.

* Say true or false


abcde-

Safe period is used in females with irregular cycles.


O.C.P can regulate irregular cycles.
In ladies using chemical methods of contraception immediate post-coital douching.
Cervical cap can be used in recently married couples.
Vaginal diaphragm has a standard size 65mm.

158

Gyna & Obstetrics Reveision by/Dr.adool

2008

Obstetrics MCQs
Index
(1) * Antepartum haemorrhage .

160

(2) * Pre-eclampsia

165

(3) * Eclampsia

169

(4) * Abnormal uterine action .

171

(5) * Contracted pelvis

175

(6) * Complications of 3rd stage

181

(7) * Fetal well being

191

(8) * Fetal & Neonatal asphyxia

193

(9) * Fetal birth injuries .

195

(10) * Operative obstetrics ..

205

(11) * CAESAREAN SECTION (C.S.)

217

159

Gyna & Obstetrics Reveision by/Dr.adool

2008

Obstetrics MCQs
(1) * Antepartum haemorrhage
1.Ante-partum hemorrhage (APH) is
(a) Genital bleeding before labor
(b) Genital bleeding after viability & before labor
(c) Genital bleeding after fetal viability up to the 1st & 2nd stages of labor
2-Antepartum hemorrhage has the following characters except:
(a) Always is of placental origin (early separation)
(b) From a placenta of lower implantation
(c) From a placenta of normal implantation site
(d) Is considered high risk pregnancy
3. Placenta previa (PP) is a placenta with:
(a) Implantation in the fundus
(b) Implantation in the cervix
(c) Implantation in the lateral wall of the upper segment
(d) Low implantation (partially or completely) in the lower segment.
4. Placenta previa is more in the followings except:
(a) Twins pregnancy
(b) Multipara
(c) Primigravida (Pg) with normal singleton pregnancy
(d) Maternal diseases as diabetes, Rh
5. What is false about degrees of placenta previa?
(a) 1st degree (lateralis): low not reaches the internal os
(b) 2nd degree (marginalis): reaches internal os but not covers it
(c) 3rd&4th degrees (centralis): covers(partially or completely)the
internal os
(d) The dangerous PP is the lateralis

160

Gyna & Obstetrics Reveision by/Dr.adool

2008

6. Bleeding of placenta previa is not usually:


(a) Causeless
(b) Painless
(c) Avoidable
(a) Recurrent
7. Placenta previa may cause the followings except:
(a) More premature labor
(b) Early engagement of the head
(c) More malpresentations
(d) Cord prolapse & PROM
8. Placenta previa does not cause
(a) Post-term pregnancy
(b) Inertia
(c) Operative delivery
(d) Post-partum bleeding
9. The best method for diagnosis of PP (placental localization) is
(a) Cystography
(b) Arnniography
(c) US
(d) Plain x-ray
10. What is false in lines of treatment of PP?
(a) At home make PV to evaluate the condition
(b) Ready available safe blood
(c) Expectant or active treatment
(d) ARM & pitocin drip (mild bleeding) or CS (severe bleeding)
11. Regarding the expectant treatment of PP what is wrong?
(a) Indicated in mild bleeding with a premature fetus

161

Gyna & Obstetrics Reveision by/Dr.adool

2008

(b) Indicated in fetal demise


(c) Includes materno-fetal observation & evaluation
(d) Ended on severe attack, labor, > 37 w
12. What is false in the active treatment of placenta previa?
(a) Done in severe, persistent bleeding or labor pains
(b) Done on reaching 40 week
(c) CS in severe bleeding, ARM in mild bleeding
(d) Care for post-partum bleeding
13. A woman coming into the emergency room presents with amenorrhea 7 months, moderate vaginal bleeding, BP
100/70, the uterus is lax with easily palpable fetal parts, audible FHS, fundus level is at level of 30 weeks, the
possible diagnosis includes the, followings except:
(a) Placental abruption
(b) Placenta previa
(c) Cervical lesions
(d) Coital trauma
14. In placenta previa amniotomy & abdominal binder are done in the followings except:
(a) Mild bleeding with dilated cervix
(b) PP lateralis
(c) PP marginalis posterior
(d) Vertex presentation
15. Lower segment cesarean section (LSCS) is indicated in the followings except:
(a) Severe bleeding, PP centralis, vasa previa, marginalis posterior
(b) Other indications for CS as malpresentations, contracted pelvis,
(c) Failed induction
(d) IUFD with mild bleeding
16. A patient at 37 weeks pregnancy with painless vaginal bleeding what is the immediate measure?
(a) CS
(b) Induction of labor

162

Gyna & Obstetrics Reveision by/Dr.adool

2008

(c) Coagulation profile


(d) US examination
17. Accidental hemorrhage means the followings except:
(a) Premature separation of a normally implanted placenta
(b) Placental abruption (PA)
(c) Abruptio placentae
(d) Premature separation of a low implanted placenta
18. The followings may be causes of placental abruption (PA) except:
(a) Preeclampsia-eclampsia
(b) Folic acid deficiency
(c) Gradual leak of amniotic fluid
(d) External trauma
19. What is false in types of accidental hemorrhage?
(a) Revealed (external): blood escapes outside through the cervix
(b) Concealed: blood retains inside the uterus
(c) Mixed: blood partially retains inside & partially escapes out
20. What is incorrect in the pathogenesis of placental abruption?
(a) Preeclampsia ---degeneration of decidual blood vessels
----retroplacental bleeding
(b) Atonic uterus with adherent placental edge ---- revealed type
(c) In severe cases blood may extravasate through muscle fibers
----peritoneum (Couvelaire's uterus)
(d) Sudden gush of the amniotic fluid ---- Decrease intra-uterine
pressure---- placental separation ---- bleeding
21. The followings are suggestive of placental abruption except:
(a) Lax abdomen & uterus
(b) Upper placenta

163

Gyna & Obstetrics Reveision by/Dr.adool

2008

(c) Severe pallor with mild vaginal bleeding


(d) Fundal level is more than period of amenorrhea
22. What is false in clinical features of the concealed placental abruption?
(a) Abdominal rigidity & tenderness
(b) Fetal distress or death
(c) Severe vaginal bleeding
(d) Retroplacental hematoma
23. Severe placental abruption can cause the following complications except:
(a) Fetal demise
(b) DIC & hypofibrinogenemia
(c) More Rh sensitization
(d) Acute renal failure
(e) Post-partum bleeding ---- may cause Sheehan's syndrome
24. If you suspect placental abruption you must do the followings except:
(a) US examination
(b) Preparation of a fresh blood
(c) Coagulation profile
(d) Immediate CS
25. woman coming to emergency room presents with severe pallor, BP 80/50. tense uterus, impalpable fetal parts,
inaudible FHS with mild vaginal bleeding what is the possibility?
(a) Placenta previa (PP)
(b) Complete ruptured uterus
(c) Accidental hemorrhage
(d) Red degeneration
26. What is false in consumption coagulopathy in the severe placental abruption?
(a) Release of placental thromboplastin--- (DIC)
(b) Hyperfibrinogenemia

164

Gyna & Obstetrics Reveision by/Dr.adool

2008

(c) Activation of fibrinolytic system--- (FDPs)


(d) Diagnosed by Weiner test
27. What is false in treatment of placental abruption?
(a) Evaluation, correction of shock & TTT of hypofibrinogenemia
(b) ARM + pitocin drip + abdominal binder
(c) CS in failed induction or severe bleeding
(d) CS the best TTT
(e) Measures for P.P. hemorrhage

(2) * Pre-eclampsia;
1. Hypertensive disorders of pregnancy include only one of the followings
(a) Pregnancy induced hypertension (PIH)
(b) Chronic nephritis
(c) Essential hypertension
(d) All of above
2. Pregnancy induced hypertension (PIH) does not include
(a) Preeclampsia (PE)
(b) Eclampsia
(c) Essential hypertension
(d) Gestational hypertension
3. The predisposing factors for preeclampsia (PE) include followings except:
(a) Multiparity
(b) Extremes of age (<20 & >35)
(c) Obstetric factors as mole, twins, hydramnios, & Rh disease
(d) Medical diseases as hypertension, diabetes, & nephritis
4. What is false in the pathophysiology of preeclampsia?
(a) Abn. placentation(deficient trophoblastic invasion of arteries)
(b) Vasodilatation

165

Gyna & Obstetrics Reveision by/Dr.adool

2008

(c) Utero-placental ischaemia (decrease perfusion)


(d) Vascular endothelial damage (endotheliosis)
5. What is false in the pathology of severe preeclampsia ?
(a) Arteriolar vasospasm (patho-physiology)
(b) Hemodilution ---- decrease serum uric acid (patho-chemistry)
(c) Subcapsular hemorrhage in the liver
(d) Renal glomerular endotheliosis
(e) Brain edema & may hemorrhage
6. In preeclampsia the following pathophysiological changes correct except:
(a) Increase Glomerular filtration rate
(b) Increase plasma uric acid
(c) Decrease Placental perfusion
(d) Decrease Intravascular volume
7. Preeclampsia (PE) is
(a) Hypertension (> 140/90 mmHg)
(b) Hypertension + edema
(c) Proteinuria (> 0.3 gm/24 h) + edema
(d) Hypertension + proteinuria with or without edema after 20 week
8. Hypertension is one of the followings except:
(a) Blood pressure of > 140/90 mmHg
(b) A rise of systolic blood pressure > 30 mmHg
(c) A rise of diastolic blood pressure > 15 mmHg
(d) Any rise of blood pressure
9. What is false in proteinuria during pregnancy?
(a) Usually it appears before hypertension in preeclampsia
(b) Detected by boiling, Albustix tapes & sulphosalicyclic acid
(c) Caused by preeclampsia and nephritis

166

Gyna & Obstetrics Reveision by/Dr.adool

2008

(d) May be due to contaminated urine by vaginal discharge or mucus


10. During pregnancy the pathological edema does not include
(a) Preeclampsia
(b) Ankle hydrostatic edema
(c) Cardiac edema (congestive heart failure)
(d) Renal edema (proteinuria, hypoproteinemia)
11. The followings are clinical features of the mild preeclampsia except:
(a) Blood pressure > 140/90 - <160/110 mmHg
(b) Epigastric pain
(c) Proteinuria > 0.3 gm /24 hour urine or 1 +
(d) May swelling of abdominal wall, face & fingers (edema)
12. To diagnose a severe preeclampsia you must detect:
(a) Blood pressure >160/110 mmHg
(b) Proteinuria > 5 gm/24 hour urine or oliguria (<400 ml/24 hour)
(c) HELLP syndrome
(d)

Any one or more of above

13. Regarding preeclampsia (PE) what is false?


(a) Blood pressure 170/115 mmHg, proteinuria +++ -> severe PE k
(b) Epigastric pain, blurring of vision - mild PE
(c) Severe PE + hyperreflexia -> imminent eclampsia
(d) Rapid deterioration of PE -> fulminating PE
14. Early maternal complications of severe preeclampsia include followings except:
(a) Eclampsia
(b) Hemorrhage (accidental, coagulopathy)
(c) Renal impairment
(d) IUGR

167

Gyna & Obstetrics Reveision by/Dr.adool

2008

15. What is false in the fetal complications of severe preeclampsia?


(a) Macrosomia
(b) Prematurity
(c) Fetal hypoxia & distress
(d) IUFD
16. Essential hypertension with pregnancy has the following features except:
(a) Antedate pregnancy
(b) No proteinuria
(c) Edema
(d) Atherosclerosis & cardiac enlargement
17. Chronic nephritis with pregnancy has the following feature except:
(a) Antedates pregnancy
(b) No edema but proteinuria
(c) May altered renal functions
(d) Hypertension precedes proteinuria
18. Prevention of preeclampsia may include the followings except:
(a) Proper antenatal care
(b) Salt restriction
(c) Early detection by roll over test
(d) Low dose aspirin
19. Treatment of mild preeclampsia includes the followings except:
(a) Bed rest, regular diet & hypotensive drugs
(b) Maternal observation
(c) Fetal observation & evaluation
(d) Magnesium sulfate (MgS04) & immediate delivery
20. What is fake in treatment of severe preeclampsia?
(a) Control of hypertension

168

Gyna & Obstetrics Reveision by/Dr.adool

2008

(b) MgS04 to avoid eclampsia


(c) Maternal & fetal evaluation
(d) Allow pregnancy to full term

(3) * Eclampsia;
1. What is false in eclampsia?
(a) Severe preeclampsia complicated by fits
(b) The post-partum eclampsia is common and occurs after 2 weeks
(c) MgS04 is the drug of choice for control of the fits
(d) Termination of pregnancy is usually essential
2. Regarding eclampsia the followings are true except:
(a) An obstetric emergency needs special intensive care
(b) Must be differentiated from other causes of convulsions as epilepsy
(c) More than 3 fit -> bad prognosis
(d) Cerebral hemorrhage is a cause of maternal mortality
3. What is wrong in treatment of eclampsia?
(a) No prophylactic treatment
(b) Aims are to keep patent air passage with control of fits and hypertension
(c) MgS04, Valium & phenytoin for control of fits
(d) Delivery of the fetus by CS or induction after patient stabilization
4. What is false in general treatment of eclampsia?
(a) Keep the patient semisetting in a completely dark room
(b) Oxygen, mouth gag and airway, skilled nurse
(c) Close observation for the maternal vital signs in the ICU
(d) Give atropine to dry the bronchial secretion
5. What is wrong about use of MgS04 in eclampsia?
(a) Anticonvulsant, vasodilator, sedative & diuretic
(b) The initial dose is 8 mg (4 infusion + 4 IM), then 4 mg/4h

169

Gyna & Obstetrics Reveision by/Dr.adool

2008

(c) Check serum level, urine output, respiratory rate and knee jerk/4 h
(d) Antidote is calcium gluconate 10 ml 10%
6. Complications ofMgS04 include the followings except:
(a) Respiratory depression of both the mother & neonate
(b) Hypocalcemic tentany
(c) Neonatal hyperreflexia
(d) May maternal cardiac arrest & post-partum bleeding (tocolysis)
7. Alternatives to MgS04for control of the eclamptic fits include the followings except:
(a) Phenytoin (Dilantin)
(b) Diazepam (Valium)
(c) Morphine
(d) Lytic cocktail
8. Side actions of diazepam (Valium) include the followings except:
(a) Decrease Fetal beat to beat variability
(b) May affect the maternal liver
(c) Neonatal respiratory depression
(d) Maternal hypocalcemia
9. The used hypotensive drugs in eclampsia include the followings except:
(a) Oral methyl dopa (Aldomet)
(b) IV hydralazine
(c) Sublingual nifedipine (Epilat)
(d) IV labetalol (Trandate)
10. What is false about delivery in eclampsia
(a) Urgent CS even with a dead fetus
(b) Induction in a favorable cervix and presentation
(c) Shortening of the 2nd stage by ventouse (forceps)
(d) Giving oxytocin instead of methargin in the 3rd stage

170

Gyna & Obstetrics Reveision by/Dr.adool

2008

11. What is false about pheochromocytoma?


(a) Arises from the adrenal cortex
(b) Produces excessive catecholamines
(c) Causes hypertension
(d) Needs surgical removal
12. HELLP syndrome includes the followings except:
(a) H: hemolysis
(b) EL: elevated liver enzymes
(c) L: Low platelets
(d) P: Peritonitis

(4) * Abnormal uterine action


1-Hypotonic uterine inertia has the following criteria except :
A. Patient at risk for postpartum hemorrhage
B. Improper use of ecbolic drugs is a cause
C. Fetus and mother are not distressed
D. Membrane is usually intact
2-Abnormal uterine action:
A. Is one of the causes of dystocia.
B. Is commonly associated with malpresentations.
C. Is rarely associated with disproportion.
D. Uterine contractions are always insufficiently strong.
E. Cervical dilatation is delayed in all cases.
3-Precipitate labor:
A. Is associated with strong uterine contractions.
B. Is rarely associated with post partum hemorrhage.
C. Labor takes place in less than 2 hours.
D. Carries an increased risk of complications to both t mother and fetus.
E. General anesthesia may be given late in the first stage.
4-Potential complications of precipitate labor include:
A. Avulsion of the umbilical cord.
B. Vaginal lacerations.
C. Obstructed labor.
D. Atonic post partum hemorrhage.
E. Puerperal sepsis.
5-Hypotonic inertia:
A. May be primary or secondary.
B. Is associated with malpresentations.

171

Gyna & Obstetrics Reveision by/Dr.adool

2008

C. May reflect the presence of unrecognized disproportion.


D. Commonly ends in precipitate labor.
E. The fetus is commonly distressed.
6-Hypotonic inertia:
A. Should be differentiated from prodroma of labor.
B. Assessment of the efficiency of uterine contractions is best achieved by internal tocography.
C. Amniotomy is beneficial during the latent phase.
D. Augmentation is required in all cases.
E. Secondary inertia is best managed by elective cesarean section.
7-Contraindications of use oxytocin include:
A. Some cases of disproportion.
B. Breech presentation.
C. Elderly primigravida.
D. Grand multipara.
E. After rupture of membranes.
8-Contraindications of use oxytocin include:
A. Face presentation.
B. Occipito posterior positions.
C. Hypertonic inertia.
D. Cervical dystocia.
E. Fetal distress.
Hypertonic inertia:
A. May be due to in-coordination of contractions.
B. May be due to lack of fundal dominance.
C. May end in precipitate labor.
D. Is commonly managed by cesarean section.
E. May end in a contraction ring.
9-Contraction ring:
A. Is a persistent localized annular spasm.
B. May cause arrest of cervical dilatation.
C. May cause prolongation of the second stage of labor.
D. May be responsible for a retained placenta.
E. Clinically; it is similar to retraction ring.
10-Cervical dystocia:
A. Involves failure of cervical dilatation with or without good uterine contractions.
B. With functional rigidity, the internal os fails to dilate.
C. Is one of the causes of hypertonic inertia.
D. Analgesics and antispasmodics usually effectively relieve functional rigidity.
E. Functional rigidity may require a cesarean section.
11-Contraction ring:
A. May cause arrest of cervical dilatation.
B. The mother is not necessarily distressed.
C. Always lies between the upper and lower segments.
D. Rises up.

172

Gyna & Obstetrics Reveision by/Dr.adool

2008

E. Can not be seen abdominally.


12-Contraction ring:
A. Is also known as Bandl's ring.
B. Is one of the causes of retained placenta.
C. The uterus is tonically retracted.
D. The fetus is distressed or dead.
E. May relax with anesthesia.
13-Incoordinate uterine contractions includes:
1- Contraction ring.
2- Cervical dystocia.
3- Colicky uterus.
4- Retraction ring.
5- Cervical incompetence.
14-- Hypertonic dysfunctional labor generally can be expected to:
A) cause little pain
B) occur in the active phase of labor
C) react favorably to oxytocin stimulation
D) respond to sedation
15-- Precipitate labor, which most often occurs in multiparous women, is associated with a greater than normal
incidence of which of the following?
A) Amniotic fluid embolism
B) Fetal hypoxia
C) Fetal cerebral trauma
D) Cervical laceration
E) All of the above
16-Dysfunctional labour is:
A) More common in parous women
B) Defined as protracted or arrested labour
C) Requiring early hospitalization
D) Normal
(17)With hypotonic inertia, the following is true EXCEPT:
A) More common in primigravida, esp. elderly.
B) There are painful repeated contractions, but not effective.
C) There are prolonged latent &/or active phase.
D) Secondary type may be a protective mechanism.
(18)In hypotonic inertia, the following can be done EXCEPT:
A) Exclude cephalopelvic disproportion or mal presentation.
B) I.V. hyoscene.
C) I.V. oxytocin (unless contraindicated).
D) Artificial rupture of membranes.
(19)In hypertonic inertia, the following is true, EXCEPT:
A) Repeated painful contraction with pain preceding & outlasting the contraction.
B) The uterus is lax at palpation.
C) Tardy cervical dilatation.
D) Early rupture of membranes.

173

Gyna & Obstetrics Reveision by/Dr.adool

2008

(20)Management of hypertonic inertia needs the following. EXCEPT:


A) Exclusion of disproportion &/ or mal presentation.
B) Alleviation of pain.
C) Oxytocin drip.
D) Caesarean section.
(21)Constriction ring is:
A) Commoner in primigravida.
B) Restricted to the junction between upper & lower uterine segments.
C) Can be diagnosed by abdominal examination.
D) Always there is maternal & / or fetal distress.
(22)In pathological retraction ring: All is true EXCEPT:
A) More common in multiparous women.
B) Can be felt vaginally.
C) Appears as a visible & palpable groove across the abdomen.
D) Needs caesarean section with lower segment vertical incision.
(23)Precipitate labour: All is true EXCEPT:
A) The whole process of labor occurs within four hours.
B) May precede or follow congenital prolapse.
C) Shows arrest of the active phase.
D) Accompnied with many fetal &/ or maternal injures.
(24) Cases of precipitate labour are, commonly, seen after 2nd or third stage. Care provided includes the following,
EXCEPT:
A) Examination of mother's birth canal.
B) Examination of the newborn infant.
C) Retodrine I.V.
D) Prophylactic antibiotics.
(25)In cervical dystocia, the more common cervical injury is:
A) Laceration at 6-o'clock.
B) Laceration at 3-o'clock.
C) Laceration at 9-o'clock.
D) Annular detachment.
(26)Least bleeding occurs with:
A) Cervical laceration at 3-o'clock.
B) Cervical laceration at 6-o'clock.
C) Cervical laceration at 9-o'clock.
D) Annular detachment of the cervix.
(27)Cervical dystocia may follow:
A) Fothergill's operation.
B) Repeated shirodkar's or McDonald's cerclage.
C) Conization of cervix.
D) All of the above.

174

Gyna & Obstetrics Reveision by/Dr.adool

2008

(5) * Contracted pelvis


1-As regard contracted pelvis, these are correct, except:
A. Multiparity is a risk factor is common.
B. Fetal distress is not a common feature.
C. Fetal and maternal injuries are liable.
D. C.S. is the rule.
2-Causes of contracted pelvis include:
A. Android pelvis.
B. Some cases of fracture pelvis.
C. Low assimilation pelvis.
D. Some cases of poliomyelitis.
E. Small gynecoid pelvis.
3-Assessment of pelvic capacity:
A. Obstetric history is not a reliable indicator of pelvic capacity.
B. History suggestive of rickets in childhood is commonly obtained with cases
of contracted pelvis.
C. The pelvic capacity is directly related to the woman's height.
D. The diagonal conjugate reflects the capacity of the inlet.
E. Palpation of the sacrum helps in assessment of the pelvic cavity.
4-Assessment of pelvic capacity:
A. A history of difficult forceps delivery may point to reduced pelvic capacity.
B. An abnormal gait may point to reduced pelvic capacity.
C. External pelvimetry of the inlet may replace cephalo-pelvic disproportion tests.
D. External pelvimetry of the outlet is of very limited use in modern obstetrics.
E. Internal pelvimetry involves assessment of the diagonal conjugate.
5-Assessment of the pelvic outlet:
A. The sub pubic angle is estimated by palpation of the ischio-pubic rami.
B. The bituberous diameter should include the 5 knuckles of the closed fist.
C. The bituberous diameter cannot be measured by the pelvimeter.
D. The sum of the bituberous and anterior sagittal diameters should exceed 15 cm.
E. Palpation of the ischial spines.
6-Internal pelvimetry involves:
A. Assessment of the diagonal conjugate.
B. Assessment of the oblique diameter.
C. Palpation of the sacrum.
D. Palpation of the side walls of the pelvis.
E. Palpation of the sub pubic angle.
7-Internal pelvimetry involves:
A. Assessment of the antero-posterior diameter of the outlet.
B. Assessment of the diagonal conjugate and thereby, calculating the true conjugate.
C. Estimating the width of the sacro-sciatic notch.
D. Palpation of the ischial spines.
E. Palpation of the ilio-pectineal eminence.

175

Gyna & Obstetrics Reveision by/Dr.adool

2008

8-The use of radiological pelvimetry in modern obstetrics is limited to some cases with:
A. Face presentation.
B. Preterm breech.
C. Previous cesarean section.
D. Fracture pelvis.
E. Hydrocephalus.
9-Radiological pelvimetry:
A. The lateral view shows the antero-posterior diameter of the inlet.
B. The lateral view shows the antero-posterior diameter of the outlet.
C. The inlet view is the most informative.
D. The outlet view shows the sub pubic angle.
E. The outlet view shows the bituberous diameter.
10-Cephalo-pelvic disproportion tests:
A. The fetal head is the best pelvimeter for the pelvis.
B. Pinard's method does not involve PV examination.
C. In Muller Kerr method, the thumb is put on the symphysis pubis.
D. If the head can be pushed into the pelvis, there is no disproportion.
E. Moderate degree of disproportion is diagnosed when the head is at the level of
the posterior surface of the symphysis pubis.
11-Management of labor in contracted pelvis:
A. Minor degree of disproportion requires a trial of labor.
B. Cesarean section is indicated if the case is associated with placenta previa.
C. Moderate degree of disproportion requires a cesarean section if trial of labor
is contraindicated.
D. Marked degree of disproportion requires a cesarean section.
E. Marked degree of disproportion requires craniotomy if the fetus is dead.
12-Trial of labor:
A. Is a test of the ability of uterine contractions to achieve vaginal delivery.
B. Is indicated in some cases of mild disproportion.
C. The presentation should be cephalic.
D. Requires repeated PV examinations.
E. May be conducted at home.
13-Trial of labor:
A. Is contraindicated in a primigravida.
B. Mild degree of outlet contraction is allowed.
C. The patient should be left in the second stage for 2 hours.
D. With successful trial, the head becomes engaged.
E. Is beneficial for some cases of post term.
14-Indications of cesarean section in cases with contracted pelvis:
A. Contracted outlet.
B. Marked degree of contracted pelvis if trial of labor fails.
C. Moderate degree of contracted pelvis if trial of labor is contraindicated.
D. Contracted pelvis in a multipara.
E. Mild degree of contracted pelvis associated with a malpresentation.

176

Gyna & Obstetrics Reveision by/Dr.adool

2008

15-Contracted outlet:
A. May be associated with rachitic pelvis.
B. May be associated with android pelvis.
C. May be associated with osteomalacia.
D. Extreme flexion and moulding occur at the outlet.
E. Is not associated with increased incidence of perineal lacerations.
16-Causes of contracted pelvis include the following:
1- Rickets.
2- Osteomalacia
3- Fractures of the pelvis.
4- T.B arthritis of the pelvic joints.
5- Osteomyelitis.
17-Which of the following disease or deformity of the lower limbs affects the pelvic capacity:
1- Poliomyelitis.
2- Hip joint disease.
3- Fracture femur.
4- Varicose veins.
5- Leg ulcer.
18-The following are complications of contracted pelvis during pregnancy:
1- Pyelonephritis.
2- Non engagement of the presenting part.
3- Mal presentation.
4- Spontaneous abortion.
5- Ectopic pregnancy.
19- Examination of the pelvis reveals prominent ischial spines, a narrow subpubic arch, a narrow high-arched
sacroscialic notch, a straight sacrum, and a shortened posterior sagittal diameter, select the pelvic type with
which it is most likely to be associated
A) Anthropoid
B) Android
C) Gynecoid
D) Platypelloid
E) None of the above
20- Examination of the pelvis reveals a wide subpubic arch, a curved sacrum, and a shortened anteroposterior
diameter, select the pelvic type with which it is most likely to be associated
A) Anthropoid
B) Android
C) Gynecoid
D) Platypelloid
E) None of the above
21- Examination of the pelvis reveals a lengthened anteroposterior diameter, a large sacrosciatic notch, prominent
ischial spines, and a straight, posteriorly inclined sacrum, select the pelvic type with which it is most likely to be
associated
A) Anthropoid
B) Android
C) Gynecoid
D) Platypelloid
E) None of the above

177

Gyna & Obstetrics Reveision by/Dr.adool

2008

(22)Contracted pelvis is, obstetrically, defined as:


A) Pelvis in which one, or more of its diameters are reduced by 1 cm or more.
B) Pelvis in which one, or more of its diameters is so much reduced that it
interferes with the normal
mechanism of labour.
C) All of the above.
(23)Causes of contracted pelvis include the following, EXCEPT :
A) Development causes
B) Osteomalacia
C) Upper dorsal kyphosis
D) Poliomyelities of one lower limb
(24)The following stigmata point to contracted pelvis ,EXCEPT :
A) High > 150 cm
B) Limping
C) Spine deformities
D) Dystocia dystrophia syndrome.
(25)Contracted pelvis is suspected if one of the following is found during abdominal examination , EXCEPT:
A) Transverse lie in a primigravida
B) Pendulous abdomen in a multigravide
C) Non- engagement in the last two weeks of pregnancy
D) Scar of previous caesarean .
(26)Pelvimetry, which is obstetriceally significant is:
A) External pelvimetry
B) Internal pelvimetry
C) Outlet pelvimetry
D) (A+ B)
E) (B+C)
(27)Measurement of the diagonal conjugate, points to the length of :
A) Inlet's true conjugate
B) Antero- posterior diameter of the mid cavity
C) Antero- posterior diameter of the plane of least pelvic dimensions.
(28)Pelvic surface of sacrum, if found straight with an acute bend at its lower end, is indicative for:
A) Normal pelvis
B) Simple flat pelvis
C) Rachetic flat pelvis
D) Anthropoid pelvis
(29)Ultrasonography is NOT used for:
A) Measurements of faetal head diameters
B) Measurements of the bony pelvis
C) Estimation of faetal weight
D) Prediction of delivery date
(30)The best pelvimeter is :
A) Thoms pelvimeter
B) Faetal head

178

Gyna & Obstetrics Reveision by/Dr.adool

2008

C) Radiologic pelvimetry
D) Ultrasonic pelvimetry
(31)Tests used to detect cephalopelvic disproportion include the following, EXCEPT:
A) Pinards test
B) Internal pelvimetry
C) Mullers test
D) Muller- Kerrs test .
(32)Obstetric outlet measurement include the following EXCEPT :
A) Subpubic angle
B) Diagonal conjugate dimeter
C) Thoms dictum
D) Four knuckles inserted between ischial tuberosities .
(33)Outlet contraction is found in the following cases, EXCEPT :
A) Android pelvis
B) Anthropoid pelvis
C) Rachetic flat pelvis
D) Spondylolisthesis.
(34)Asynclitism may be encountered in:
A) Android pelvis.
B) Anthropoid pelvis
C) Flat pelvis
D) High assimilation pelvis .
(35)The undetermined factors of labour include :
A) Head moulding
B) Yielding of the pelvis
C) Cervical dilatation
D) Uterine force
E) All of the above.
(36)Contraindication to trial of labour include the following, EXCEPT :
A) Yuong primingravida with moderate disproportion
B) Multigravida with bad obesteric history .
C) Any degree of outlet contraction
D) Presence of malpresenttion .
(37)Caesarean section is indicated in cases of contracted pelvis with:
A) Extremely contracted pelvis
B) Marked disproportion
C) Moderate disproportion with contraindication to trial of labour .
D) All the above.
(38)In the cases of flat pelvis , which is MORE RISKY :
A) Simple flat pelvis
B) Rachetic flat pelvis .

179

Gyna & Obstetrics Reveision by/Dr.adool

2008

(39)Forceps delivery is possible in :


A) Cases with marked disproportion
B) Cases with a contraindication to trial of labour
C) Cases with successful trial of labour and head at + 2 station
D) Cases with failed trial of labour .

Say true or false:


abcdefghijklmnopqrstuvwxyz-

Pathological retraction ring occurs in the 1st stage of labor.


Cervical dystocia may end with annular detachment of the cervix.
Cervical dystocia may occur o top of previous cerclage operation.
Hypertonic inertial occurs in irritable primigravidas with malpresentations.
Oxytocin should be used in cases of twins with hypotonic inertia.
Mild degree of contracted pelvis with breech presentation deserve a trial of labor.
1st degree cephalopelvic disproportion is managed by CS
The management of obstructed labor is immediate forceps delivery.
Pathological retraction ring appears as rising transverse groove in the abdomen.
Contraction ring in the lower segment is managed by upper segment CS.
C.I.N with pregnancy is an indication of CS.
Rupture is the most common complication of ovarian tumors with pregnancy
Infection of ovarian tumors occurs most common at time of puerperium.
Most of fibroids with pregnancy are asymptomatic.
The most common ovarian tumor with pregnancy is ovarian fibroma.
Shoulder dystocia occurs due to failure of shoulder delivery at the outlet of the pelvis.
Cephalo-pelvic disproportion test should be done in all pregnancy ladies at 36 weeks.
3rd degree cephalo-pelvic disproportion is managed by elective CS.
Retained after coming head of breech fetus is managed by perforation of behind the mastoid.
The diagnosis of invasive cancer cervix with pregnancy occurs early.
Oxytocine administration can be used in treating hypotonic inertia in grand multipara.
Cases of hypertonic inertial are treated with oxytocin drip.
Cervical dystocia may result from previous cerclage operation.
Dermoid cyst is the commonest ovarian tumor with pregnancy.
Rupture of a surface vein over a subserous fibroid presents with acute abdominal pain.
Myomectomy is better avoided during pregnancy

(6) * Complications of 3rd stage


1-As regard rupture uterus, all the following are correct, except:

180

Gyna & Obstetrics Reveision by/Dr.adool

2008

A. Multiparity is a risk factor.


B. Abruptio placenta is a common differential diagnosis.
C. Preservation of the uterus is difficult to be done.
D. Fetus is always in the peritoneal cavity.
2-Shock is out of proportion to the amount of bleeding in :
a) 1ry postpartum haemorrhage
b) Retained placenta
c) Acute puerperal inversion of uterus
d) Hypofibrinogenemia
3-Complications of the third stage of labor include:
A. Rupture uterus.
B. Postpartum hemorrhage.
C. Puerperal sepsis.
D. Retained placenta.
E. Obstetric shock.
4-Postpartum hemorrhage:
A. Is defined as excessive blood loss during or after the third stage of labor.
B. Is one of the main causes of maternal mortality in developing countries.
C. Is classified into atonic and traumatic.
D. The average amount of blood loss with cesarean section is 500 cc.
E. Lacerations are the main cause.
5-Primary postpartum hemorrhage:
A. Can occur during the first week after delivery.
B. Can occur during the third stage of labor.
C. Is usually due to a coagulation defect.
D. Is commonly due to uterine inertia.
E. May be due to retained parts of the placenta.

181

Gyna & Obstetrics Reveision by/Dr.adool

2008

6-Causes of primary postpartum hemorrhage include:


A. Atony due to prolonged labor.
B. Rupture uterus.
C. Hypofibrinogenemia associated with missed abortion.
D. Atony due to prolonged anesthesia or excessive analgesia.
E. Retained parts of the placenta.
7-Management of primary postpartum hemorrhage includes:
A. Correction of the general condition.
B. Delivery of the placenta is postponed after stoppage of bleeding.
C. Injection of prostaglandins E2 0.25 intramyometrial.
D. Total hysterectomy.
E. Ligation of the internal iliac artery.
8-Manual removal of the placenta:
A. Is done under local anesthesia.
B. May be done in cases of retained placenta.
C. May be done in cases of atonic postpartum hemorrhage.
D. Is the best management in cases of placenta accreta.
E. Is done in all cases of placenta previa.
9-A 30 years old patient presented with moderate to severe post partum hemorrhage, the placenta was still not
delivered, management should include:
A. Correction of the general condition.
B. Monitoring of vital signs and urine output.
C. Crede method to deliver the placenta.
D. Oxytocin drip.
E. Prostaglandin F2 alpha IV.
10-A 36 years old patient, G5 P4 presented with severe post partum hemorrhage, the placenta was already delivered,
management options include:
A. Correction of the general condition.

182

Gyna & Obstetrics Reveision by/Dr.adool

2008

B. Ergometrine IV.
C. Exploration of birth canal.
D. Brandt Andrews method.
E. Hysterectomy.
11-Causes of secondary postpartum hemorrhage include:
A. Uterine atony.
B. Retained part of the placenta.
C. Choriocarcinoma.
D. Vaginal and perineal lacerations.
E. Infected CS wound.
12-Retained placenta:
A. Is defined as failure of delivery of the placenta for one hour after delivery of
the fetus.
B. It complicates 2% of all deliveries.
C. The main cause is atony.
D. Placenta accreta is more common with previous cesarean section.
E. Bleeding is an uncommon presentation.
13-Placenta accreta:
A. Is more common with placenta previa.
B. Is more common with cesarean section scar.
C. The villi commonly reach the peritoneal coat of the uterus.
D. May require hysterectomy.
E. Is easily managed by manual removal of the placenta.

14-Placenta accreta may be associated with:


A. Placenta previa.
B. Uterine scar.

183

Gyna & Obstetrics Reveision by/Dr.adool

2008

C. Placental insufficiency.
D. Post partum hemorrhage.
E. Ante partum hemorrhage.
15-Management of retained placenta:
A. Ergometrine should not be given.
B. The problem is not encountered with cesarean section.
C. Manual removal of the placenta is successful in the majority of cases.
D. Some cases of placenta accreta require hysterectomy.
E. The retained part of the placenta may form a placental polyp or
develop choriocarcinoma.
16-Hypofibrinogenemia and DIC:
A. Fibrinogen level normally decreases during pregnancy.
B. Important causes of hypofibrinogenemia include concealed accidental
hemorrhage and septic shock.
C. The process of DIC is initiated by release of thromboplastin into the
maternal circulation.
D. Bleeding time is prolonged while coagulation time remains within normal range.
E. Wiener test does not require a laboratory.
17-Hypofibrinogenemia and DIC:
A. FDPs are decreased.
B. The main line of treatment is fresh blood transfusion.
C. Fibrinogen is given in a dose of 4-10 mg. IV.
D. Antifibrinolytics are given if there is evidence of decreased fibrinolytic activity.
E. Heparin may be given prophylactically.
18-Acute puerperal inversion:
A. Is commonly iatrogenic.
B. Occurs only when the uterus is lax.

184

Gyna & Obstetrics Reveision by/Dr.adool

2008

C. The placenta should be removed before reposition of the uterus.


D. Commonly requires hysterectomy.
E. May be associated with shock.
19-Shock in obstetrics:
A. Is usually of the surgical type.
B. Is commonly associated with vagal stimulation.
C. Irreversible shock is due to paralysis of the post capillary sphincter.
D. May occur with apparently normal blood pressure.
E. Neurogenic type is due to failure to maintain the peripheral vascular resistance.
20-Factors contributing to development of shock include:
A. Anemia.
B. Hypervolemia associated w3ith pregnancy.
C. Ante partum hemorrhage.
D. Preeclampsia and eclampsia.
E. Prolonged labor.
21-Clinical picture of hemorrhagic shock:
A. Hypotension.
B. Loss of consciousness.
C. Rapid weak pulse.
D. Pallor.
E. Polyuria.
22-Clinical picture of septic shock:
A. Hypotension.
B. Tachycardia in proportion to fever.
C. Fever.
D. Pallor.
E. Oliguria or anuria.

185

Gyna & Obstetrics Reveision by/Dr.adool

2008

23-Management of hemorrhagic shock:


A. Semi sitting position.
B. Warmth to decrease oxygen requirements.
C. Morphia to relieve irritability.
D. Replacement by crystalloids and blood transfusion.
E. Monitoring of vital signs.
24-Management of hemorrhagic shock:
A. Fowler's position.
B. Oxygen administration to minimize tachypnea.
C. Corticosteroids in small doses.
D. Plasma; to avoid the potential risks of blood transfusion.
E. Monitoring of CVP through a catheter inserted in the IVC.
25-Management of septic shock:
A. Elevation of the patient's legs to increase venous return.
B. Warmth to decrease oxygen requirements.
C. Antibiotics.
D. Blood transfusion.
E. Monitoring of urine output.
26-Indications of blood transfusion in obstetrics:
A. Ante partum or post partum hemorrhage.
B. Hemoglobin level below 9 gm %.
C. Septic shock.
D. Erythroblastosis fetalis.
E. Severe preeclampsia or eclampsia.
27-Potential complications of blood transfusion:
A. Major anaphylactic reaction.
B. Pyrogen reaction.

186

Gyna & Obstetrics Reveision by/Dr.adool

2008

C. Polycythemia.
D. Circulatory overload.
E. Transmission of hepatitis.
28-Amniotic fluid embolism:
A. Is not uncommon.
B. The majority of cases are associated with placental abruption.
C. Mortality reaches up to 30%.
D. Presents as dyspnea, cyanosis, tachypnea, tachycardia and hypotension.
E. Management requires correction of shock and treatment of DIC.
29-Primary postpartum hemorrhage is associated with:
1 - Placenta previa.
2- Polyhydramnios.
3- Forceps delivery.
4- The use of tocolytic agents.
5- Prolonged labor.
30-In the absence of an obvious vaginal hemorrhage, postpartum collapse may be due to:
1- A ruptured uterus.
2- Uterine inversion.
3- Amniotic fluid embolism.
4- A paravaginal hematoma.
5- Eclampsia.
31-Amniotic fluid embolism:
1 - Causes cyanosis.
2- Commonly caused by accidental hemorrhage.
3- Is complicated by disseminated intravascular coagulopathy.
4- Is a complication of the puerperium.
5- Commonly caused by placenta previa.
32-Coagulation failure is an important complication of:

187

Gyna & Obstetrics Reveision by/Dr.adool

2008

1 - Amniotic fluid embolus.


2- Gram-negative septicemia.
3- Abruptio placenta.
4- Placenta previa.
5- Ruptured ovarian cyst with pregnancy.
33- Primary postpartum hemorrhage, all are true EXCEPT:
A) Is defined as a blood loss of greater than 500 ml within the first 48 hours after delivery
B) Uterine atony is the commonest cause
C) Is associated with uterine fibroids
D) Is associated with cervical, vaginal or perineal tears
34- Placenta accreta, the following are false EXCEPT:
A)
B)
C)
D)
E)

Is associated with a previous caesarean section scar


Is characterized by profuse bleeding immediately after delivery of the fetus
Should be removed 'piecemeal' under general anesthesia
There is always penetration of the serosa by chorlonic villi
Is an etiological! factor in amniotic fluid embolism

35- Postpartum hemorrhage , all are TRUE EXCEPT :


A)

is defined as genital tract bleeding in excess of 500 ml from delivery of the fetus until six weeks after
the birth
B) is associated with placenta previa
C) is associated with placental abruption
D) is associated with hydramnios
36- Primary post-partum hemorrhage is associated with all EXCEPT:
A) Multiple pregnancy.
B) Retained placenta.
C) Ergometrine administration.
D) Polyhydramnios.
E) Cervical tear
37- Best immediate treatment of severe post-partum hemorrhage after delivery of a complete placenta:
A) IV Ergometrine
B) Blood transfusion
C) Evacuation of uterus without blood transfusion
D) Uterine artery ligation
E) Aortic compression

38-Post partum haemorrhage is defined as:


A) Loss of more than 500mls of blood after vaginal delivery.
B) Loss of more than, 1000mls of blood after caesarian section.
C) Loss of blood that affects the general condition of patient.
D) (A+B).

188

Gyna & Obstetrics Reveision by/Dr.adool

2008

E) (A+B+C).
39-The leading cause of maternal mortality in Egypt is:
A) Antepartum haemorrhage.
B) Postpartum haemorrhage.
C) Purperal sepsis.
D) Abortion- haemorrhage.
40-Placental- site postpartum haemorrhage is NOT caused by:
A) Constriction ring.
B) Atony of uterus.
C) Cervical laceration.
D) Adherent placenta.
41-Dissiminated intravascular clotting, is NOT encountered in:
A) Abruptio placenta.
B) Threatened abortion.
C) Missed abortion left for more than four weeks.
D) Amniotic fluid embolism.
42-Dissiminated intravascular clotting, is NOT diagnosed by:
A) Estimation of fibrinogen in blood.
B) Tube clotting test.
C) Total & differential white count.
D) Estimation of fibrinogen degradation products.
43-Dissiminated intravascular clotting is NOT TREATED by:
A) Fibringen injection.
B) Transfusion of double- strength plasma.
C) Uterine artery ligation/ or embolization.
D) Tranexemic acid (anti plasmin) injection.
44-Ecbolics, used in the managementof atonic postpartum haemorrhage, are the following, EXCEPT:
A) Oxytocin.
B) Methergin.
C) Mifeprestone.
D) Mesoprostol.
E) Prostaglandin F2.
45-In the treatment of postpartum haemorrhage, one of the following is to be discouraged:
A) Brandt- Andrew's technique.
B) Crede's manaeuvre.
C) Manual separation of the placenta.
D) Bimanual compression.
E) Uterine massage.
46-In dealing with the general condition of a case of postpartum haemorrhage, the following should be measured,
EXCEPT:
A) Haematocrit value.
B) Urine output.
C) Sedementation rate.

189

Gyna & Obstetrics Reveision by/Dr.adool

2008

D) Coagulation profile.
E) Liver function tests.
47-In manual separation of the placenta, the right hand is introduced through the vagina to the uterus. There it is:
A) Intrauterine intra- amniotic.
B) Intrauterine extra-amniotic.
C) Not important.
48-In the case of cervical laceration, which statement is NOT TRUE:
A) Lateral laceration may injure the uterine vessels.
B) Annular detatchment is associated with severe bleeding.
C) Bucket- handle tears often occur after intra-amniotic injection of postaglandins.
D) None of the above.
49-Cervical laceration may result in the following,(Mark the WRONG statement):
A) Excessive bleeding.
B) Cervical incompetence.
C) Broad ligament haematoma.
D) Involvement of the lower uterine segment.
E) None of the above.
50-Colporrhexis indicates:
A) Lateral cervical laceration.
B) Injury to the vaginal fornix or fornices.
C) Injury to the perineum with involvement of anal sphinter.
D) Injury to the vulva with involvement of the frenular artery.
51-In cervical laceration, the site affected is MOSTLY at:
A) 3 Oclock.
B) 6 Oclock.
C) 9 Oclock.
D) 12 Oclock.

Say true or false

a. Primary post partum hemorrhage may occur after prolonged labor.


b. Primary post partum hemorrhage, bleeding must to start after delivery of the placenta.
c. In post-partum hemorrhage, the patient may be markedly shocked although there may be a little vagianl bleeding.
d. Evacuation of the bladder is a good prophylaxis against post-partum hemorrhage.
e. In acute inversion of the uterus due to excessive cord traction removal of the placenta is a must before reduction.
f. Pathological retraction can retain the placenta.
g. Total placenta percreta is a common cause of post-partum hemorrhage.
h. Placental polyp removed 2 weeks after delivery should be examined pathologically.
i. Ligation of the uterine artery is the final step before doing hysterectomy for post-partum hemorrhage.
j. Chronic inversion of the uterus may present by infertility.
k. Manual reduction of acute inversion of the uterus usually fails if the placenta is not separated.
l. 1st degree inversion of the uterus is usually diagnosed by inspection.
m. Contraction rig may result into failure of placental separation.
n. Incomplete rupture uterus may result into retained separated placenta.
o. Coagulation defect may result into atonic post-partum hemorrhage.
p. Atonic post-partum hemorrhage may develop in cases of placenta previa due to retained parts or anemia

190

Gyna & Obstetrics Reveision by/Dr.adool

2008

q.
r.
s.
t.
u.
v.
w.
x.
y.
z.

from ante-partum hemorrhage.


Placenta acreta develop due to defective layer between decidu and myometrium.
Routine administration of ecbolics may result into decrease the incidence of atonic post-partum hemorrhage.
Manual separation of the placenta may result into perforation of the uterus.
Crede maneuver has a high success rate especially with co-perativwe patients how relax their abdominal wall muscles.
Retained placenta may result into shock state even in absence of vaginal bleeding.
Cases of atonic pos-partum hemorrhage, bleeding is usually proportionate to the amount of vaginal bleeding.
Augmentation of labor may result into atony if oxytocin is stopped in post-partum period.
Cases of chronic inversion of the uterus may be due to Obstetrics or Gynecological causes.
The best management of total placenta accrete is piece meal cutting of the placenta.
Gel foam embolization of the common iliac arteries is recently tried to manage cases of atonic post-partum hemorrhage.

(7) * Fetal well being


(1)Evaluation of the state of the newborn is done by:
A) Bishop score.
B) Apgar score.
C) Zatuchni Andro's score.
D) Cervical score.
(2)Apgar score contains the following items EXCEPT:
A) Pulse rate.
B) Respiratory effort.
C) Skin colour.
D) Body weight.
E) Grimace.
(3)In relation to Apgar score the following is true, EXCEPT:
A) Score at 1 minute indicates the need for immediate resuscitation.
B) Score at 5 minutes indicates the effectiveness of resuscitation.
C) It correlates well with the cause of asphyxia .
D) It correlates poorly with the results.
(4)The most important diagnostic & prognostic of the five items of Apgar score is:
A) Heart rate.
B) Respiratory effort.
C) Skin color.
D) Muscle tone.
E) Grimace.
(5)Active resuscitation of the newborn include the following, EXCEPT:
A) Clearing of air passages.
B) Oxygenation.
C) Endotracheal intubation.
D) Treatment of alkalosis.

191

Gyna & Obstetrics Reveision by/Dr.adool

2008

E) Drugs e.g. nalorphine, epinephrine, antibiotics.


6-Methods of assessment of fetal well being include:
A. Daily fetal movement count.
B. Ultrasonography.
C. Non stress test.
D. Chorion villus sampling.
E. Doppler.
7- Indicators of fetal distress include ALL EXCEPT:
meconium stained liquor
fetal tachycardia
fetal bradycardia
maternal acidosis
8- Fetal scalp pH , all INCORRECT EXCEPT:
A) a result of 7.28 is normal
B) accurately determines the condition of the baby
C) carries no risk
D) cannot be performed in breech presentation
9- Decreased fetal heart variability is due to:
A) Fetal anencephaly
B) Maternal sleep
C) Spinal anesthesia
D) Post dates fetus
10- Early decelerations can be due to one of the following:
A) maternal hypotention
B) cord compression
C) vagal.
D) None of the above

11- Late decelerations seen in one of the following:


A) Pressure on Fetal head
B) CPD
C) Cord Compression
D) Utero Placental insufficiency
E) Placenta praevia
12- During labor, deceleration of the fetal heart rate correlates most closely
with which of the following fetal values?
A) Arterial pH
B) Arterial PO2
C) Arterial PCO2
D) Central venous pressure
E) Venous potassium levels
13- The pH of a fetal scalp sample is abnormal if it is less than:

192

Gyna & Obstetrics Reveision by/Dr.adool

2008

A) 7.25
B) 7.30
C) 7.35
D) 7.40
E) 7-45
14- Late deceleration patterns on a fetal monitoring strip represent:
A) cord compression
B) pulmonary immaturity
C) fetal hypoxia secondary to decreased perfusion of the intervillous spaces
D) congenital cardiac conduction defects
E) entry of the fetal head into the pelvic brim
15- Fetal monitoring, all are not true EXCEPT:
A) Gives a reliable indication of the state of the fetus at birth
B) An abnormal fetal heart trace in the second stage is always associated with low Apgar scores
C) A good correlation exists between l-minute Apgar scores and acid base measurements
D) Cardiotocography (CTG) has a low false negative rate
E) Fetal distress is the commonest indication for caesarean section
16- Meconium in the amniotic fluid, all are not true EXCEPT:
A) The incidence of fetal distress is similar whether the meconium is thick or thin
B) Is not seen prior to 24 weeks gestation
C) Is more commonly seen in the preterm labour than in a full-term labour
D) Is a normal finding during examination of breech presentation.
E) Syntocinon during labour is contraindicated
17- Which of the following would NOT be a treatment for a non-reassuring FHR pattern:
A) Initiate oxytocin
B) Acoustic or scalp stimulation
C) Amnioinfusion
D) Plan immediate delivery

(8) * Fetal & Neonatal asphyxia


1-Fetal asphyxia:
A. Is a state of inadequate oxygenation.
B. Is associated with inadequate elimination of carbon dioxide.
C. Is always associated with acidosis.
D. Is best diagnosed by umbilical blood sampling.
E. Intra uterine asphyxia ends with delivery of the fetus.
2-Fetal asphyxia:
A. Always requires an immediate cesarean section.
B. Passage of meconium always indicates fetal asphyxia.
C. Increased fetal activity may indicate asphyxia.
D. Variable deceleration is not associated with asphyxia.
E. Fetal scalp blood sampling is a sure evidence of asphyxia.
3-Causes of intra uterine asphyxia:

193

Gyna & Obstetrics Reveision by/Dr.adool

2008

A. Maternal heart failure.


B. During an eclamptic fit.
C. Placental compression or separation.
D. False knot of the cord.
E. Polyhydramnios.
4-Causes of intra uterine asphyxia:
A. Placental insufficiency.
B. True knot of the cord.
C. Intracranial hemorrhage.
D. Tight coils of the cord.
E. Maternal diabetes.
5-Fetal heart rate patterns that may occur with asphyxia:
A. Bradycardia.
B. Beat to beat variability.
C. Tachycardia.
D. Variable deceleration.
E. Early deceleration.
6-Abnormal fetal heart rate patterns that may occur with asphyxia:
A. Can be detected only by electronic monitoring.
B. Bradycardia does not always indicate asphyxia.
C. Variable deceleration indicates cord compression.
D. Late deceleration is a reliable indicator of asphyxia.
E. Loss of beat to beat variability may occur with use of ritodrine.
7-Asphyxia neonatorum:
A. May be due to persistence of intra uterine asphyxia.
B. Is diagnosed with a low bishop score.
C. Always reflects the quality of management of delivery.
D. May result in cerebral palsy and mental retardation.
E. May cause pneumonia.
8-Causes of asphyxia neonatorum:
A. Obstruction of air passages.
B. Rh incompatibility.
C. Depression of respiratory center by oxytocin.
D. Intra cranial hemorrhage.
E. Congenital anomalies and prematurity are significant predisposing factors.
9-Apgar score:
A. Is only evaluated in cases of asphyxia.
B. Is devaluated at 1 minute post delivery to determine the need for resuscitation.
C. Is devaluated at 15 minutes post delivery to determine the efficiency
of resuscitation.
D. Is devaluated at 10 minutes post delivery to determine the long term prognosis.
E. A score of 4 indicates severe asphyxia.
10-Apgar score:
A. A score of 1 is assigned to pink color.

194

Gyna & Obstetrics Reveision by/Dr.adool

2008

B. A score of 0 is assigned to a heart rate below 100 beats per minute..


C. A score of 2 is assigned to a cough or sneeze reflex.
D. A score of 1 is assigned to muscle limp.
E. A score of 2 is assigned to good cry.
11-Management of asphyxia neonatorum:
A. Proper management of cases of intra uterine asphyxia.
B. Meperidine should not be given within 3 hours of anticipated delivery.
C. Endotracheal intubation is required with Apgar score less than 5.
D. Warming is essential to minimize oxygen requirements.
E. Sodium bicarbonate is required in all cases.
12-Management of asphyxia neonatorum:
A. Avoiding intra cranial hemorrhage.
B. Warming is essential to avoid attacks of apnea.
C. Cardiac resuscitation is required with heart rate less than 100 beats per minute.
D. Air passages should be cleared within 3 minutes of delivery.
E. Nalorphine is required in all cases.

(9) * Fetal birth injuries


1-The followings are characters of Cephalohematoma, except:
A. Appears after delivery.
B. Limited to the borders of one cranial bone.
C. Persist for few weeks.
D. Soft and pits under pressure.
2-Erbs palsy include the followings, except:
A. The upper limp at the side of the trunk.
B. Internally rotated.
C. The wrist is flexed.
D. Weakness of the flexors of the wrist and fingers.
3-Fetal birth injuries:
A. Fracture skull is commonly associated with cephalhematoma.
B. Sternomastoid injury occurs with excessive traction on the neck.
C. Vertebral column injuries are most common with transverse lie.
D. Fractures and dislocations of long bones are most common with vaginal
breech delivery.
E. Facial palsy most commonly occurs with face presentation.
4-Fetal birth injuries:
A. Do not occur with cesarean section.
B. Facial palsy commonly turns into facial paralysis.
C. Brachial plexus injury occurs with excessive traction on the neck.
D. Complete recovery usually occurs with injuries of abdominal viscera.
E. Pelvic caput is the second common type of fetal birth injuries.
5-Fracture of the fetal skull:

195

Gyna & Obstetrics Reveision by/Dr.adool

2008

A. Is commonly related to difficult delivery.


B. Is sometimes due to precipitate labor.
C. Requires a CT scan.
D. A linear fracture may be associated with cephalhematoma.
E. The outcome of fracture base is usually favorable.
6-Caput succedaneum:
A. Is edema of the subcutaneous tissue of the fetal scalp.
B. Disappears 2-3 hours after delivery.
C. A cervical caput may be associated with rupture of membranes before full
cervical dilatation.
D. A pelvic caput is usually smaller than a cervical caput.
E. An artificial caput should not overlie the occiput.
7-Caput succedaneum:
A. A cervical caput is commonly associated with excessive moulding.
B. A pelvic caput may give a false impression about the degree of flexion.
C. An artificial caput results from use of forceps for delivery.
D. A cervical caput is commonly associated with intra cranial hemorrhage.
E. Caput should be differentiated from cephalhematoma.
8-Cephalhematoma:
A. Is collection of blood under the skull.
B. Appears at delivery.
C. Is limited to the borders of one cranial bone.
D. Persists for few days.
E. Requires a CT scan.
9-Cephalhematoma:
A. May be caused by forceps delivery.
B. May occur with delivery using the ventouse.
C. May cause jaundice in the newborn.
D. May cause neonatal hypoglycemia.
E. Should be evacuated.
10- Ritrodrine can cause one of the following:
A) bradycardia
B) heart block
C) hypotension
D) peripheral vasoconstriction

11- Which phrase best defines a teratogen?


A) a life-support system that protects the fetus
B) an agent that stimulates the formation of organs
C) an abnormality in infants of alcoholic mothers
D) an environmental factor that produces birth defects
12- A common characteristic of babies born to women who smoke during their pregnancies is:

196

Gyna & Obstetrics Reveision by/Dr.adool

2008

A) a missing arm or leg


B) facial deformities and below-average intelligence
C) restlessness and irritability

D) lower birthweights
13- The Apgar primarily assesses a newborn's:
A) psychological status
B) reflexes
C) physiological health
D) responsivity to people
14- With regard to the Apgar Score, all are not true EXCEPT:
A) heart rate < 100 beats per minute scores 0
B) grimace in response to stimulation scores 2
C) some flexion of extremities scores 1
D) gasping, weak cry scores 0
E) is normally recorded at 1 and 10 minutes
15- The following factors positively influence high birth weight:
A) Maternal growth hormones.
B) Prolonged pregnancy (294) days.
C) Fetal hyperinsulinaemia.
D) Primiparity
E) Social class
16- Birthweight > 4.5 kg, all are true EXCEPT:
A) is always due to poorly controlled diabetes.
B) increases the risk of shoulder dystocia.
C) is a contra-indication to vaginal delivery of a baby presenting by the breech.
D) another large baby is likely in a subsequent pregnancy.
17- Cervical Incompetence, all are true EXCEPT:
A) usually manifests itself in late miscarriage
B) is treated with 2nd. trimester cervical cerclage
C) usually causes ante-partum hemorrhage
D)may present with premature rupture of membranes
18- In circlage operation stitch should be removed:
A) after delivery
B) at labour
C) at 38 weeks
D) at 28 weeks.
19- The following are recognized risk factors for preterm labour EXCEPT:
A) Maternal employment
B) Genital infection
C) Low pre-pregnancy maternal weight
D) Smoking
20- The following regimens will delay delivery in preterm labour EXCEPT:

197

Gyna & Obstetrics Reveision by/Dr.adool

2008

A)
B)
C)
D)

IV ritodrine
Progesterone supplementation
Nifedipine
Tndomethacin

21- The following are absolute contraindications to ritodrine EXCEPT:


A) Cervical dilatation of 4 cm
B) Ruptured membranes
C) Hyperthyroidism
D) Insulin-dependent diabetes
22- Signs of fetal postmaturity that may be noted prior to delivery include one of the following:
A) oligohydramnios
B) maternal edema
C) clear amniotic fluid
D) maternal weight loss
23- The postmaturity syndromethe following do not apply EXCEPT:
A) Increased incidence of placental abruption
B) Meconium staining of membranes
C) Body weight increased in relation to body length
D) Fetal macrosomia
24- Of the conditions listed below, which one most accurately describes a risk for preterm labour and birth:
A) Caucasian race
B) Maternal age between 20 - 25
C) Bicornuate uterus
D) One previous first-trimester miscarriage
E) Singleton pregnancy
25- Bells palsy is dysfunction of
A) Mandibular nerve
B) Maxillary nerve
C) Facial nerve
D) Ophthalmic nerve
26- Match the following tocolytic agents with their mechanism of action.
1) Nifedipine
2) Terbutaline
3) Ethanol
4) Indomethacin
5) Magnesium sulfate
A) Suppresses synthesis of endogenous prostaglandins
B) Inhibits oxytocin release
C) Stimulates cyclic AMP formation
D) Inhibits calcium influx
27- History of preterm births may indicate all of the following EXCEPT:
A) DES exposure
B) Uterine anomaly
C) Incompetent cervix

198

Gyna & Obstetrics Reveision by/Dr.adool

2008

D) Previous breech delivery


28- The predominant fetal pulmonary surfactant is:
A) Sphingomyelin
B) Phosphatidylcholine
C) Phosphatidylserine
D) Phosphatidylinositol
E) Steroids

29- Maternal administration of steroids:


A) Decreases the incidence and severity of hyaline membrane disease
B) Is effective only in female fetuses
C) Exerts its maximum effect after 32 weeks
D) Carries unacceptable neonatal complications
30- Management of the neonate with a heart rate between 60 and 90 beats per minute (bpm) should include which of
the following?
A) Rubbing infant's back with a drying towel
B) Placing infant under radiant heat source
C) Suctioning the oropharynx and nose
D) All of the above
31- The obstetrician encountering an infant suspected of having meconium aspiration should do all of the following
except:
A) Suction the oropharynx and nasopharynx immediately upon delivery of the head
B) Insert a laryngoscope, irrespective of whether the baby is actively breathing
C) Check the airway before inserting positive-pressure ventilation to avoid forcing meconium
D) Complete delivery by cesarean section if that is mode of delivery, then suction the oropharynx and nasopharynx
32- Which of the following characterizes intraventricular hemorrhage?
A) Occurs in 75% of infants weighing 1,000-1,800 g
B) Occurs universally in infants under 28 weeks of gestation
C) Most often originates in the subependymal layer of the peri ventricular matrix that overlies the head of the
caudate nucleus
D) Most commonly is associated with meconium aspiration
33- Erb-Duchenne upper arm palsy is due to injury of which cervical nerves?
A) 1 and 2
B) 3 and 4
C) 5 and 6
D) 7 and 8
34- All of the following statements characterize hyaline membrane disease (HMD) except:
A) Elevated respiratory rate (> 60/min)
B) Expiratory grunting
C) Nearly 100% incidence below 29-30 weeks
D) Amelioration of acute HMD by intratracheal administrations of modified bovine surfactant
35- Of neonatal deaths not attributed to congenital malformations, what percentage result from preterm delivery?
A) 40%

199

Gyna & Obstetrics Reveision by/Dr.adool

2008

B) 50%
C) 75%
D) 90%
36- Very low-birth-weight refers to infants weighing less than:
A) 750 g
B) 1000g
C) 1.500 g
D) 2,000 g

37- All of the following factors have been consistently associated with preterm delivery except:
A) Previous first-trimester abortion
B) Low socioeconomic status
C) Non white race
D) Smoking
E) l.ow prepregnancy weight
38- A patient who has had a prior preterm birth presents for counseling. You should tell her that her risk of having a
second preitrm delivery is approximately:
A) 5%-10%
B) 20%-40%
C) 50%-75%
D) Over 75%
39- Which of the following uterine abnormalities is associated with the highest incidence of preterm delivery?
A) Unicornuate
B) Complete bicornuate
C) Arcuate
D) Complete septum
40- All of the following have been associated with an increased risk of preterm delivery in a current pregnancy except:
A) Placenta previa
B) Oligohydramnios
C) Hyperemesis gravidarum
D) First -trimester bleeding
E) Multiple gestation
41- Maternal complications of parenteral -mimetic tocolytic therapy include:
A) Hyperglycemia
B) Hypokalemia
C) Chest pain
D) Hypotension
E) All of the above
42- Which of the following statements correctly describes the use of magnesium sulfate as a tocolytic agent?
A) Magnesium sulfate is the preferred tocolytic in patients with myasthenia gravis
B) Serum magnesium levels correlate well with tocolytic effect
C) Pulmonary edema has not been reported in patients receiving magnesium sulfate tocolysis
D) Magnesium sulfate is the preferred tocolytic in patients with diabetes mellitus

200

Gyna & Obstetrics Reveision by/Dr.adool

2008

43- Which of the following is an absolute contraindication to the administration of -mimetic tocolytic agents?
A) Chorioamnionitis
B) Maternal chronic hypertension
C) Maternal history of severe migraine headaches
D) Multiple gestation
44- All of the following have been observed in association with indomethacin tocolysis except:
A) Oligohydramnios
B) Pulmonary hypertension in the neonate
C) Constriction of the ductus arteriosus
D) Fetal growth retardation
45- Fetal and neonatal complications associated with premature rupture of the membranes (PROM) include:
A) Neonatal sepsis
B) Pulmonary hypoplasia
C) Cord prolapse
D) Skeletal deformities
E) All of the above
46- Characteristics of infants with the postmaturity syndrome include all of the following except:
A) Skin maceration
B) Brownish green disco lor a lion of the skin
C) Increased subcutaneous fat deposits
D) Failure of intrauterine growth
47- Prolonged pregnancy is associated with an increased incidence of all of the following except:
A) Fetal hypoxia
B) Macrosomia
C) Meconium staining
D) Polyhydraninios
48- If the last menstrual period was withdrawal bleeding following oral contraceptives. the expected date of
confinement should be calculated later by at least:
A) 2 weeks
B) 4 weeks
C) 6 weeks
D) 8 weeks
49- Which of the following techniques for anteparturm fetal surveillance has been associated with the lowest falsenegative rate in the prolonged pregnancy?
A) Nonstress test
B) Biophysical profile
C) Contraction stress test
D) Maternal assessment of fetal activity
50- All of the following tests are useful in antepartum surveillance of prolonged pregnancy except:
A) Biophysical profile
B) Nonstress test
C) Contraction stress test
D) Amniocentesis for meconium
51- A post-date or post-term pregnancy is defined as one which has reached:
A) 40 weeks of gestation

201

Gyna & Obstetrics Reveision by/Dr.adool

2008

B) 41 weeks of gestation
C) 42 weeks of gestation
D) 43 weeks of gestation
52- In the prolonged pregnancy, the primary maternal risk is associated with:
A) Preeclampsia
B) Cesarean section
C) Gestational diabetes
D) Excessive weight gain
53- The possibility of RDS would be most reliably predicted by:
A) U/S measurement of BPD
B) U/S measurement of femur length
C) Creatinine concentration in amniotic fluid > 2
D) L/S ratio in amniotic fluid < 2
E) Decrease of bilirubin in amniotic fluid
54-Intracranial hemorrhage:
A. Is sometimes due to intrauterine asphyxia.
B. Is more common with preterm labor.
C. The intra-ventricular type is related to traumatic delivery.
D. With intra-cerebral hemorrhage, the anterior fontanelle is tense and bulging.
E. Magnesium sulfate is sometimes required to prevent convulsions.
55-Intracranial hemorrhage:
A. CT is essential in diagnosis and follow up.
B. Ultrasonography is essential in diagnosis and follow up.
C. Lumbar puncture may be used for diagnosis.
D. Lumbar puncture may be a part of the treatment.
E. Vitamin k 10 mg IM is given to improve coagulation.
56-Intracranial hemorrhage:
A. The majority of etiological factors are preventable.
B. Commonly causes asphyxia.
C. Neurological manifestations are uncommon.
D. Breast milk should be evacuated and given to the newborn by a dropper.
E. Antibiotics are given to prevent pulmonary infection.
57-Etiological factors of intracranial hemorrhage:
A. Rapid compression and decompression of the fetal head.
B. Cervical caput.
C. Moulding.
D. Excessive compression of the fetal head.
E. Rh incompatibility.
58-Sites of intracranial hemorrhage:
A. Subdural.
B. Subperiosteal.
C. Subarachnoid.
D. Intra-dural.
E. Intracerebral.

202

Gyna & Obstetrics Reveision by/Dr.adool

2008

59-Shortly after vaginal delivery of an infant in an occiput posterior position, a vaguely demarcated edematous area
over the midline of the skull was noted. This observed lesion is most likely:
a) A cephalhematoma.
b) A skull fracture.
c) Caput succedaneum.
d) A subdural hematoma.
e) Subgaleal hematoma.
60- Cut-off value of cervical length at 24 weeks of gestation for prediction of preterm delivery is:
A) 0.5cm.
B) 1.5cm
C) 2.5cm
D) 3.5cm
70- Neonatal intraventricular hemorrhage, all are TRUE EXCEPT:
A) is associated with prematurity
B) The incidence is more in babies delivered by CS
C) can be diagnosed by lumbar puncture
D) may lead to hydrocephalus
71- Respiratory Distress Syndrome, all TRUE EXCEPT:
A) is more common in the offspring of diabetic mothers
B) is due to deficient levels of surfactant
C) artificial surfactant can be administered via an endotracheal tube
D) more common in term than preterm fetus
72- Preterm labour, all TRUE EXCEPT:
A)
B)
C)
D)

is associated with an increased risk of breech presentation.


is associated with uterine anomaly.
is associated with genital tract infection.
tocolytic therapy can always postpone delivery.

73- Preterm delivery , all are TRUE EXCEPT:


A)
B)
C)
D)

More likely in women with previous history of preterm delivery


is more likely with multiple pregnancy.
is more likely if the mother smokes cigarettes.
is defined as the delivery of a baby weighing < 5 lb.

74- Post term pregnancy is more than .. days:


A)
280
B)
287
C)
294
D)
300

203

Gyna & Obstetrics Reveision by/Dr.adool

2008

75- A neonate with asymmetrical intrauterine growth restriction, all are true EXCEPT:
A)
Is more likely to develop hypoglycaemia than a normally grown neonate.
B)
Has increased liver glycogen storage.
C)
Is at increased risk of hypothermia.
D)
Has a body length that is less restricted than its body weight.
76- Causes of neonatal hypoglycaemia include only one of the following:
A) cow milk allergy
B) urea cycle defects
C) glycine sensitivity
D) infants of diabetic mothers
77- Hyaline membrane disease, all are not true EXCEPT:
A) usually occurs within 12 hours of delivery
B) usually occurs after Caeserean section
C) has a better prognosis if steroids are given to the infant
D) is more common in multiple pregnancies
1. Say true or false
a. Post maturity is a common cause of placental insufficiency.
b. Mgso4 is used for tocolytic therapy.
c. R.D.S is common among infants of pre-eclamptic ladies.
d. Infant weighting less than 2.5 Kg are called S.G.A
e. Direct Comb's test is a very effective screening for pregnant RH-ve mothers.
f. Beat to beat variability is very important assessment of fetal wellbeing in utero.
g. Fetal scalp blood sampling is the most informative test for fetal condition in labor.
h. Miscalculation is the final diagnosis for many cases of suspected post-maturity.

(10) * Operative obstetrics


1-External cephalic version is contraindicated in:
A. Preeclampsia.
B. Previous C.S..
C. Before 32 weeks.
D. All of the above.
2-Episiotomy:
A. May be done under either local or general anesthesia.
B. The incision involves the medial fibers of the levator muscle.

204

Gyna & Obstetrics Reveision by/Dr.adool

2008

C. Is not indicated in multiparas.


D. Is usually not needed if the fetus is small or preterm.
E. A generous episiotomy is a large midline episiotomy.
3-Episiotomy:
A. A generous episiotomy is required with narrow sub pubic angle.
B. Repair of episiotomy should take place in separate layers.
C. Is one of the most common causes of hematoma of the vulva.
D. A generous episiotomy is required with malpresentations.
E. Extension to the rectum is more common with midline episiotomy.
3-Cesarean section:
A. Is defined as delivery of a living fetus through an abdominal and uterine incision.
B. The most common indication is previous cesarean section.
C. May be done under either general or epidural anesthesia.
D. A cesarean hysterectomy may be required with placenta accreta or
severe uncontrollable postpartum hemorrhage.
E. Vaginal delivery after cesarean section is allowed in some cases.
4-Classical cesarean section:
A. Is indicated in cases of transverse lie.
B. Is indicated in cases of placenta previa.
C. Is indicated in cases with previous successful repair of vesico-vaginal fistula.
D. Leaves a strong scar.
E. Is performed through a transverse incision in the upper segment.
5-Indications of cesarean section include:
A. Most of cases with previous cesarean section.
B. All cases of contracted pelvis.
C. Most of cases of accidental hemorrhage.
D. Some cases of twin pregnancy.
E. Some cases of uterine inertia.
6-Indications of cesarean section include:
A. All cases with previous cesarean section.
B. Most of cases of placenta previa.
C. Preterm breech.
D. The majority of cases of occipito-posterior.
E. Cases of heart disease with pregnancy.
7-Indications of upper segment cesarean section:
A. Extensive adhesions over the lower segment.
B. Placental implantation over the lower segment.
C. Transverse lie.
D. Previous successful repair of vesico-vaginal fistula.
E. Concealed accidental hemorrhage.
8-Advantages of lower segment over upper segment incision in cesarean section:
A. Less liability to bladder injury.
B. Stronger scar.
C. Less liability to parametritis.

205

Gyna & Obstetrics Reveision by/Dr.adool

2008

D. Less liability to post operative adhesions.


E. Less liability to injury of the uterine vessels.
9-Indications of vertical incision in the lower segment during cesarean section:
A. Varicose veins over the lower segment.
B. Some cases of shoulder presentation.
C. Large fetus or hydrocephalus.
D. Extensive adhesions over the lower segment.
E. Contraction ring.
10-Potential complications of cesarean section include:
A. Post operative thrombosis and pulmonary embolism.
B. Bladder injury.
C. Hot flushes.
D. Development of endometriosis.
E. Adhesions and intestinal obstruction.

11-Which of the following is NOT a characteristic of a mediolateral episiotomy?


a) Difficulty of repair.
b) Common faulty healing.
c) Occasional dyspareunia.
d) Relatively common extension through anal sphincter and into rectum.
e) Occasional faulty anatomic result.
12-Advantage of a median episiotomy include the following EXCEPT:
a) Increased area of vaginal outlet to facilitate delivery.
b) Less blood loss compared to mediolateral technique.
c) Avoidance of major perineal lacerations.
d) Decreased risk of injury to the anal sphincter and mucosa.
e) Greater ease of repair compared to mediolateral technique.
13-The following are associated with a successful external cephalic version EXCEPT:
a) Frank breech.
b) Large amniotic volume.
c) Unengaged fetus.
d) High parity.
e) Use of tocolytics.
14-The following statements regarding instrumental vaginal delivery are correct EXCEPT:
a) It is common in laboring women with epidural anesthesia.
b) With the vacuum extractor, it is too slow to be useful when rapid delivery is required.
c) It may be avoided by the appropriate use of syntocinon in the second stage of labor.
d) It leads to third degree perineal tears more frequently with forceps than with the vacuum extractor.
e) The vacuum extractor is associated with more maternal trauma than the obstetric forceps.
15-Causes for increased CS rate include the following EXCEPT:
a) Dystocia.
b) Breech presentation.
c) Fetal distress.

206

Gyna & Obstetrics Reveision by/Dr.adool

2008

d) Intra-uterine fetal death.


e) Repeat CS.
16-Regarding preoperative preparations for CS, the following statements are true EXCEPT:
a) The bladder must be full to identify its edge during the operation.
b) Adequate counseling and consent is essential.
c) The wound site is prepared with antibacterial wash.
d) Blood is prepared for high risk pregnancies.
e) Anesthetic evaluation is ordered.
17-Regarding anesthesia for CS, the following statements are true EXCEPT:
a) Local anesthesia is used in certain conditions.
b) Maternal position during the procedure is very important.
c) Anesthetic complications are important causes for maternal mortality.
d) Postoperative aspiration is a common cause of maternal mortality.
e) Epidural anesthesia is associated with postoperative aspiration.

18-The following statements regarding cesarean section are true EXCEPT:


a) It is safer than vaginal delivery for the preterm breech.
b) Associated infection is markedly reduced by the use of prophylactic antibiotics.
c) Performed electively under regional anesthesia is as safe for the mother as normal vaginal delivery.
d) It is commonly performed for prolonged labor (dystocia).
e) The lower segment one is safer than the upper segment one.
19-Indications of caesarean section include all the following EXCEPT:
a) Previous classic C.S.
b) Failed forceps delivery.
c) Non reassuring fetal heart rate tracing.
d) McDonald cervical cerclage.
e) Cord prolapse.
20-Criteria to allow vaginal birth after cesarean section (C.S.) include the following:
a) The previous C.S was because of contracted pelvis.
b) The previous C.S was a classic one (upper segment).
c) Oversized fetus in the current pregnancy.
d) Breech presentation in the current pregnancy.
e) The post partum period following the C.S was uneventful.
21-Advantages of lower segment C.S over upper segment C.S. include the following EXCEPT:
a) Less bleeding unless extended.
b) Less paralytic ileus.
c) Used if there are pelvic adhesions.
d) Less incidence of subsequent uterine rupture.
e) Less infection.
22- Elective episiotomy:
is advocated in primigravid patients
is advocated in patients who have previously had episiotomy

207

Gyna & Obstetrics Reveision by/Dr.adool

2008

is advocated in patients who have had Caesarean section


is advocated in patients who have had a previous 3rd. degree tear
23- Recognised complications of external cephalic version are all EXCEPT :
A) Fetal bradycardia
B) Chorioamnionitis
C) Placental abruption
D) Premature rupture of the membranes
24- External cephalic version, All are true EXCEPT:
Anti-D should be given to Rhesus negative mothers.
is best done at thirty two weeks.
is more successful if a beta-sympathomimetic is used.
may cause ruptured uterus.
E) reduces the incidence of breech presentation at term.
25- The following STD is an indication of cesarean section:
A) HSV 2
B) HSV 1
C) HPV
D) Condyloma acuminate
26- Bishop's score is based on the following EXCEPT:
A) Length of cervix
B) Cervical dilatation
C) Rupture of membranes
D) Cervical softening
27- The bladder is at risk of damage at the time of :
A) lower segment Caesarean section .
B) repair of enterocoele .
C) classical Caesarean section .
D) uterine myomectomy .
E) laparoscopy .
28- Classical Caesarean Section, all are false EXCEPT:
A) is Caesarean section performed through a mid-line abdominal incision .
B) is never done nowadays .
C) carries an increased risk of dehiscence of the abdominal wound .
D) is an indication for elective repeat Caesarean section .
E) is an indication for emergency tubal ligation at the time of the operation .
29- Favourable Bhishops score is above:
A)
4
B)
6
C)
8
D)
10
30- In a lower uterine segment cesarean section, the advantages of a vertical incision as compared with a transverse
incision include one of the following:
A) Less blood loss
B) more likelihood of extending to the broad ligaments
C) less likelihood of extending into the cervix and vagina
D) more room for extraction of the fetus
31- Which of the following abnormal presentations is an absolute indication
for a vertical cesarean section incision?

208

Gyna & Obstetrics Reveision by/Dr.adool

2008

A) Single footling breech at term


B) Complete breech at 32 weeks
C) Back up transverse lie
D) Back down transverse lie
E) Double footling breech at term
32- Which of the following prerequisites for forceps deliveries does NOT apply to vacuum extraction deliveries?
A) The head must be engaged
B) The cervix must be completely dilated
C) The position of the fetal head must be known
D) The membranes must be ruptured
E) None of the above
33- Barton forceps can be described by which of the following statements?
A) It is useful when the fetal head is in an oblique position in an platypelloid pelvis
B) It is useful when traction and rotation are to be performed simultaneously
C) Application of the blades can be adjusted by a sliding lock
D) The posterior blade is hinged
34- For each clinical situation described below, choose the appropriate type of obstetrical forceps.
A) Simpson forceps
B) Kielland forceps
C) Barton forceps
D) Chamberlain forceps
E) Piper forceps
1) A breech delivery with an aftercoming head
2) The occiput transverse position in a flat pelvis
3) Fetal rotation
35- For each of the following clinical descriptions, select the procedure that would be most appropriate.
A) External version
B) Internal version
C) Mid forceps rotation
D) Low transverse cesarean section
E) Classic cesarean section
1) A 24-year-old primigravid woman, at term, has been in labor for 16 hours and has been 9 cm dilated for 3 hours.
The fetal vertex is in the right occiput posterior position, at +1 station, and molded. There have been mild late
decelerations for the last 30 minutes. Twenty minutes ago, the fetal scalp pH was 7.27; it is now 7.20
2) A 24-year-old woman (gravida 3, para 2) is at 40 weeks gestation. The fetus is in the transverse lie presentation
3) You have just delivered an infant weighing 2.5 kg at 39 weeks gestation. Because the uterus still feels large, you
do a vaginal examination. A second set of membranes is bulging through a fully dilated cervix, and you feel
footling presentation in the sac. A fetal heart is auscultated at 60 beats per minute
36- The following are absolute indications for caesarean section EXCEPT:
A) Previous repair of vesicovaginal fistula
B) Fibroids in the lower segment of the uterus
C) Diabetic pregnancies
D) Posterior placenta praevia
37- External cephalic version (ECV), all are not true EXCEPT:
A) Reduces the incidence of breech presentation at term
B) Is contraindicated in rhesus isoimmunisation
C) The fetal mortality associated with ECV is chiefly due to cord problems

209

Gyna & Obstetrics Reveision by/Dr.adool

2008

D) Failed ECV may be due to a short umbilical cord


38- Induction of labour, all are true EXCEPT:
A) With simultaneous amniotomy and oxytocin 80% of women will deliver within 12 hours
B) Vaginal prostaglandin pessaries reduce the induction to delivery Interval
C) Cervical ripening is dependent on oxytocin
D) Cervical ripening can be achieved with estradiol
39- Forceps, all are true EXCEPT:
A) The Pajot maneuver can be used in absence of axis traction
B) A pudendal block is usually adequate for a Kielland's forceps delivery
C) Forceps should never be applied if the sinciput can be felt per abdomen
D) In a low forceps delivery, the application is correct when the anterior edge of the forceps blade is the same
distance on both sides from the lambdoidal suture
40- Forceps, all are not true EXCEPT:
A) When applying Kielland's if a blade cannot be applied easily then the procedure should be abandoned and
vacuum extraction performed
A) Long-term neurological deficit following Kielland's delivery is an established complication
B) Pudendal nerve damage may occur following a forceps delivery
C) Forceps exert most of their pressure on the cranium
D) The Kielland's forceps have a pelvic curve and an English lock
41- The following statements regarding the VENTOUSE (vacuum extractor) are true EXCEPT:
A) The cup is applied over the centre of the posterior fontanelle
B) Its use is associated with more maternal trauma than forceps
C) Should not be undertaken when the fetal head is above 'the level of the ischial spines
D) Minor scalp abrasions and subgaleal haematomas are more frequent than after forceps delivery
42- Previous scars on the uterus all are true EXCEPT:
A) An elective caesarean section should be performed if at myomectomy the uterine cavity was entered
B) The conduct of labour may be benefited by the provision of an epidural
.
C) Following a classical caesarean section rupture is at least as probable in the last 19 days of pregnancy as
during labour
D) The incidence of scar rupture in labour following a previous lower segment caesarean section (LSCS) is 10%
E) Digital assessment of the lower segment may be required after delivery in patients with a previous LSCS
43- Late in the second stage of labour, your patient develops a non-reassuring fetal heart rate tracing necessitating
quick action on your part. Which of the following is a correct statement regarding the use of forceps to assist this
delivery:
A) The right forceps blade is applied first.
B) The fenestration should admit no more than a fingertip when correctly applied
C) The cervix does not need to be fully dilated
D) It is not necessary to know the position of the fetal head
E) An indwelling catheter should be left in the mother's bladder for the first 24 hours
44- Your gravida 2, para 1 patient has been pushing for two-and-one-half hours, and you are considering the use of
vacuum to assist delivery. This would be contraindicated if:
A) The gestational age is less than 35 weeks
B) The delivery will not require excessive traction
C) The head is fully engaged
D) The cervix is completely dilated
45- Midline episiotomy, compared with mediolateral episiotomy, is associated with:
A) Less blood loss
B) More pain
C) A higher chance of subsequent dyspareunia
D) Decreased chance of fourth-degree extension
46- A bolus injection of oxytocin is conraindicated because of the risk of:

210

Gyna & Obstetrics Reveision by/Dr.adool

2008

A) Hyperkalemia
B) Hypokalemia
C) Hyperglycemia
D) Hypotension
47- How much time is required for the full effect of an increase in oxytocin dosage to be evident?
A) 5 minutes
B) 10-15 minutes
C) 30-40 minutes
D) 60-80 minutes
48- Which of the following defines outlet forceps delivery?
A) Scalp visible at the introitus without separating the labia
B) Fetal head on the perineum
C) Angle between the AP line and the sagittal suture not more than 45 degrees
D) All of the above
49- Which one of the following forceps lacks a pelvic curve?
A) Simpson
B) Tucker-McLane
C) Elliot
D) Kielland
50- In the Bishop score of inducibilily, points are given for all of the following except:
A) Effacement
B) Station
C) Consistency
D) Contractions
51- Which of the following is a risk of amniotomy?
A) Prolapse of the cord
B) Adverse change in position of the head
C) Rupture of unsuspected vasa previa
D) All of the above

52- A patient is receiving an oxytocin infusion, and uterine hyperstimulation is noted. No FHR abnormalities are
noted. All of the following should be performed except:
A) Discontinue oxytocin
B) Turn patient on her left side
C) Give oxygen
D) Give tocolytic agents
53- All of the following increase the risk of uterine rupture and are relative contraindications to the use of oxytocin
except:
A) Grand multiparity
B) Previous fundal uterine surgery
C) Ovcrdisiended uterus
D) Myomata uteri

211

Gyna & Obstetrics Reveision by/Dr.adool

2008

54- If ECV for a transverse lie is unsuccessful, the next step would be:
A) Cesarean section
B) Internal podalic version
C) Breech extraction
D) None of the above
55- The characteristic feature of Piper forceps is its:
A) Ability to allow both traction and rotation
B) Ability to allow reversed traction
C) Capacity to be applied directly to the fetal head
D) Absence of fenestration on its blade
56- All of the following indications have contributed to the increase in the cesarean section rate except:
A) Electronic fetal heart rate monitoring
B) Dystocia
C) Breech
D) Prior cesarean section
E) Maternal infection with human papilloma virus
57- Match the skin incision type with the appropriate description.
1) Vertical
2) Transverse Pfanestiel
3) Transverse Maylard
4) Cherney
A) More rapid access lo the lower uterine segment
B) Requires incision of anterior rectus sheath and rectus muscles
C) Peritoneum incised transversely
D) Easily extended
E) Preferred cosmetic result
58- All of the following complications are more common with a vertical uterine incision than a transverse uterine
incision except:
A) Subsequent uterine rupture in labor
B) Lateral extension into uterine vessels
C) Extension inferiorly
D) Intraoperative bleeding
E) Subsequent adhesions
59- At the time of a cesarean section, management of a laceration of the dome of the bladder should include which of
the following?
A) A two-layer closure
B) Use of nonabsorbable suture
C) Drainage of the space of Retzius
D) Placement of an indwelling catheter for 48 hours
60- When delivery is accomplished by postmortem cesarean section, how long can one delay after cardiac arrest and
still expect fetal survival?
A) 5 minutes
B) 7 minutes
C) 10 minutes
D) 12 minutes
61- The most frequent complication arising from cesarean delivery is:

212

Gyna & Obstetrics Reveision by/Dr.adool

2008

A) Urinary tract infection


B) Wound infection
C) Endomyometritis
D) Ileus
62- Which of the following is the most important risk factor for the development of endomyometritis?
A) Length of labor
B) Length of surgery
C) Preoperative hematocrit
D) Type of anesthesia
63- Which of the following is the most valuable sign of uterine rupture during a trial of labor after prior cesarean
section?
A) Cessation of labor
B) Loss of station of the presenting pan
C) Vaginal bleeding
D). Non reassuring fetal heart rate patterns
64- Which of the following is the most common emergency indication for cesarean hysterectomy?
A) Placenta accreta
B) Uterine rupture
C) Leiomyoma preventing uterine closure
D) Uterine atony
65- All of the following are indications for a classic cesarean section except:
A) Inaccessible lower uterine segment
B) Impacted transverse lie
C) Carcinoma of the cervix
D) Premature twins in breech/vertex presentation

Forceps-ventouse
1-The following are advantages of forceps over ventouse, except:
A. Less risky on the fetus.
B. Could be applied on medically diseased mother.
C. Less time for application.
D. Could not be applied in non-vertex presentation.
2-The maximal negative pressure of the ventouse should not exceed :
A. 0.6 Kgm/cm
B. 0.8 Kgm/cm
C. 1 Kgm/cm
D. 1.2 Kgm/cm
3-The obstetric forceps:
A. The blades are named right or left according to the side of the fetal head to
which the blade is applied.
B. The axis traction forceps is the most commonly used type in modern obstetrics.
C. With mid forceps, the biparietal diameter lies between the pelvic inlet and
the ischial spines.
D. Actions of the forceps include traction, rotation, compression and lever action.
E. With Scanzoni double application, the first application is cephalic.
4-Kielland forceps:
A. Blades are named right and left.

213

Gyna & Obstetrics Reveision by/Dr.adool

2008

B. Are lighter than the classic long forceps.


C. Can be used when the head is not engaged.
D. Can be used on an asynclitic head.
E. Can be used with deep transverse arrest.
5-Indications of delivery using the forceps include:
A. Fetal distress during the second stage.
B. Moderate degree of cephalo-pelvic disproportion.
C. Breech presentation.
D. Maternal distress during the second stage.
E. Uterine inertia.
6-Conditions to be fulfilled before forceps application include:
A. Engaged head.
B. Vertex presentation.
C. No outlet contraction.
D. General anesthesia.
E. Rupture of membranes.
7-Contraindications to application of forceps application include:
A. Unengaged head.
B. Incomplete cervical dilatation.
C. Face presentation.
D. Prematurity.
E. Complete absence of uterine contractions.
8-Potential complications of forceps delivery include:
A. Cervical lacerations.
B. Atonic postpartum hemorrhage.
C. Intra cranial hemorrhage.
D. Facial nerve palsy.
E. Injury to pelvic joints and nerves.
9-Failed forceps:
A. Is failure of delivery using the forceps after repeated attempts.
B. May be due to uterine inertia.
C. May be due to unrecognized malposition.
D. Usually does not result in complications.
E. The case is usually managed by repeating the attempts.
10-Infants born by forceps should be examined for:
A. Facial nerve paralysis.
B. Ear abrasions.
C. Intracranial hemorrhage.
D. Cephalhematoma.
E. Fracture femur.
11-The vacuum extractor:
A. Is an instrument designed to apply traction on the fetal head.
B. Can be used without anesthesia.
C. The cervix should be fully dilated.

214

Gyna & Obstetrics Reveision by/Dr.adool

2008

D. Can be used with face presentation.


E. Is more suitable than the forceps for emergency situations
12-Indications of delivery using the ventouse include:
A. Fetal distress during the second stage.
B. Moderate degree of cephalo-pelvic disproportion.
C. Persistent oblique occipito-posterior.
D. Maternal distress during the second stage.
E. Deep transverse arrest of the head.
13-Value of pelvic curve of obstetric forceps is:
A) To adapt with concavity of the sacrum.
B) To get a central grip upon the fetal head, thus will promote flexion during traction.
C) To grasp a high head.
D) All of the above.
14-The cephalic curve acts to:
A) Get a firm grip upon the faetal head.
B) Minimize head compression.
C) All of the above.
15-Kielland forceps was mainly used for cases of:
A) Occipito anterior.
B) Occipito posterior.
C) Asynclitism.
D) (A+B).
E) (B+C).
16-Kielland forceps is characterized by the following, EXCEPT:
A) Anterior & posterior blades rather than right & left.
B) Minimal pelvic curve.
C) Perineal curve.
D) Sliding lock.
17-Simpson's forceps is:
A) Useful for a rotated head at station +2.
B) Short & straight.
C) Useful for delivery of after coming head of breech.
D) All of the above.
18-The forceps function, most commonly used nowadays, is:
A) Traction.
B) Rotation.
C) Lateral lever action.
D) Slight head compression.
19-In modern obstetrics, forceps is used as:
A) High forceps.
B) Low & outlet forceps.
C) Rotational forceps.
D) All of the above.

215

Gyna & Obstetrics Reveision by/Dr.adool

2008

20-Forceps delivery is, most commonly, used for:


A) Cases with complete uterine inertia.
B) To shorten the second stage of labor in an exhausted mother.
C) Prolapsed pulsating cord with fully dilated cervix.
D) Mento anterior with face on the perineum.
21-Prerequesites for forceps application include the following, EXCEPT:
A) Fully dilated & effaced cervix.
B) Empty bladder & rectum.
C) Vault of fetal skull above the ischial spine.
D) Presence of uterine contractions.
22-Cephalo - pelvic application of forceps is ONLY possible in:
A) Direct occipito anterior.
B) Face to pubis.
C) Direct mento anterior.
D) All of the above.
23-In contracted pelvis, forceps is used to:
A) Overcome cephalo-pelvic disproportion by head compression.
B) Shorten 2nd stage of labor after successful trial of labor.
C) Deal with outlet contraction.
D) All of the above.
24-Epidural analgesia has:
A) Increased the need for forceps delivery.
B) Decreased the need for forceps delivery.
C) Not affected such a need.

VACUUM EXTRACTION
(1)Vacuum extraction is contraindicated in the following conditions, EXCEPT:
A) Non-vertex presentation.
B) Occipito posterior presentation.
C) Prematurity.
D) Fetal distress.
(2)To promote head flexion during vacuum delivery, the cup is applied centrally against the sagittal suture, but:
A) Nearer to the anterior fontanelle.
B) Nearer to the posterior fontanelle.
C) Midway between the two fontanelles.
(3)Advantages of vacuum extraction, over forceps delivery, include the following, EXCEPT:

216

Gyna & Obstetrics Reveision by/Dr.adool

2008

A) Does not encroach upon the maternal pelvic space.


B) Does not need general anaesthesia.
C) Can be applied with cervix not fully dilated.
D) Can be used to rotate a case of occipito posterior.
(4)Fetal head injuries due to vacuum extraction include the following, EXCEPT:
A) Cephalhaematoma.
B) Avulsion of scalp.
C) Facial nerve injury.
D) Tear of vein of Galen & intracranial haemorrhage.
(5)With metal cup vacuum extractor, followings are true, EXCEPT:
A) Higher incidence of shoulder dystocia.
B) Higher incidence of retinal haemorrhage.
C) Lower incidence of shoulder dystocia.
D) Increased incidence of intra cranial haemorrhage.

(11) * CAESAREAN SECTION (C.S.)


(1)During C.S., head delivery is effected by:
A) Lifting the head by a hand below it.
B) One blade of Wrigley's forceps inserted to act as a lever.
C) Vacuum extraction.
D) All of the above.
(2)Upper segment CS is indicated in the following, EXCEPT,:
A) Unavailable lower uterine segment.
B) Successfully repaired high vesico vaginal fistula.
C) Constriction ring.
D) Impacted shoulder.
(3)In the case of constriction ring, C.S. should be:
A) Upper segment C.S.
B) Lower segment C.S. with a transverse uterine incision.
C) Lower segment C.S. with a vertical uterine incision.
(4)To save a living child after his mother's death, post mortum C.S. should be done within:
A) 5 minutes.
B) 10 minutes.
C) 15 minutes.
(5)Which of the following is considered to be strongest uterine scar:
A) Upper segment C.S.
B) Lower segment C.S.
C) Myomectomy scar for interstitial or sub serous myomata.
D) Repaired uterus didylphes.

(6)Trial of vaginal delivery after C.S. is indicated, in the following conditions, EXCEPT:
A) Non- persistant indication.

217

Gyna & Obstetrics Reveision by/Dr.adool

2008

B) Previous two C.S.


C) No tenderness on the C.S. scar.
D) Vertex presentation.
7- Magnesium sulfate
E) Suppresses synthesis of endogenous prostaglandins
F) Inhibits oxytocin release
G) Stimulates cyclic AMP formation
H) Inhibits calcium influx

218

Gyna & Obstetrics Reveision by/Dr.adool

2008

Mcqs exam 1
Answer all the following questions with True or False
1-

Bicornuate uterus might predispose to:


a)
Recurrent preterm labor. T
b) Primary amenorrhea. F
c)
Recurrent oblique lie. T
d) Retention of the placenta after delivery. T
e) Menorrhagia. T

2-

Ovulation in the human:


a) Is associated with a surge of luteinising hormone. T
a)
b)
c)
d)

Is characteristically followed by the development of secretory endometrium. T


Is associated with an increase in motility of the fallopian tube. T
Is associated with a sustained fall in basal body temperature. F
Is followed by a rise in urinary pregnantriol. F

3- Treatment of dysfunctional uterine bleeding include:


a) Progestin therapy. T
b) Estrogen therapy. T
c) Danazol. T
d) Endometrial ablation. T
e) Anti-fibrinolytic drugs. T
4-

Amenorrhea could be due to which of the following:


a)
Combined oral contraceptive. T
b) Depot medroxy progesterone acetate. T
c)
GnRH analogues. T
d) Danazole. T
e) Bromocreptine. F

5-

The main supports of the uterus include the:


a) Round ligaments. F
b) Cardinal (lateral cervical, Mackenrodts) ligaments. T
c) Broad ligaments. F
d) Uterosacral ligaments. T
e)Levator ani muscles. T

6-

Genital prolapse is associated with:


a) Multiparity. T
b) Prolonged second stage of labor. T
c) Unrepaired hidden perineal tear. T
d) Improperly applied forceps delivery. T
e)More prevalent in white than in black women. F
Factors important in the development of genital prolapse include:
a) Poor tissue strength. T
b) Chronic straining at bowel movements. T
c) Menopause. T
d) Childbirth trauma. T

7-

219

Gyna & Obstetrics Reveision by/Dr.adool

2008

8- The following are advantages of laparoscopy in the diagnosis of infertility:


a) Good evaluation of tubal factor. T
b) Diagnosis of endometriosis. T
c) Diagnosis of Ashermans syndrome. F
d) Good visualization of pelvic adhesions. T
e) Direct visualization of corpus luteum. T
9- The following statements are true as regards serum progestome measurement in diagnosis of infertility:
a) The normal level is 5 ng/ml. F
b) Measurement is usually done at day 21 of the cycle. T
c) Can be used for assessment of the luteal phase defect. T
d) The normal level is variable between individuals. T
10-

For each evaluation, select the most appropriate day of a normal 28-day menstrual cycle:
a) Day 3. 5
1- Endometrial biopsy for evaluation of infertility.
b) Day 8. 3

2- Post-coital test.

c) Day 14. 2

3- Hysterosalpingogram.

d) Day 21. 4

4- Determination of serum progesterone level to


document ovulation.

e) Day 26. 1

5- Gonadotrophin evaluation.

11-

The character of normal cervical mucus at the time of ovulation is:


a) Highly viscous and turbid. F
b) Spinnbarkiet test is less than 6 cm. F
c)
Contains low amount of crystals of sodium and potassium chloride. F
d) Positive ferning test. T

12-

Polycystic ovarian disease characterized by all of the following:


a) Increase LH level. T
b) Increase in androgen levels. T
c)
Decrease in estrogen levels. F
d) Oligomenorrhea. T

13-

Hysteroscopy is important for diagnosis of the following factors in infertility:


a) Intrauterine adhesions. T
b) Sub mucous fibroids. T
c)
Peritubal adhesions. F
d) Cornual block. T

14-

Rectovaginal fistulas result from:


a)
Obstetrical delivery. T
b) Irradiation to the pelvis. T
c)
Carcinoma. T
d) Hemorrhoidectomy. T
e) Herpes vulvitis. F

220

Gyna & Obstetrics Reveision by/Dr.adool

2008

15-

Uterine fibroids are:


a)
Mainly corporeal. T
b) Common with low parity. T
c)
Increase in size after the menopause. F
d) Hyaline degeneration is the commonest secondary change. T
e) Sarcomatous change occurs in 5%. F

16-

Characteristic symptoms of endometriosis include:


a) Dysmenorrhoea. T
b) Amenorrhea. F
c)
Pre-menstrual tension. F
d) Infertility. T

17-

The following is essential in diagnosis of pelvic endometriosis:


a)
Ultrasonography. F
b) Laparocopy. T
c)
Hysteroscopy. F
d) CA-125. F
e) Biopsy form the suspicious nodules. T

18-

Functional ovarian cysts include:


a) Follicular cysts. T
b) Endometriomas. F
c) Dermoid cysts. F
d) Luteomas. T
e) Fibromas. F

19- A 54-year-old woman is found to have endomertial hyperplasia on endometrial biopsy. A functional ovarian
tumor to be suspected is a:
a) Lipid cell tumor. F
b) Granulosa-theca cell tumor. T
c) Sertoli-Leydig yumor. F
d) Mucinous cystadenocarcinoma. F
e) Polycystic ovary. F
20-

Complications of benign ovarian neoplasms include


a) Torsion. T
b) Infection. T
c) Rupture. T
d) Incarceration. T
e) Malignant change. T

21-

The following are contraindications to postmenopausal estrogen replacement therapy:


a)
Recent deep vein thrombosis. T
b) Acute liver disease. T
c)
High serum triglycerides. T
d) Premenopausal mastectomy for breast cancer. T
e) Recent myocardial infarction. T

221

Gyna & Obstetrics Reveision by/Dr.adool

2008

22-

Postmenopausal bleeding is a common presentation of:


a)
Cervical ectropion. F
b) Carcinoma of the endometrium. T
c)
Atrophic vaginitis. T
d) Carcinoma of the cervix. T
e) Uterine fibroids. F

23-

The use of combined oral contraceptives reduces the risk of:


a)
Ectopic pregnancy. T
b) Hepatic adenoma. F
c)
Salpingitis. T
d) Ovarian cancer. T
e) Endometrial cancer. T

24-

All these methods could be used during lactation except:


a) Depot provera. F
a)
b)
c)
d)

Mini pills. F
Combined contraceptive pills. T
I.U.D. F
Male condom. F

25-

All the following methods protect against S. T.D except:


a) Male condom. F
b) Female condom. F
c) Diaphragm. F
d) IUD. T

26-

The amniotic fluid:


a) Is of fetal contribution only. F
b) Its PH is acidic. F
c) Is increased in pre-eclampsia and anencephaly. T
d) Its marked diminution leads to skeletal abnormalities in the fetus. T
e) It has functions during pregnancy and labor. T

27- Warning symptoms that are given to a pregnant lady are:


a) Sudden loss of fluid per vagina. T
b) Bleeding per vagina. T
c) Excessive salivation (ptyalism). F
d) Leg cramps. F
e) Abdominal pain. T
28-

Fetal presentation:
a) Is the part of the fetus that enters the pelvis first. T
b) Is always cephalic. F
c) In cephalic presentation, it is more common to be face. F
d) In face presentation the occiput and back comes into contact. T
e) The denomintor is always the lowest part of the presenting area. F

222

Gyna & Obstetrics Reveision by/Dr.adool

2008

29- Engagement of the head can be assessed by:


a) Abdominal examination. T
b) Degree of moulding. F
c) Relationship to ischial tuberosity. F
d) Degree of caput formation. F
e) Relationship to ischial spines. T
30- The first stage of labor:
a) Ends in expulsion of the fetus. F
b) Begins when membranes rupture. F
c) Can be shortened with the use of oxytocin. T
d) Normally last for more than 24 hours in a primigravida. F
e) Is prolonged in malpositions of the head. T
31-

True labor pains are:


a) Regular. T
b) Relieved by enema. F
c) Associated with cervical dilatation. T
d) May be associated with ROM. T
e) Increase in intensity by time. T

32- The major causes of first trimester abortion:


a) Chromosomal anomalies. T
b) Circumvallate placentation. F
c) Endocrine factors. T
d) Immunological factors. T
e)
33Missed abortion:
a)
Implies that despite fetal death, the pregnancy has been retained. T
b) Exaggerated pregnancy symptoms. F
c)
Milk secretion may start spontaneously from the breast. T
d) There may be brownish vaginal discharge. T
e) Ultrasound is not helpful in its diagnosis. F
34-

The risk factors for an ectopic pregnancy:


a)
History of pelvic inflammatory disease. T
b) Previous ectopic pregnancy. T
c)
Use of combined oral contraceptives. F
d) Assisted reproductive techniques. T
e) IUD use. T

35-

The best contraceptive method for patients with gestational trophoblastic diseases:
a) Tubal ligaion. F
b) Hysterectomy. F
c) Combined oral contraceptive pills. T
d) Intra-uterine contraceptive device. F
e) Subcutaneous implants. F

223

Gyna & Obstetrics Reveision by/Dr.adool

2008

36-

Complications of abruptio placentae:


a)
Hemorrhagic shock. T
b) Consumptive coagulopathy. T
c) Postpartum haemorrhage.T
d) IUFD occurs in all cases if less than of the placenta is separated. F

37-

A couvelaire uterus:
a)
Is an indication for hysterectomy. F
b) Usually contracts well with stimulation. T
c)
Results from excessive oxytocin. F
d) Complications of severe abruption. T

38-

Management of placenta previa:


a)
Patients with severe bleeding & immature fetus are treated conservatively. F
b) Hysterectomy may be resorted to. T
c)
Active management is needed in cases of minimal bleeding & mature fetus. T
d) Vaginal delivery could be attempted in cases of low lying placenta. T
e) Puerperal sepsis is rare following delivery. F

39-

Which of the following is a sign of heart disease in pregnancy?


a)
Lower extremity edema.
b) Systolic murmur.
c)
Increased respiratory effort.
d) Arrhythmia. T

40-

Medical history suggestive of diabetes in pregnancy include the following:


a)
Past history of macrosomia. T
b) Past history of congenital malformations. T
c)
Past history of unexplained stillbirths. T
d) Positive family history. T

41-

Neonatal complications of infants of diabetic mothers includes the following:


a)
Neonatal hypoglycaemia. T
b) RDS. T
c)
Hypocalcemia. T
d) Hyperbilirubinaemia. T
e) Birth trauma. T

42-

The following are congenital fetal malformations causing polyhydramnios:


a)
Duodenal atresia. T
b) Syndectyly. F
c)
Anencephaly. T
d) Renal agenesis. F
e) Omphalocele. T

43-

Complications of polyhydramnios include:


a)
Preterm labor. T
b) Preterm premature rupture of membranes. T
c)
Antepartum hemorrhage. T

224

Gyna & Obstetrics Reveision by/Dr.adool

2008

d)
e)

Postpartum hemorrhage. T
Malpresentations. T

44-

Causes of oligohydramnios include:


a)
Postdate pregnancy. T
b) Chronic placental insufficiency. T
c)
Twin pregnancy. F
d) Renal agenesis in the fetus. T
e) Rupture of membranes. T

45-

Pathological retraction ring:


a)
It is also called Bandls ring. T
b) It occurs in the middle of the upper uterine segment. F
c)
It can be felt and seen abdominally. T
d) It has a fixed position and does not rise up with labor progression. F
e) It is a complication of obstructed labor. T

46-

Obstructed labor:
a)
Commonly occurs in cardiac patients. F
b) Commonly occurs in primigravidas. F
c)
Is reduced by proper maternal care. T
d) Vaginal examination shows dry, warm vagina. T
e)
Best management is by instrumental delivery. F

47-

Clinical picture of ruptured uterus following obstructed labor:


a)
Feeling of obstruction is replaced by feeling of giving way. T
b) The patient becomes noisy and irritable. F
c)
P/V reveals receding freely mobile presenting part. T
d) Strong uterine contractions suddenly stop. T
e) General condition of the patient is good. F

48-

Primary postpartum hemorrhage occurs more commonly:


a) In primigravidae. F
b) When there is a history of a previous postpartum hemorrhage. T
c) When fibroids are present in the uterus. T
d) In premature labor. F
e) After placental abruption. T

49-

Maternal mortality refers to the number of maternal deaths that occur as the result of the reproductive
process per:
a) 1000 births. F
b) 10.000 births. F
c) 100.000 births. T
d) 10.000 live births. F
e) 100.000 live births. F

50- Indications of use of vacuum extractor in assisted vaginal delivery include:


a) Fetal distress in the second stage. T
b) Face presentation. F
c) Marked cephalopelvic disproportion. F

225

Gyna & Obstetrics Reveision by/Dr.adool

2008

d) Shortening the second stage in hypertensive mothers. T


e) Suspected fetal coaulopathy. F
51-

Pre requisites for a forceps delivery include:


a)
A completely dilated cervix. T
b) An empty bladder. T
c)
The fetus in the transverse lie. F
d) Ruptured membranes. T
e) The head must not be engaged. F

52-

Advantages of lower segment C.S over upper segment C.S.:


a)
Less bleeding unless extended. T
b) Less paralytic ileus. T
c)
Less incidence of subsequent uterine rupture. T
d) Less infection. T

226

Gyna & Obstetrics Reveision by/Dr.adool

2008

Exam 2
Answer All the following Questions with True or False
1- Dysfunctional uterine bleeding (DUB) is frequently associated with:
o
o
o
o
o

Endometrial polyps.
Anovulation.
Cervicitis.
Systemic lupus erythematosus.
Von Willebrand's disease.

2- Rectal examination in the gynecological practice:


o
o
o
o
o

Is to be preformed as a routine in the gynecological practice.


Is useful in the diagnosis of enterocele.
Is indicated in the assessment of carcinoma of the cervix.
Is useful in assessment of ectopic pregnancy.
Is useful in suspected pelvic abscess.

3- In luteinized unruptured follicle:


o
o
o
o
o

Progesterone is secreted normally.


Menstrual cycles are usually regular
Antiprostaglandins are needed for rupture of ovarian follicles.
Induction of ovulation is a useful modality for treatment.
Patient can benefit from human chorionic gonadotrophin (hCG) injections given in high doses at midluteal phase.
4-Galactorrhea (non-gestational lactation) may result from:
o Pituitary adenoma.
o Hypothyroidism.
o Renal failure.
o Intrapartum hemorrhage.
o Bronchogenic carcinoma.
5- Bromocryptine drug:
o Is a dopamine agonist.
o Inhibits prolactin secretion.
o Can be administered vaginally.
o Is more potent than lisuride drug in treatment of hyperprolactinemia.
o Can cause hypertension.
6-Patients with testicular feminization characteristically:
o
o
o
o
o

227

Are 46, XY.


Have a hypoplastic uterus.
Are chromatin positive.
Have poorly developed breasts.
Have secondary amenorrhoea.

Gyna & Obstetrics Reveision by/Dr.adool

2008

7-Cryptomenorrhae may present with which of the following:


o
o
o
o
o

Acute urinary retention.


Hematocolpus.
Lower abdominal pain.
Premenstrual spotting.
Lower abdominal swelling

8- Genital prolapse is associated with:


o
o
o
o
o

Multiparity.
Prolonged second stage of labor.
Unrepaired hidden perineal tear.
Improperly applied forceps delivery.
More prevalent in white than in black women.

o
o
o
o
o

The general incidence of infertility is 10-15%.


The normal conception rate (following normal unprotected sexual life) after one month is 15-20%.
According to WHO, the normal sperm concentration 10 million/ml.
25% of infertility is unexplained.
More than one cause of infertility could be found in one couple.

9-Infertility:

10-Accepted parameters for a normal semen analysis are:


o
o
o
o
o

A sperm count of 10-20 million per ml.


A volume of 1 ml.
60% motility.
60% abnormal forms.
A positive mixed agglutination reaction.

11-The character of normal cervical mucus at the time of ovulation is:


o
o
o
o
o

Highly viscous and turbid.


Spinnbarkiet test is less than 6 cm.
Contains low amount of crystals of sodium and potassium chloride.
Positive ferning test.
May help in the diagnosis f ovulation

12-Polycystic ovarian disease characterized by all of the following:


o
o
o
o
o

228

Increase LH level.
Increase in androgen levels.
Decrease in estrogen levels.
Oligomenorrhea.
Bilateral Ovarian wedge resection is the most accepted method of treatment

Gyna & Obstetrics Reveision by/Dr.adool

2008

13-Hysteroscopy is important for diagnosis of the following factors in infertility:


o
o
o
o
o

Intrauterine adhesions.
Sub mucous fibroids.
Peritubal adhesions.
Cornual block.
Polycystic Ovarian Disease F

14- The natural defences of the vagina to infection include:


o
o
o
o
o

The vaginal pH.


The presence of Doderleins bacilli.
The physical apposition of the pudendal cleft and the vaginal walls.
The bacteriostatic secretions of vaginal glands.
The vaginal stratified squamous epithelium.

15-Gardnerella vaginalis vaginitis:


o Is usually asymptomatic.
o May present with a foul smell discharge.
o May progress to acute pelvic imflammatory disease.
o Should always be treated.
o Can be diagnosed by cule cells on Gram staining of vaginal discharge.

16- Human Papilloma Virus:


o
o
o
o
o

It is a rare type of STD.


It is classified into subtypes according to types of antibodies.
Has a definite pathological role in development of CIN.
Koilocytotic atypia is the main feature in the cytological study.
The most effective anti-viral is acyclovir.

17-Rectovaginal fistulas result from:


o
o
o
o
o

Ostetrical delivery.
Irradiation to the pelvis.
Carcinoma.
Hemorrhoidectomy.
Herpes vulvitis.

18- Indications for cesarean section include which of the following:


o
o
o
o
o

229

Previous Cesarean section


Failed forceps delivery
Fetal distress
Cervical cerclage
Cord prolapse

Gyna & Obstetrics Reveision by/Dr.adool

2008

19-

Endometrial hyperplasia could be expected in the following conditions:


o Endodermal sinus tumor.
o Cystic teratoma.
o Polycystic ovary disease.
o Sertoli-Leydig cell tumor.
o Dysgerminoma.

20-Risk factors for the development of cancer cervix include:


o
o
o
o
o

Early sexual activity.


White race.
HPV infection.
High social class.
Multiple sexual partners.

21-Which of the following signs or symptoms characterize severe placental abruption:


o
o
o
o
o

Fetal Demise
Tetanic uterine contractions
Retroplacental Clot
Extensive external bleeding
Severe abdominal Pain

22- The absolute contraindications to the use of COCs pills:


o
o
o
o
o

Hypertension.
Diabetes mellitus.
Sickle cell anemia.
D.V. T.
Heavy smoking.

23-All these methods could be used during lactation except:


o
o
o
o
o

Depot provera.
Mini pills.
Combined contraceptive pills.
I.U.D.
Male condom.

24-The normal umbilical cord:


o
o
o
o
o

Contains two arteries.


Contains Wharton's jelly.
Pulsates at a rate equal to the maternal pulse rate.
Is covered by amnion and chorion.
Contains two veins.

230

Gyna & Obstetrics Reveision by/Dr.adool

2008

25-The following vessels contain oxygenated blood in the fetus:


o
o
o
o
o

Umbilical artery.
Ductus venosus.
The inferior vena cava as it enters the right atrium.
Carotid artery.
Umbilical vein.

26-The following laboratory investigations are component of the routine antenatal care
o
o
o
o
o

HCG.
Rh typing.
Urine analysis for sugar and protein.
Urine culture.
Test for syphilis.

27-A pregnant uterus that is larger than the period of amenorrhea could be due to:
o
o
o
o
o

Generalized edema.
Obesity.
Intrauterine growth retardation.
Breech presentation.
Polyhydramnios.

28-Management of intrapartum acute fetal distress:


o
o
o
o
o

Oxygen administration is a useful step.


Change of maternal position is not an important factor.
Correction of maternal hypotension is necessary.
C.S should be considered if the cervix is no fully dilated.
Vacuum extraction is the best choice to cut short the 2nd stage of labor

29- Ultrasound in obstetrics:


o
o
o
o
o

Transabdominal transducer has a frequency more than 10 MHz.


Useful in diagnosis of incompetent cervix.
Can diagnose fetal cardiac anomalies in first trimester.
Has a role in diagnosis of the type of twins.
most accurate in the 3rd trimester to estimate the gestational age

30-Diagnosis of early ectopic pregnancy:


o
o
o
o
o

231

Clinical examination is almost always diagnostic of ectopic.


-subunit hCG is doubled every 48 hours in ectopic pregnancy.
Transvaginal ultrasound is helpful in conjunction with hCG in diagnosis of early ectopic.
Culdocentesis is a current method for early diagnosis.
Laparoscopy is the golden standard for diagnosis.

Gyna & Obstetrics Reveision by/Dr.adool

2008

Exam 3
(Select ONLY ONE item)
1.The most common indication for cesarean section is:
A. Malprepresentations.
B. Ante partum hemorrhage.
C. Prematurity.
D. Previous cesarean section.
E. Contracted pelvis.
2.The following are etiological factors of atonic post partum hemorrhage except:
A. Prolonged labor.
B. Over distension of the uterus.
C. Accidental hemorrhage.
D. Cervical lacerations.
E. Full bladder.
3.The following are included in the management of hemorrhagic shock except:
A. Oxygen.
B. Magnesium sulfate.
C. Monitoring of urine output.
D. Corticosteroids.
E. Monitoring of CVP.
4.Pap smear:
A. The next step in dysplastic smear is colposcopy.
B. Is simple but inaccurate test.
C. Should be carried out every 5 years.
D. Has no role in screening of asymptomatic women.
E. All of the above.
5.All of the followings are histological types of endometrial cancer except:
A. Adeno-carcinoma
B. Transitional cell carcinoma
C. Adeno-squamous carcinoma
D. Papillary serous carcinoma
E. Clear cell carcinoma
Non-neoplastic cysts of the ovary include all of the followings except:
A. Follicular cyst.
B. Theca lutein cyst.
C. Dermoid cyst.
D. Corpus luteum cyst.
E. Endometrioid cyst.
6.Non-neoplastic cysts of the ovary include all of the following EXCEPT:
A. Follicular cysts.
B. Simple serous cysts.
C. Endometriotic cysts.

232

Gyna & Obstetrics Reveision by/Dr.adool

2008

D. Dermoid cysts.
E. Theca lutein cysts.
7.Indications of cesarean section in breech presentation include all of the following EXCEPT:
A. Large fetus.
B. Premature fetus.
C. Breech with extended legs.
D. Footling presentation.
E. Post term.
8.All of the following regarding cord presentation is true EXCEPT:
A. Is more common with malpresentations.
B. May be associated with abnormal shape of the pelvis.
C. Carries the risk of cord prolapse.
D. Can be diagnosed by PV.
E. Is associated with rupture of membranes.
9.With multiple pregnancy, there is a higher incidence of all of the following EXCEPT:
A. Abortion.
B. Vesicular mole.
C. Preeclampsia.
D. Placenta previa.
E. Polyhydramnios.

10.A 30 years old patient presented with moderate to severe post partum hemorrhage, the placenta was still not
delivered, management should include all of the following EXCEPT:
A. Correction of the general condition.
B. Monitoring of vital signs and urine output.
C. Crede method to deliver the placenta.
D. Oxytocin drip.
E. Prostaglandin F2 alpha IV.

MCQs (one or more)


11-Peripheral conversion of androstenedione results in the formation of:
A. Estrone.
B. Estradiol.
C. Estriol.
D. Dehydroepiandrosterone.
E. Progesterone.
12-A couple presented complaining of absence of pregnancy after 4 months of marriage, proper management
includes:
A. Waiting for another 8 months before starting any management.
B. Examination of the female.
C. Full history taking.
D. Semen analysis.
E. Induction of ovulation.

233

Gyna & Obstetrics Reveision by/Dr.adool

2008

13-A 25 year old virgin presented with dull aching pain in the left iliac fossa, together with delayed menses for 18
days, abdominal ultrasonography revealed a unilocular ovarian cystic swelling of 6.2 cm in maximal diameter,
appropriate management alternatives include:
A. Follow up only.
B. Surgical excision.
C. Oral contraceptive pills and follow up.
D. Laparoscopic aspiration.
E. Laser therapy.
14-Causes of sub involution of the uterus include:
A. Infection.
B. Retained placental fragment.
C. Uterine over distension during pregnancy.
D. Non-suckling.
E. Prolapse.
15-Management options of occipito posterior position include:
A. The majority of cases are delivered by cesarean section.
B. Cases of direct occipito posterior may be delivered using the ventouse.
C. Cases of persistent occipito posterior may be delivered using the forceps.
D. Craniotomy is indicated if the fetus is dead.
E. Cases with long anterior rotation are delivered as normal labor.

Problem solving
(One or more items may be selected)
1-A 32 year old, third gravida, pregnant 38 weeks presented to the antenatal clinic, abdominal examination &
ultrasonography revealed complete breech presentation with no apparent anomalies, the estimated fetal weight
(EFW) was 3250 gm, appropriate management alternatives include:
A. Induction of labor.
B. External cephalic version.
C. External cephalic version followed by induction of labor.
D. Elective cesarean section.
E. Follow up, waiting for onset of labor.
2-A couple presented complaining of absence of pregnancy after 4 months of marriage, proper management
includes:
A. Waiting for another 8 months before starting any management.
B. Examination of the female.
C. Full history taking.
D. Semen analysis.
E. Induction of ovulation.
3-A 25 year old virgin presented with dull aching pain in the left iliac fossa, together with delayed menses for 18 days,
abdominal ultrasonography revealed a unilocular ovarian cystic swelling of 6.2 cm in maximal diameter, appropriate
management alternatives include:
A. Follow up only.
B. Surgical excision.
C. Oral contraceptive pills and follow up.

234

Gyna & Obstetrics Reveision by/Dr.adool

2008

D. Laparoscopic aspiration.
E. Laser therapy.
4-A 44 years lady presented asking to postpone menstruation before going to Mecca, she expects her menstruation
after 18 days, appropriate and reliable method(s) include:
A. Combined oral contraceptive pills.
B. Progestin only pills.
C. Norethisterone acetate.
D. Progesterone.
E. GnRH analogues.
5-A 58 years old lady G3 P2, menopausal since 5 years, complained of offensive vaginal discharge associated with
recurrent spotting, she is hypertensive since 20 years, occasionally uncontrolled, appropriate steps include:
A. Measurement of blood pressure.
B. Abdominal ultrasonography.
C. Treatment of vaginitis.
D. Fractional curettage.
E. Urine analysis.

Short answer questions


1-A 38 years old fifth gravida gave vaginal birth to a live birth male of 4100 gm birth weight; the placenta was
delivered with no missing parts. The duration of labor was 16 hours. Half an hour after delivery, she was found to be
very pale, irritable, her pulse was very weak, with a rate of 120 beats/min, her blood pressure was 80/50 mmHg and
there was moderate bleeding seen soiling the bed.
The most likely diagnosis is:
First 2 steps in management are:
1.
2.
2-A 24 years old G2 P2 female complained of headache and extreme breast tenderness together with temperature
reaching 38.6 degrees 3 days after delivery of a preterm infant who was admitted to neonatal critical care unit, she
denies any excessive lochia or infected vaginal discharge.
The most likely diagnosis is:
Appropriate management is:
1.
2.
3-In this HSG picture:
A. Both tubes are patent.
B. The uterine cavity shows a normal outline.
C. Peritoneal spill is not evident.
D. The arrows point to sites of tubal occlusion.

235

Gyna & Obstetrics Reveision by/Dr.adool

2008

4-The method used for delivery of the head is known as:


A. Kristeller method.
B. Brandt Andrews method.
C. Mauricau Smellie Viet method.
D. Pinard's method.

5-The technique shown in the diagram is known as:

It is used in cases of:


1..
2..

6-The complication shown in the diagram is:

Its predisposing factors are:


1..
2..
3..
Its main presenting symptom is:

7-A 38 year old sixth gravida, had a pre term vaginal delivery, one
hour later, the placenta was still undelivered, manual removal of the
placenta under anesthesia was repeatedly attempted but failed,
proper management is:
..
8-. A 47 year old woman underwent a total abdominmal hysterectomy
for chronic pelvic pain due to endemetriosis, on the second post
operative day, she noticed leakage of watery fluid from the vaqgina,
and her blood pressure was 130/80, pulse 84 beats/min and
temperature 37 C.
The most likely diagnosis is:
.
The first line in management is:
..

236

Gyna & Obstetrics Reveision by/Dr.adool

2008

True/False questions
1-The mechanism of labor of occipito posterior position:
A. The degree of deflexion determines the outcome.
B. Deflexion is corrected in the great majority of cases.
C. The head always rotates 3/8 of a circle anteriorly.
D. With major deflexion, there is no mechanism and no labor is obstructed.
E. With direct occipito posterior, the head can be delivered in flexion.
2-Ultrasonography in diagnosis of multiple pregnancy:
A. Diagnosis early in pregnancy.
B. Detection of number of fetuses.
C. Detection of presentation and position.
D. Exclusion of congenital anomalies and conjoined twins.
E. Assessment of pelvic capacity.
3-Choriocarcinoma:
A. Usually follows an event of pregnancy.
B. A friable hemorrhagic polypoidal mass projects into the uterine cavity.
C. Shows early lymphatic spread.
D. The main presentation is bleeding.
E. The most significant method of investigation is ultrasonography.
4-Dysfunctional uterine bleeding:
A. May be associated with hypothyroidism.
B. May be associated with postmenopausal bleeding.
C. May be associated with functional ovarian cysts.
D. May present as menorrhagia.
E. May present as metropathia hemorrhagica.

Extended matching questions (EMQs)


1. Atonic post partum hemorrhage.
2. Traumatic post partum hemorrhage.
1 -For each item below, choose the SINGLE most appropriate answer from the above list
of options. Each option may be used once, more than once or not at all.
A. The bleeding is bright red. 2
B. Passage of blood clots with pressure on the fundus. 1
C. Is more common with malpresentations.
D. Blood transfusion is required in some cases.
E. Internal iliac artery ligation may be required in some cases.
F. Ergometrine is used for treatment.
G. Is more common with induced labor.
H. The general condition is affected in proportion to the amount of revealed bleeding.
I. Bimanual compression is required in some cases.
1. Mono zygotic twins.
2. Di zygotic twins.
3. Breech presentation.

237

Gyna & Obstetrics Reveision by/Dr.adool

2008

2-For each item below, choose the appropriate answer from the above list of options.
Each option may be used once, more than once or not at all.
A. More in primigravida.
B. Higher incidence of growth retardation.
C. Related to pre term labor.
D. Higher incidence of cesarean section.
E. Related to family history.
F. Higher incidence of preeclampsia.
G. Ultrasonography is essential in the diagnosis.
H. Over sized abdomen.
1. A couple presented complaining of failure of conception for 4 years. Semen analysis was subfertile, serum
progesterone was 9.4 ng/ml at day 22 of the cycle, hysterosalpingography revealed normal findings, further
management could include:
2. A 33 year old virgin presented with moderate lower abdominal pain lasting for the last 20 days, medical history was
irrelevant apart from troublesome dysmenorrheal partly relieved by strong analgesics, trans-abdominal
ultrasonography revealed no abnormality, further management could include:
3-For each management below, choose the appropriate answer from the above list of
options. Each option may be used once, more than once or not at all.
A. Laparoscopy.
B. Super ovulation induction.
C. Danazol.
D. HMG.
E. HCG.
F. GnRH analogues.
G. Progestins.
H. IUI.

238

Gyna & Obstetrics Reveision by/Dr.adool

2008

Exam 4
1. Excessive maternal weight gain is most strongly associated with which of the following?
a. fetal macrosomia
b. preterm delivery
c. placental praevia
d. oligohydramnios
2. Abortion include which of the following?
a. pregnancy intentionally ended prior to 20 weeks
b. pregnancy spontaneously ended prior to 20 weeks
c. pregnancy intentionally ended with a fetus weighing <500 g
d. all of the above
3. What is the term for no visible fetus in the gestational sac?
a. blighted ovum
b. miscarriage
c. septic abortion
d. polar body
4. Your patient has had two first-trimester and one second-trimester spontaneous abortions. Her screen for anticardiolipin
antibodies is positive. The most appropriate treatment option for this patient during her next pregnancy includes which of the
following?
a. low-dose aspirin plus heparin
b. low-dose aspirin plus plaquenil sulfate
c. low-dose aspirin plus steroids
d. reassurance
5. What causes pregnancy loss in women with antiphospholipid antibodies?
a. placental thrombosis and infarction
b. increased vascularization of decidua basalis
c. increased prostacyclin release
d. protein C activation
6. Which of the following is the most appropriate and accurate test to confirm the diagnosis of Asherman syndrome?
a. ultrasonography
b. antinuclear antibody assay
c. hysteroscopy
d. lupus anticoagulant assay
7. Optimally, when should prophylactic cerclage procedures be performed?
a. preconceptionally
b. between 6 to 12 weeks gestation
c. between 12 to 16 weeks gestation
d. at first sign of cervical dilatation
8. Your patient presents with bleeding at 6 weeks gestation. She is afebrile and isovolemic. Physical examination reveals
tissue protruding through her cervical os. Which of the following is the next best management step for this patient?
a. suction curettage
b. observation and bed rest
c. serial ultrasonographic examinations
d. intramuscular progesterone

239

Gyna & Obstetrics Reveision by/Dr.adool

2008

9. Appropriate management of first-trimester missed abortion includes which of the following?


a. intravaginal misoprostol
b. dilatation and curettage
c. expectantly await spontaneous abortion
d. all of the above
10. The dilatation and evacuation method of surgical abortion should be used before which of the following gestational
ages?
a. 12 weeks
b. 16 weeks
c. 18 weeks
d. 20 weeks
11. With which of the following are tubal pregnancies NOT increased?
a. assisted reproduction
b. history of pelvic infection
c. previous tubal surgery
d. abnormal embryos T
12. Which method of contraceptive failure has an increased relative risk of ectopic pregnancy?
a. intrauterine device
b. progestin-only minipills
c. tubal sterilization
d. all of the above T
13. What is the most common ectopic tubal implantation site?
a. fimbria
b. ampulla T
c. isthmus
d. cornua
14. Sonographic evidence of an ectopic pregnancy includes which of the following?
a. adnexal mass
b. fluid in the cul de sac
c. lack of intrauterine gestational sac
d. all of the above T
15. What is the doubling time for Serum hCG levels in early pregnancy?
a. 24 hr
b. 48 hr T
c. 72 hr
d. 96 hr
16. In which surgical approach to ectopic pregnancy is the tube opened to remove the gestational products, then left
unsutured?
a. salpingectomy
b. salpingotomy
c. salpingorrhaphy
d. salpingostomy T
17. Complete molar pregnancies most commonly have which of the following genetic compositions?
a. 45, XO
b. 46, XY
c. 46, XX T
d. 47, XXY

240

Gyna & Obstetrics Reveision by/Dr.adool

2008

18. What is the etiology of theca-lutein cysts?


a. abnormal karyotype
b. increased prolactin receptors
c. increased chorionic gonadotropin T
d. increased follicle-stimulating hormone
19. What imaging technique is most useful in diagnosing molar pregnancy?
a. radiography
b. ultrasonography T
c. magnetic resonance imaging
d. computed tomographic imaging
20. Primary treatment of molar pregnancy is which of the following?
a. hysterotomy
b. suction curettage T
c. oxytocin induction
d. misoprostol induction
21. Choriocarcinoma is spread metastatically by which of the following methods?
a. lymphatic
b. hematogenous T
c. contiguous invasion
d. disseminated via peritoneal fluid
22. The cervix:
a) Does not undergo cyclic changes during the menstrual cycle.
b) Loses its lining during menstruation.
c) Has columnar epithelium lining the canal. T
d) Produces a thick scanty discharge at ovulation.
e) Has the same proportion of muscle in its wall as the corpus uteri.
23. Imperforate hymen:
a)
Can not lead to hematocolpos.
b)
Is usually identified earlier in life than at puberty.
c)
Is often found with uterus didelphys.
d)
Generally requires reconstructive surgery. T
24. Imperforate hymen is not a cause of:
e) Delayed puberty.
f)
Acute retention of urine.
g)
Abdominal mass.
h) Hypomenorrhea. T
25. Clinical findings of imperforate hymen include all of the following except:
i)
Hirsutism. T
j)
Cryptomenorrhea.
k)
Hematometra.
l)
Pelvi-abdominal swelling.
m) Retention of urine.
26. In a normal human menstrual cycle the corpus luteum:
a) Remains active for 3-4 weeks.
b) Is maintained by human chorionic gonadotrophin.
c) Secretes progesterone. T
d) Secretes pregnanediol.

241

Gyna & Obstetrics Reveision by/Dr.adool

2008

e)Does not secret estrogen.


27. In the first half of a normal menstrual cycle:
a)
Serum progesterone levels are high.
b)
The endometrium is rich in glycogen.
c)
The corpus luteum begins to degenerate.
d)
Some ovarian follicles degenerate. T
28. The average blood loss resulting from menstruation is:
a) 10 to 15 ml.
b) 25 to 50 ml.
c) 75 to 100 ml. T
d) 101 to 125 ml.
e) 130 to 150 ml.
29. Dysfunctional uterine bleeding (DUB) is frequently associated with:
n) Endometrial polyps.
o) Anovulation. T
p) Cervicitis.
q) Systemic lupus erythematosus.
r)
von Willebrand's disease.
30. Dysfunctional uterine bleeding is associated with:
a) Chronic pelvic inflammatory disease.
b) Abnormal hormone profiles in more than 50% of patients.
c) Increased levels of PGE2 in the endometrium. T
d) Estrogens inhibiting the arachidonic acid cascade.
31. Uterovaginal prolapse:
a) Is an acute very painful condition.
b) The condition is worse in the erect position. T
c) Is common in negroes.
d) May cause intestinal obstruction if there is a large rectocele.
33. Candidal infection:
a) The infection never occurs in pregnancy.
b) Vaginal PH is usually alkaline.
c) Vulval itching may occur. T
d) Vaginal isoconazol or miconazol are not effective.
34. Trichomonal vaginalis (T.V.):
a) T.V is not a sexually transmitted disease.
b) Most patients having TV have symptoms. T
c) The disease does not cause itching.
e) Unlike bacterial vaginosis, mitronidozole is not effective therapy.
35. For whom are the painless, irregular uterine contractions that begin early in gestation named?
a. Braxton Hicks T
b. Casey Alexander
c. Sheffield Yost
d. Wendel Smith

242

Gyna & Obstetrics Reveision by/Dr.adool

2008

36. What is pigmentation of the midline, anterior abdominal skin during pregnancy called?
a. striae gravidarum
d. melasma
b. linea nigra T
c. chloasma
37. In which of the following positions is cardiac output most increased in the pregnant
patient?
a. left lateral recumbent T
c. standing
b. right lateral recumbent
d. supine
38. Alteration of the ratio of which of the following is thought to be important in the etiology of
preeclampsia?
a. atrial natriuretic peptide: B-type natriuretic peptides
b. angiotensin: angiotensinogen
c. progesterone: dihydroprogesterone
d. prostacyclin: thromboxane T
39. Maternal obesity is related to an increase in which of the following maternal complications?
a. hypertension
c. preeclampsia
b. gestational diabetes
d. all of the above T
40. The Naegele rule estimates gestational age based on which of the following formulas?
a. add 7 days to LMP and count back 3 months T
b. subtract 7 days from LMP and count back 3 months
C. add 21 days to LMP and count back 3 months
d. subtract 21 days from LMP and count back 3 months
41. Duration of pregnancy is most correctly measured clinically by which of the following
units?
a. completed weeks since first day of LMP T
b. number of weeks, rounded to the nearest whole week, since the first day of LMP
c. completed weeks since estimated date of conception
d. number of weeks, rounded to the nearest whole week, since the estimated date of
conception
42. The relation of the long axis of the fetus to that of the mother is called
a. presentation
b. lie
T
c. attitude
d. posture
43. What is the maneuver used to facilitate delivery of the fetal head over the perineum in a controlled manner?
a. MacRoberts
b. Ragu
c. Ritgen T
d. Woods
44. During the third stage of labor, which of the following is NOT a sign of placental separation?
a. a gush of blood
b. uterus rises in the abdomen
c. umbilical cord protrudes farther out of the vagina
d. uterus becomes flaccid
T
45. What is baseline fetal bradycardia?

243

Gyna & Obstetrics Reveision by/Dr.adool

2008

a. < 90 bpm
b. < 100 bpm
c. < 110 bpm
d. < 120 bpm

46. What is baseline fetal tachycardia?


a. > 160 bpm T
b. > 170 bpm
c. > 180 bpm
d. > 190 bpm
47. Beat-to-beat baseline variability is predominantly regulated by which of the following?
a. maternal temperature
b. arterial chemoreceptors
c. autonomic nervous system T
d. density of myocyte acetylcholine receptors
48. Which of the following is the most common postpartum complication of precipitate labor?
a. hemorrhage T
b. endometritis
c. poor mother-neonate bonding
d. vulvar hematoma
49. Which of the following is a common cause of transverse lie?
a. placental abruption
c. postterm pregnancy
b. normal uterus
d. contracted pelvis T
50. Which of the following is NOT a risk factor for breech?
a. multiple fetuses
b. hydramnios
c. uterine anomalies
d. low parity T
51. Which of the following describes a frank breech presentation?
a. flexion of the hips and extension of the knees T
b. flexion of the hips and flexion of the knees
c. extension of the hips and flexion of the knees
d. extension of the hips and extension of the knees
52. Threatened abortion include the following except:
a) The cervix is closed.
b) Bleeding is mild.
c) Vaginal ultrasonography is contraindicated.T
d) Bed rest is strongly advisable.
e) Pregnancy continues in about 50% of cases.
53. Precipitate labor & delivery may lead to the following complications except:
a)
Uterine rupture.
b) Amniotic fluid embolism.
c)
Vagina & perineal tears.
d) Constriction ring in the uterus. T
e) Uterine atony.

244

Gyna & Obstetrics Reveision by/Dr.adool

2008

54. The following findings are suggestive of contracted pelvis during examination except:
a. Height 170 cm. T
b. Limping gait.
c. Kyphosis.
d. Pendulous abdoment.
e. Non engagement of the head.
55. Indication of trial of labor in patient with contracted pelvis include:
a) Non vertex presentation.
b) Moderate degree of cephalo pelvic disproportion. T
c) Previous C.S.
d) El derly primigravida.
e) Heart disease with pregnancy.
56. Fetal complications of contracted pelvis during labor include the following except:
a. Caput saccedaneum.
b. Over moulding.
c. Intra cranial Hge.
d. Fetal malformation. T
e. Fetal death.
57. Causes of contracted pelvis include the following except:
a. Rickets.
b. Osteo malcia.
c. Fractures of the pelvis.
d. T.B arthritis of the pelvic joints.
e. Diabetes. T
58. Management of asymptomatic bacteruria includes:
a) Expectant management.
b) Induction of labor.
c) Antibiotics. T
d) Diuretics.
59. Which of the following factors does NOT contribute to an acute urinary tract infection during
pregnancy, delivery, and the puerperium?
a) Compression of the ureter by the large uterus at the pelvic brim.
b) Increased ureteral tone and peristalsis. T
c) Symptomatic bacteriuria.
d) Bladder catheterization following delivery.
60. The pathophysiology of preeclampsia is characterized by:
a. Vasodilatation.
b. Vasospasm. T
c. Hemodilution.
d. Hypervolemia.

245

Gyna & Obstetrics Reveision by/Dr.adool

2008

61. Eclampsia is defined by:


a. Severe, unremitting headache.
b. Hyperreflexia.
c. Grand mal seizures. T
d. Petit mal seizures.
e. Visual scotomata.
62. Which of the following is most likely to be seen on a urinalysis in a patient with preeclampsia?
a. Proteinuria. T
b. Hematuria.
c. Glycosuria.
d. Ketonuria.
63. Which of the following is NOT included in the biophysical profile for the risk assessment of a
postterm fetus?
a. Fetal breathing.
b. Amniotic fluid volume.
c. Fetal tone.
d. Contraction stress test (CST). T
e. Fetal movement.
64. The most common site of metastatic disease in chorocarcinoma is:
a) Brian.
b) Liver.
c) Vagina.
d) Lung. T
e) Ovary.
65. Theca-lutein cysts are characterized by the following except:
a) Are best treated conservatively.
b) Are best treated surgically. T
c) Are seen occasionally in normal pregnancy.
d) Are seen in ovarian hypertimulation syndrome.
e) Liable to complications including hemorrhage, rupture and torsion.
66. The best contraceptive method for patients with gestational trophoblastic diseases:
a) Tubal ligaion.
b) Hysterectomy.
c) Combined oral contraceptive pills. T
d) Intra-uterine contraceptive device.
e) Subcutaneous implants.
67. Diagnosis of ectopic pregnancy can be ruled out if:
a. Transvaginal sonar showed no abnormalities in the adnexal region.
b. Serum hCG was negative. T
c. Patient did not miss her period.
d. Patient is using the IUCD for contraception.
e. Pelvic fluid by travaginal uterine ultrasound.

246

Gyna & Obstetrics Reveision by/Dr.adool

2008

68. Most common site of occurrence ectopic pregnancy is:


a. Isthmic or interstitial portion of the Fallopian tube.
b. Corneal end of the tube.
c. Ampullary portion of the fallopian tube. T
d. Fimrial end of the Fallopian tube.
e. Ovary.
69. Cephalopelvic disproportion in the absence of gross pelvic abnormality can be diagnosed by:
a. Ultrasound.
b. A maternal stature of <175 cm.
c. Trial of labor. T
d. Chest examination.
70. Endometrial hyperplasia could be expected in the following conditions:
a. Endodermal sinus tumor.
b. Cystic teratoma.
c. Polycystic ovary disease. T
d. Sertoli-Leydig cell tumor.
e. Dysgerminoma.
71. The character of normal cervical mucus at the time of ovulation is:
a. Highly viscous and turbid.
b. Spinnbarkiet test is less than 6 cm.
c. Contains low amount of crystals of sodium and potassium chloride.
d. Positive ferning test. T
72. Second degree uterine prolapse:
a) Can be diagnosed when the cervix protrudes through the vulval orifice. T
b) Is also known as complete procidentia.
c) Present with menorrhagia.
d) Causes lumbar backache.
73.
a)
b)
c)
d)

A cystocele:
Is always associated with stress incontinence.
Contains bladder. T
Is treated by fothergills operation.
Is best treated with a ring pessary.

74. An entrocele is best characterized by which of the following statements?


a) It is not a true hernia.
b) It is a herniation of the bladder floor into the vagina.
c) It is a prolapse of the uterus and vaginal wall outside the body.
d) It is a protrusion of the pelvic peritoneal sac and vaginal wall into the vagina. T
e)It is a herniation of the rectal and vaginal wall into the vagina.
75. The following statement about genital prolapse are correct:
a) After leforts operation, there is no intercourse. T
b) Grade 4-uterine prolapse is a procidentia.
c) Manchester repair includes cervical conization.

247

Gyna & Obstetrics Reveision by/Dr.adool

2008

d) The rubber ring pessary can be left for up to one year.


78. The diagnosis of PCOD:
a. History of oral contraception intake.
b. High FSH levels.
c. LH/FSH ratio of greater than 2.5. T
d. Absence of ovarian Graafian follicles on pelvic ultrasound.
e. Normal serum hormonal level.
79. Vaginal condidiasis is predisposed to by :
A) Diabetes mellitus.
B) Pregnancy.

C) Oral contraceptives.
D) All of the above. T

80. Menstrual disorders encountered with uterine myomata include the following, EXCEPT:
A) Menorrhagia.
B) Metrorrhagia.

C) Hypomenorrhea. T
D) Polymenorrhea.

81. Placenta is formed by:


A) Decidua basalis and chorion frondosum T
B) Decidua basalis and chorion laeve
C) Decidua capsularis and chorion frondosum
D) Decidua vera and chorion frondosum
82. Fertilization of the ovum by a sperm normally occurs in the:
A) Fimbria of the fallopian tube.
B)
Isthmus of the fallopian tube
C)
Ampullary region of the fallopian tube. T
D) Uterine cornua.
E)
Uterine trigone.
83. During the third stage of labor,
A) The baby is born.
B) Maximal cervical dilation occurs.
C) The beginning of cervical dilation occurs.
D) The regular onset of contractions occurs.
E) The placenta is delivered. T
84. Most common cause of first trimester abortion is:
A) Chromosomal abnormalities. T
B) Syphilis.
C) Rhesus isoimmunisation.
D) Cervical incompetence
85. A 21-year-old woman presents as an emergency with a four-hour history of a lower abdominal pain
and bright red vaginal blood loss. She has not had menstrual period for nine weeks and has a
positive home pregnancy test one week ago. On vaginal examination, the uterus is tender and

248

Gyna & Obstetrics Reveision by/Dr.adool

2008

bulky. The cervical os is open. What is the most likely diagnosis?


A) Inevitable miscarriage T
B) Pelvic inflammatory disease
C) Normal pregnancy
D) Missed abortion
E) Septic abortion
86 - The gold standard in diagnosing ectopic pregnancy is:
A) Laparoscopy T
D) US
B) Culdocentesis
E) Progesterone
C) Beta HCG
87- What is the most common side effect with methotrexate (MTX) therapy for ectopic pregnancy?
A) Transient pelvic pain 3 - 7 days after starting treatment T
B) Stomatitis
C) Bone marrow suppression
D) Gastritis
88- Pregnant patient with HELLP syndrome, all are found EXCEPT:
A) Hemolysis
B) Raised bilirubin
C) Decreased platelets
D) Raised liver enzymes
E) Hepatosplenomegaly T
89. Which of the following is UNLIKELY to be a complication of magnesium administration in preeclampsia- Eclampsia?
A) Decreased respiratory rate
B) Decreased beat to beat variability
C) Decreased deep tendon reflexes
D) Hypotension
E) Preterm labor T
90- Gestational diabetes is not associated with:
A) Macrosomia
B) Neural tube defects T
C) Altered GTT
D) Hydramnios
91-

Which of the following does NOT occur as a complication of diabetes mellitus in the pregnant
patient?
A) Increased risk of oligohydramnios T
B) Greater risk of fetal death in the third trimester
C) Progression of retinopathy
D) Increased CS rate

249

Gyna & Obstetrics Reveision by/Dr.adool

2008

92- All are neonatal complications of diabetes in pregnancy EXCEPT:


A) Hypocalcemia
D) hyperviscosity syndrome
B) Hypoglycemia
E) hyperthyroidism T
C) Hyperbilirubinemia
93- Most common cause of death in a pregnant diabetic is
A) Renal failure due to chronic pyelonephritis
B) Renal failure due to diabetic nephropathy
C) Diabetic keto - acidosis T
D) Hyperosmolar coma
94- Diabetes in pregnancy , all are correct EXCEPT:
A) Is associated with a reduced incidence of fetal abnormality. T
B) Is associated with an increased risk of respiratory distress syndrome in the
newborn.
C) Leads to insulin requirements that are greater at 28 weeks gestation than prior to
pregnancy.
D) Is associated with neonatal hypoglycaemia.
E) Is associated with shoulder dystocia at delivery.
95- Initial treatment of women diagnosed with gestational diabetes mellitus primarily consists of:
A) low carbohydrate diet and exercise T
B) metformin
C) insulin
D) glyburide
96- The insulin requirement is decreased in a pregnant diabetic patient:
A) with toxemia of pregnancy.
B) With successive pregnancies.
C) Immediately after delivery. T
D) During the second trimester.
E) During the third trimester.
97- The earliest detection of gestation diabetes is at:
A) 28-32 wks
B) 24-28wks T
C) 16-20 wks
D) First trimester

250

Gyna & Obstetrics Reveision by/Dr.adool

2008

251

Gyna & Obstetrics Reveision by/Dr.adool

2008

252

Gyna & Obstetrics Reveision by/Dr.adool

2008

S-ar putea să vă placă și